current affairs -february 1-15, 2019 · 15-02-2019  · 10, theme: ‘sadak suraksha-jeevan...

130
GKTODAY Current Affairs [PDF] -February 1-15, 2019 This is a dynamic PDF E-book by GKToday. The latest version of this book can be downloaded from this link Published by: GKTODAY.IN GKToday © 2019 | All Rights Reserved The authors and publisher have made every effort to ensure that the information in this E-book is correct. However, GKToday does not assume and hereby disclaims any liability to any party for any loss, damage, or disruption caused by errors or omissions, whether such errors or omissions result from negligence, accident, or any other cause. This document is a property of GKToday. Unauthorized Duplication is not allowed. sri vishnu charan | [email protected] | https://t.me/PDF4Exams https://t.me/IAS201819 https://t.me/PDF4Exams https://t.me/TheHindu_Zone_official

Upload: others

Post on 24-Mar-2020

3 views

Category:

Documents


1 download

TRANSCRIPT

Page 1: Current Affairs -February 1-15, 2019 · 15-02-2019  · 10, theme: ‘Sadak Suraksha-Jeevan Raksha’ National Testing Agency (NTA) launches mobile app through which students can

GKTODAY

Current Affairs [PDF] -February 1-15, 2019

This is a dynamic PDF E-book by GKToday. The latest version of this book can be downloaded from this linkPublished by: GKTODAY.IN

GKToday © 2019 | All Rights Reserved

The authors and publisher have made every effort to ensure that the information in this E-book iscorrect. However, GKToday does not assume and hereby disclaims any liability to any party for anyloss, damage, or disruption caused by errors or omissions, whether such errors or omissions resultfrom negligence, accident, or any other cause.This document is a property of GKToday. Unauthorized Duplication is not allowed.

sri vishnu charan | [email protected] |

https://t.me/PDF4Examshttps://t.me/IAS201819 https://t.me/PDF4Exams

https://t.me/TheHindu_Zone_official

Page 2: Current Affairs -February 1-15, 2019 · 15-02-2019  · 10, theme: ‘Sadak Suraksha-Jeevan Raksha’ National Testing Agency (NTA) launches mobile app through which students can

Current Affairs [PDF] -February 1-15, 2019

© 2019 GKToday | All Rights Reserved | https://www.gktoday.in 2

Contents......................... .. .. ....... ......... ............................................................

News Headlines: February 1-15, 2019 3 ......................................................................................................................................................................... February 1, 2019 12 ............................................................................................................................................................................................................. February 2, 2019 18 ............................................................................................................................................................................................................ February 4, 2019 25 ........................................................................................................................................................................................................... February 5, 2019 30 ........................................................................................................................................................................................................... February 6, 2019 36 ........................................................................................................................................................................................................... February 7, 2019 43 ........................................................................................................................................................................................................... February 8, 2019 47 ........................................................................................................................................................................................................... February 9, 2019 55 ........................................................................................................................................................................................................... February 10, 2019 60 ......................................................................................................................................................................................................... February 11, 2019 64 .......................................................................................................................................................................................................... February 12, 2019 70 ......................................................................................................................................................................................................... February 13, 2019 73 .......................................................................................................................................................................................................... February 14, 2019 77 ......................................................................................................................................................................................................... February 15, 2019 84 ......................................................................................................................................................................................................... Multiple Choice Questions 88 ..........................................................................................................................................................................................

sri vishnu charan | [email protected] |

https://t.me/PDF4Examshttps://t.me/IAS201819 https://t.me/PDF4Exams

https://t.me/TheHindu_Zone_official

Page 3: Current Affairs -February 1-15, 2019 · 15-02-2019  · 10, theme: ‘Sadak Suraksha-Jeevan Raksha’ National Testing Agency (NTA) launches mobile app through which students can

Current Affairs [PDF] -February 1-15, 2019

© 2019 GKToday | All Rights Reserved | https://www.gktoday.in 3

News Headlines: February 1-15, 2019Current Affairs – News Headlines: February 1, 2019

IndiaPresident addresses the joint sitting of two Houses of Parliament on the first day of Budget sessionDoordarshan’s travelogue programme ‘Rag Rag Mein Ganga’ and ‘Meri Ganga’ quiz series launchedHaryana: BJP wins Jind Assembly seat in bypoll, raises its tally to 48 in 90-member AssemblyRajasthan: Congress wins Ramgarh Assembly seat in bypoll, raises its tally to 100 in 200-memberAssemblyRajasthan govt. to give monthly unemployment allowance of Rs 3,500 to girls and Rs 3,000 to boysfrom March 1Delhi Police launches an online facility for granting arms licensesAcquisition of 5,000 Milan anti-tank guided missiles for the Army approved by Defence MinisterAssam, AP and Odisha occupy top-3 slots in the ranking of best practices followed by states inBudget formulation: Transparency International

Economy & CorporateGovernment revises GDP growth rate for 2017-18 upward to 7.2% from 6.7% earlierGrowth of eight core industries in April-December at 4.8% compared to 3.9% in same period of2017-18Subsidised LPG price cut by Rs 1.46; non-subsidised rate reduced by Rs 30 a cylinderRBI removes BoI, BoM, OBC from Prompt Corrective Action framework, a move that will lift lendingrestrictions on them34th edition of the India International Leather Fair in Chennai from Jan 31 to Feb 2WhatsApp launches “Startup India-WhatsApp Grand Challenge”; top-5 start-ups to receive a totalgrant of $250,000

WorldEuropean Parliament recognizes US-backed opposition leader Juan Guaido as the interim Presidentof VenezuelaWorld’s top-2 shipbuilders — South Korea’s Hyundai Heavy Industries and Daewoo Shipbuilding &Marine Engineering announce mergerPak successfully test-fires 70 km range ballistic missile ‘Nasr’

SportsNZ (93/2 in 14.4) beat India (92/10 in 30.5) by 8 wickets in 4th ODI of 5-match series at Seddon Parkin HamiltonHaryana Hammers beat Punjab Royals 6-3 in final to win Pro Wrestling League (PWL) season 4 titlein Greater NoidaCoca-Cola enters into 5-year partnership with International Cricket Council till 2023

Current Affairs – News Headlines: February 2, 2019India

20th edition of Bharat Rang Mahotsav being organised by the National School of Drama (NSD) inNew Delhi from Feb 1 to 21Haryana: 33rd Surajkund International Crafts Mela being organised in Faridabad from Feb 1 to 17MP: Monthly honorarium for priests serving at govt.-managed temples increased of Rs 3,000 fromRs 1,000Two IAF pilots killed after their Mirage fighter aircraft crashed during take-off at HAL airport inBengaluru

Interim BudgetGovt seeks Parliament’s nod (Vote on Account) for spending Rs 34.17 lakh crore in April-July 2019In 2018-19, fiscal deficit at 3.4% of GDP, current account deficit (CAD) at 2.5% of GDPDirect tax (corporate tax & personal income tax) collection in 2018-19 at Rs 12 lakh croreDirect tax collection target for 2019-20 at Rs 13.80 lakh crore

sri vishnu charan | [email protected] |

https://t.me/PDF4Examshttps://t.me/IAS201819 https://t.me/PDF4Exams

https://t.me/TheHindu_Zone_official

Page 4: Current Affairs -February 1-15, 2019 · 15-02-2019  · 10, theme: ‘Sadak Suraksha-Jeevan Raksha’ National Testing Agency (NTA) launches mobile app through which students can

Current Affairs [PDF] -February 1-15, 2019

© 2019 GKToday | All Rights Reserved | https://www.gktoday.in 4

Disinvestment target at Rs 90,000 crore for 2019-20Income tax exemption limit raised from Rs 2.5 lakh to Rs 5 lakh for individual taxpayersStandard tax deduction for salaried persons raised from Rs 40,000 to Rs 50,0002% interest subvention for MSMEs with Rs 1 crore loanNo tax deducted at source (TDS) on interest income up to Rs 40,000 per annum (Rs 10,000 earlier)No TDS on annual rental income up to Rs 2.4 lakh (Rs 1.8 lakh earlier)Pradhan Mantri Kisan Samman Nidhi: A cash transfer scheme that will provide direct incomesupport of Rs 6,000 per year to farmers with landholdings of up to two hectaresPradhan Mantri Shram Yogi Mandhan: A pension scheme for unorganised sector workers; assures amonthly pension of Rs 3,000 when a worker attains 60 years of ageRashtriya Kamdhenu Aayog will be set up for the welfare of cows with funding of Rs 750 crore fromRashtriya Gokul MissionDefence allocation raised by 7.9% to over Rs 3.18 lakh crore

Economy & CorporateForeign exchange reserves rise USD 1.497 billion to reach USD 398.178 billion for the week toJanuary 25

WorldAustralia: Behrouz Boochani wins USD 72,600 Victorian Prize for Literature for his book “No FriendBut the Mountains: Writing from Manus Prison”UNSC adopts resolution to extend sanctions against the Central African Republic (CAR) for anotheryearDAVIS CUPItaly take 2-0 lead over India on Day 1 of Davis Cup World Group qualifier tie at KolkataMatteo Berrettini beats Prajnesh Gunneswaran 6-4, 6-3Andreas Seppi beats Ramkumar Ramanathan 6-4, 6-2

SportsIndian captain Mithali Raj becomes 1st woman cricketer to play 200 one-day internationals

Current Affairs – News Headlines: February 3, 2019India

Former MP DGP Rishi Kumar Shukla appointed new Director of CBIHaryana: Country’s 22nd AIIMS will be established in Rewari districtGujarat: Asha Patel, Congress MLA from Unjha constituency, resigns from the partyWest Bengal: PM launches 294-km-long Andal-Sainthia-Pakur-Malda and Khana-Sainthia electrifiedrailway line at Thakurnagar

Economy & CorporateRevenue collection under GST stood at Rs 1,02,503 crore in in January 2019

WorldWorld Wetlands Day observed on February 2 with theme as ‘Wetlands and Climate Change’Russia suspends participation in Intermediate-Range Nuclear Forces agreement of 1987 after similarmove by USIran announces ‘successful test’ of 1350-km range Hoveizeh cruise missileGoogle starts notifying users about the shutdown of Google+ in the coming two months

SportsQatar beat Japan 3-1 in final to win 2019 AFC Asian Cup football tournament in Abu DhabiDAVIS CUP TENNISItaly beat India 3-1 in Kolkata to enter Davis Cup World FinalsRohan Bopanna & Divij Sharan beat Simone Boelleli & Matteo Berrettini in Doubles 4-6, 6-3, 6-4Andreas Seppi beat Prajnesh Gunneswaran 6-1, 6-4 in first reverse SinglesItaly had taken 2-0 lead by winning both Singles matches on February 1

sri vishnu charan | [email protected] |

https://t.me/PDF4Examshttps://t.me/IAS201819 https://t.me/PDF4Exams

https://t.me/TheHindu_Zone_official

Page 5: Current Affairs -February 1-15, 2019 · 15-02-2019  · 10, theme: ‘Sadak Suraksha-Jeevan Raksha’ National Testing Agency (NTA) launches mobile app through which students can

Current Affairs [PDF] -February 1-15, 2019

© 2019 GKToday | All Rights Reserved | https://www.gktoday.in 5

Current Affairs – News Headlines: February 3-4, 2019India

J&K: PM lays foundation stones of 2 AIIMS at Awantipora in Pulwama district and Vijaypur inSamba districtJ&K: PM launches University of Ladakh at Leh, the 1st university in Ladakh regionCJI Ranjan Gogoi and AP CM N. Chandrababu Naidu inaugurate judicial complex in Amravati thatwill serve as an interim High Court for the stateJustice Gogoi and Naidu lay foundation stone for a permanent High Court for AP to be built inGuntur district6 dead after 9 coaches of the Delhi-bound Seemanchal Express derailed in Vaishali district of BiharVisually impaired and blind voters to be provided with braille voter slips in the Lok Sabha electionsManipuri filmmaker Aribam Syam Sharma to return Padma Shri in protest against the Citizenship(Amendment) Bill 2016

Economy & CorporateThe Sovereign Gold Bond Scheme 2018-19 (Series 6) will be opened February 4Country’s tourism sector fetched USD 234 billion revenue (+19%) in 2018: Union Tourism Minister K.J. Alphons

WorldFormer Law Minister Nilambar Acharya appointed as Nepal’s new Ambassador to India

SportsIndia 252/10 in 49.5) beat NZ (207/10 in 44.1) by 35 runs in 5th ODI at Wellington, win 5-match series4-1West Indies (306, 17/0) beat England (187, 132) by 10 wickets in 2nd test at North Sound, Antigua; win3-test series 2-0India’s Ramit Tandon beats Egypt’s Mohamed El Sherbini in the final 11-4, 11-7, 2-11, 11-2 to win Men’sSingles title at Seattle Open squash

Current Affairs – News Headlines: February 5, 2019India

Ministry of Road Transport & Highways observing 30th Road Safety Week Campaign from Feb 4 to10, theme: ‘Sadak Suraksha-Jeevan Raksha’National Testing Agency (NTA) launches mobile app through which students can take mock tests ontheir smartphonesOver five crore people take Holy dip at Kumbh in Prayagraj on Mauni AmavasyaHaryana government to double the monthly honorarium given to ‘Lambardars’ (tax collectors) fromRs 1,500 to Rs 3,000Filmmaker Govind Nihalani to receive Kalamaharshi Baburao Painter Award at KolhapurInternational Film Festival: Feb 7-14Noted freedom fighter and former Odisha MLA Priyanath Dey dies in Bhubaneswar at the age of 97

Economy & CorporateFDI in India declined 11% to $ 22.66 billion during April-September period of the current fiscal: Govt.Fiscal deficit touched 112.4% of the full-year budget target of Rs 6.24 trillion at the end of DecemberEY selects Azim Premji, Chairman of Wipro Ltd, for its Lifetime Achievement AwardUK Home Secretary orders extradition of Vijay Mallya to India

WorldWorld Cancer Day observed on February 4 with its theme as ‘I Am and I Will’Sri Lanka celebrated 71st Independence Day on February 4Former San Salvador mayor Nayib Bukele wins Presidential election in El SalvadorChinese lunar calendar: ‘Year of the Dog’ ended on Feb 4; ‘Year of the Pig’ begins on Feb 5

SportsIndian cricket team ranked 2nd in ICC ODI rankings behind England

sri vishnu charan | [email protected] |

https://t.me/PDF4Examshttps://t.me/IAS201819 https://t.me/PDF4Exams

https://t.me/TheHindu_Zone_official

Page 6: Current Affairs -February 1-15, 2019 · 15-02-2019  · 10, theme: ‘Sadak Suraksha-Jeevan Raksha’ National Testing Agency (NTA) launches mobile app through which students can

Current Affairs [PDF] -February 1-15, 2019

© 2019 GKToday | All Rights Reserved | https://www.gktoday.in 6

Current Affairs – News Headlines: February 6, 2019India

PM Modi meets Prince Albert II of Monaco in New DelhiCommander-in-Chief of Indonesian National Defence Forces, Air Chief Marshal Hadi Tjahjantomeets Defence Minister Nirmala Sitharaman in New Delhi

Economy & CorporateRBI imposes Rs. 2 crore penalty on UCO Bank for non-compliance of apex bank’s instructions on‘frauds-classification & reporting’SBI’s regional head in UK Sanjiv Chadha honoured with the ‘Freedom of the City of London’ awardfor promoting bilateral relationsNikkei India Services Business Activity Index falls from 53.2 in December to 52.2 in JanuaryAsia LPG Summit organised by World LPG Association (WLPGA) in New Delhi

World18 EU countries recognise Venezuela’s National Assembly speaker Juan Guaido as country’s interimPresidentSiamese fighting fish named Thailand’s national aquatic animal

SportsFormer cricket coach Rajabhau Oak dies in Pune at the age of 85

Current Affairs – News Headlines: February 7, 2019India

India’s communication satellite GSAT-31 launched from spaceport in French Guiana on Ariane 5launch vehiclePart two of Darwaza Band campaign launched in Mumbai by Swachh Bharat Mission Grameen tofurther promote the use of toilets and to sustain the open defecation free status of villages acrossIndia‘Parmanu Tech 2019’ conference organised by the Ministry of External Affairs and Department ofAtomic Energy (DAE) in New Delhi; discussed issues related to Nuclear Energy and RadiationTechnologies92-year old Father Franois Laborde conferred Légion d’Honneur, the highest civilian award ofFrance, in recognition of his work for specially-abled children in West BengalBiju Janata Dal leader Ladu Kishore Swain, who represented Aksa Lok Sabha seat in Odisha, dies inBhubaneswar at 71Himachal Pradesh: CM Jai Ram Thakur flags off Paprola-Pathankot Express Train in Kangra districtRajasthan Govt. launches ‘Rajiv Gandhi Career Portal’ for students from classes 9 to 12

Economy & CorporateAadhaar-PAN linking mandatory for filing income tax return: Supreme CourtBank liable for unauthorised withdrawal even if customers did not respond to SMS alert: Kerala HCRBI imposes penalties on UCO Bank, Axis Bank and Syndicate Bank for not complying with differentregulatory normsCabinet approves proposal for Official Amendments to Banning of Unregulated Deposit SchemesBill, 2018Cabinet approves abolition of Ombudsman for direct, indirect taxesCabinet approves amendment to the Cinematograph Act for imposing strict penalties to curb filmpiracyChief Managing Director (CMD) of Air India Pradeep Singh Kharola appointed Civil AviationSecretary

WorldMacedonia signs accord to join NTOSafer Internet Day observed on February 5 with the theme ‘Together for a better internet’International Day of Zero Tolerance for Female Genital Mutilation observed on February 6

sri vishnu charan | [email protected] |

https://t.me/PDF4Examshttps://t.me/IAS201819 https://t.me/PDF4Exams

https://t.me/TheHindu_Zone_official

Page 7: Current Affairs -February 1-15, 2019 · 15-02-2019  · 10, theme: ‘Sadak Suraksha-Jeevan Raksha’ National Testing Agency (NTA) launches mobile app through which students can

Current Affairs [PDF] -February 1-15, 2019

© 2019 GKToday | All Rights Reserved | https://www.gktoday.in 7

SportsNZ (219/6) beat India (139/10 in 19.2) by 80 runs in 1st T20 International at Wellington

Current Affairs – News Headlines: February 8, 2019India

United States approves sale of 2 missile defence systems to India for security of planes flying PM andPresidentForeign Minister of Bangladesh A. K. Abdul Momen calls on PM Modi in New DelhiAssam govt. to give 1 ‘tola’ (10 grams) of gold to brides of poor families under ‘Arundhati’ scheme

Monetary Policy ReviewMonetary Policy Committee (MPC) cuts policy repo rate under the liquidity adjustment facility (LAF)by 25 basis points from 6.5% to 6.25%Reverse repo rate under the LAF stands adjusted to 6%, and the marginal standing facility (MSF) rateand the Bank Rate to 6.5%MPC changes the monetary policy stance from calibrated tightening to neutral6-member MPC voted 4:2 in favour of the rate cut, while the decision to change policy stance wasunanimousRetail inflation forecast revised downwards to 2.8% in Q4:2018-19, 3.2-3.4% in H1:2019-20 and 3.9% inQ3:2019-20Economic growth rate forecast for 2018-19 at 7.4%Limit of collateral-free agricultural loans raised to Rs 1.6 lakh from the current Rs 1 lakh

Economy & CorporateUnion Cabinet approves establishment of a unified authority for regulating all financial services inInternational Financial Services Centres (IFSCs) in IndiaIndia ranked 36th among 50 economies on International Intellectual Property (IP) Index of the USChamber of Commerce’s Global Innovation Policy Center (GIPC); the US, the UK, Sweden, France andGermany are top-5 economiesItalian super sports carmaker Lamborghini launches Huracan Evo in India priced at Rs 3.73 crore(ex-showroom).

WorldUS President Donald Trump nominates economist David Malpass as next President of World BankInstagram’s new feature to blur self-harming content on app until the viewer opts in

SportsVidarbha (312, 200) beat Saurashtra (307, 127) by 78 runs in final to win their 2nd successive RanjiTrophy title at Nagpur

Current Affairs – News Headlines: February 9, 2019India

Helina, the helicopter-launched version of anti-tank guided missile ‘Nag’ with a hit range of 7-8 km,test fired in OdishaJ&K Govt. declares Ladakh as the third administrative division of the state (comprising Leh andKargil districts) after Jammu and Kashmir‘Law, Justice and Judicial Power – Justice PN Bhagwati’s Approach’, written by CJI Ranjan Gogoi,released‘Ramayan Retold With Scientific Evidences’ book written by former bureaucrat Saroj Bala, releasedIndia and Bangladesh sign MOU for training 1800 Bangladesh civil servants in National Centre ForGood GovernanceCentre announces Great Indian Bustard as mascot for 13th UN Convention on the Conservation ofMigratory Species (CMS) (COP 13), scheduled to be held in Gujarat in February 2020Election Commission launches Voter Verification and Information Programme (VVIP)Health and Family Welfare Ministry conducts 8th round of National Deworming Day (NDD) acrossthe country

sri vishnu charan | [email protected] |

https://t.me/PDF4Examshttps://t.me/IAS201819 https://t.me/PDF4Exams

https://t.me/TheHindu_Zone_official

Page 8: Current Affairs -February 1-15, 2019 · 15-02-2019  · 10, theme: ‘Sadak Suraksha-Jeevan Raksha’ National Testing Agency (NTA) launches mobile app through which students can

Current Affairs [PDF] -February 1-15, 2019

© 2019 GKToday | All Rights Reserved | https://www.gktoday.in 8

BSP chief Mayawati should pay the govt. for using public money for erecting statues of herself andher party’s symbol elephant: Supreme CourtGujarat: Asha Patel, who resigned as a Congress MLA on Feb 2, joins BJP

Economy & CorporateCountry’s foreign exchange reserves increased by $2.063 billion to $400.24 billion in the week toFebruary 161 individuals declared an income of more than Rs 100 crore during the assessment year 2017-18

WorldIndia Pavilion at the Berlin International Film Festival, (Berlinale) 2019 inauguratedFilmmaker Deepa Mehta honoured with Lifetime Achievement Award Academy by CanadianCinema & TelevisionActress Priyanka Chopra gets a wax figure at Madame Tussauds museum in New YorkBritish actor Albert Finney dies at the age of 82

SportsIndia (162/3 in 18.5) defeat NZ (158/8 in 20) by 7 wickets in 2nd T20I at Eden Park in Auckland, serieslevel at 1-1Rohit Sharma (2,288 runs) overtakes Martin Guptill of NZ as the leading run-scorer in T20IsBadminton player P. V. Sindhu signs 4-year sponsorship deal worth Rs. 50 crore with Chinese sportsbrand Li Ning

Current Affairs – News Headlines: February 10, 2019India

Arunachal Pradesh: PM launches DD Arun Prabha 24×7 satellite television channel in ItanagarArunachal Pradesh: PM inaugurates 110 MW Pare Hydroelectric Plant and upgraded Tezu airportArunachal Pradesh: PM lays foundation stone of the Sela Tunnel Project that would provide all-weather connectivity to Tawang and forward areasPM unveils statue of Tripura’s last ruler, Maharaja Bir Bikram Kishore Manikya, at airport inAgartalaTripura: PM inaugurates 23.32-km railway track and new complex of Tripura Institute ofTechnology (TIT) at NarsingarhPM declares 100% electrification of all households in Arunachal PradeshShillong: Kolkata Police Commissioner Rajeev Kumar questioned by CBI over Saradha chit fundscamCentre and Gujarat govt. announce ₹85 crore Asiatic Lion Conservation Project‘Undaunted-Save the Idea of India’ by former Union Minister P. Chidambaram launched

Economy & CorporateMEA will help textiles ministry to promote silk in global markets: Sushma Swaraj

WorldFrench cartoonist, author and illustrator Tomi Ungerer dies at the age of 87 in Ireland

SportsMeghalaya govt. to rename Dikkibandi stadium in Tura town after ex-Lok Sabha Speaker Purno A.SangmaArgentine and Cardiff City club footballer Emiliano Sala, who was missing from Jan 21, found dead ina plane crash in UK

Current Affairs – News Headlines: February 11, 2019India

1st LAWASIA Human Rights Conference organised in New Delhi on February 9-10Sivagiri Spiritual Tourism Circuit inaugurated at Varkala, Kerala by MoS for Tourism (I/C AlphonsKannanthanamUP CM Yogi Adityanath flags off 33 ‘Sarthi Sandesh Vahini’ vehicles from Gorakhpur to promotefamily planning

sri vishnu charan | [email protected] |

https://t.me/PDF4Examshttps://t.me/IAS201819 https://t.me/PDF4Exams

https://t.me/TheHindu_Zone_official

Page 9: Current Affairs -February 1-15, 2019 · 15-02-2019  · 10, theme: ‘Sadak Suraksha-Jeevan Raksha’ National Testing Agency (NTA) launches mobile app through which students can

Current Affairs [PDF] -February 1-15, 2019

© 2019 GKToday | All Rights Reserved | https://www.gktoday.in 9

Economy & CorporateTamil Nadu: PM inaugurates crude oil pipeline from Chennai Port to Chennai PetroleumCorporation Limited’s Manali refineryTamil Nadu: PM inaugurates passenger service for a10 km section of Chennai Metro connecting AG-DMS and Washermenpet metro stationsTamil Nadu: PM inaugurates ESIC Hospital Chennai and BPCL’s Ennore Coastal TerminalAP: PM inaugurates 1.33 million metric tonnes capacity Visakhapatnam Strategic Petroleum Reservefacility of the Indian Strategic Petroleum Reserve Ltd (ISPRL)Karnataka: PM inaugurates 1.5 MMT Mangalore Strategic Petroleum Reserve (SPR) facility and 2.5MMT Padur SPR facility of ISPRLKarnataka: PM inaugurates City Gas Distribution Project at DharwadPETROTECH – 2019 being organised by Petroleum and Natural Gas Ministry at Greater Noida, UPfrom February 10 to 12Cremica Food Park, 1st Mega Food Park of Himachal Pradesh, inaugurated in Una District by UnionMinister for Food Processing Industries Harsimrat Kaur Badal4 Chinook military helicopters, made by Boeing for the Indian Air Force, arrive at Mundra port inGujarat

WorldEgypt’s President Abdel-Fattah el-Sissi elected Chairman of African Union at the summit in AddisAbaba, EthiopiaIndian historian Sanjay Subrahmanyam among winners of Israel’s Dan David PrizeWorld Government Summit being organised in Dubai (UAE) from February 10 to 12

SportsMen’s cricket: NZ (212/4 in 20) beat India (208/6 in 20) by 4 runs in 3rd T20I at Seddon Park,Hamilton, win 3-mach series 2-1Women’s cricket: NZ (161/7 in 20) beat India (159/4 in 20) by 2 runs in 3rd T20I at Seddon Park,Hamilton to win 3-match series 3-0Shane Warne named brand ambassador of Rajasthan Royals for next edition of IPLPankaj Advani clinches his 9th Men’s National Snooker title at IndoreRailway Sports Promotion Board beat Punjab 3-2 in final to win Senior National Men’s HockeyChampionship at GwaliorFrance’s Corentin Moutet wins Men’s Singles title at Chennai Open ATP Challenger tennistournament

Current Affairs – News Headlines: February 12, 2019India

World Sustainable Development Summit 2019 being organized by The Energy and ResourcesInstitute (TERI) in New Delhi from February 11 to 13Bill to make NRI men register marriages done in India within 30 days introduced in Rajya Sabha;failure to register may result in passport getting impounded or revokedEarthen drum Ghumot to be declared Goa’s heritage musical instrumentRajasthan Assembly passes bills to end the minimum education criterion for panchayat and civicpolls candidates3 billionth meal by NGO Akshaya Patra Foundation served at a school in Vrindavan (UP)Akshaya Patra foundation serves as implementing partner of Mid-Day Meal Scheme in twelve statesNational Conference on Unani Medicine organised by the Central Council for Research in UnaniMedicine in New DelhiMinistry of External Affairs organises training course for Nepal Govt. officials under IndianTechnical and Economic Cooperation (ITEC) programme

Economy & CorporateLok Sabha passes Interim Budget for 2019-20

sri vishnu charan | [email protected] |

https://t.me/PDF4Examshttps://t.me/IAS201819 https://t.me/PDF4Exams

https://t.me/TheHindu_Zone_official

Page 10: Current Affairs -February 1-15, 2019 · 15-02-2019  · 10, theme: ‘Sadak Suraksha-Jeevan Raksha’ National Testing Agency (NTA) launches mobile app through which students can

Current Affairs [PDF] -February 1-15, 2019

© 2019 GKToday | All Rights Reserved | https://www.gktoday.in 10

PETROTECH-2019, India’s flagship hydrocarbon conference, being organised in Greater Noida (UP)from February 10 to 12Petroleum & Natural Gas Ministry launches general guidelines for setting up of Dealer OwnedDealer Operated (DODO) CNG stationsNational Productivity Council (NPC) to celebrate Productivity Day on February 12NPC to celebrate National Productivity Week from February 12 to 18Unani Day celebrated on February 11: the birth anniversary of Hakim Ajmal Khan (1868-1927)

WorldBAFTA (British Academy of Film and Television Arts) Awards: ‘Roma’ directed by Alfonso Cuarn isthe Best Film2019 Grammy Awards: ‘This Is America’ by Childish Gambino is ‘Record of the Year’Dera Baba Nanak land check post designated as immigration centre for pilgrims visiting KartarpurSahib gurdwara in PakistanRuchira Kamboj appointed as the Ambassador of India to Bhutan

SportsPetroleum Sports Promotion Board (PSPB) retain National Team Chess Championship in both men’sand women’s categoriesAirports Authority of India (AAI) win Inter State-Inter Zonal Badminton Championships title inGuwahatiClouded Leopard will be mascot for National Games 2022 in MeghalayaTennis: Prajnesh Gunneswaran is the only Indian in top-100 of men’s singles

Current Affairs – News Headlines: February 13, 2019India

Supreme Court refers to seven-judge bench the issue of determining the correctness of minoritystatus granted to Aligarh Muslim UniversityCentre releases publication titled ‘India – Spearheading Climate Solutions’ highlighting actionstaken by the country to combat climate changePresident unveils life size portrait of Atal Bihari Vajpayee in ParliamentDelhi: 17 dead in fire at Hotel Arpit Palace in Karol BaghTiger spotted in Gujarat after 3 decades in Mahisagar districtIndian Army holds annual ‘Exercise Topchi’ at Deolali Camp near NashikIndia signs contract to procure 72,400 Sig Sauer Assault Rifles from the USChhattisgarh: New Congress govt. renames 5 schemes named after Pandit Deendayal Upadhyaya byprevious BJP Govt.Haryana: PM inaugurates an ESIC medical college and 510-bedded hospital in FaridabadHaryana: PM inaugurates National Cancer Institute at Badhsa in Jhajjar districtAssam govt. to give free textbooks to girls up to degree level under Gyan Deepika schemeProminent Telugu film director-producer Vijaya Bapineedu dies in Hyderabad at 83Grand Mufti of Jammu and Kashmir Bashiruddin Ahmad dies in Srinagar at 82

Economy & CorporateConsumer Price Index (CPI)-based retail inflation declines to a 19-month low of 2.05% in January2019Factory output as measured in terms of the Index of Industrial Production (IIP) grows 2.4% inDecember 2018Productivity Day observed by National Productivity Council (NPC) on February 12, theme: ‘CircularEconomy for Productivity & Sustainability’National Productivity Week: February 12-18Siddhartha Lal, MD & CEO of Eicher Motors, named EY Entrepreneur of the Year 2018The Institute of Chartered Accountants of India (ICAI) elects Prafulla P. Chhajed as its President for2019-20

sri vishnu charan | [email protected] |

https://t.me/PDF4Examshttps://t.me/IAS201819 https://t.me/PDF4Exams

https://t.me/TheHindu_Zone_official

Page 11: Current Affairs -February 1-15, 2019 · 15-02-2019  · 10, theme: ‘Sadak Suraksha-Jeevan Raksha’ National Testing Agency (NTA) launches mobile app through which students can

Current Affairs [PDF] -February 1-15, 2019

© 2019 GKToday | All Rights Reserved | https://www.gktoday.in 11

‘The Third Pillar: How Markets and the State Leave the Community Behind’ authored by former RBIGovernor Raghuram RajanTRAI gives time till March 31 for consumers to opt for channels of their choice under the newframework for broadcasting and cable services

WorldThailand hosting multi-nation Cobra Gold military exercise from February 12 to 22Sibghatullah Mujadidi, former President of Afghanistan, dies in Kabul at 93

SportsGordon Banks, England’s goalkeeper during their triumphant 1966 World Cup campaign, dies at 81

Current Affairs – News Headlines: February 14, 2019India

Budget session of Parliament concludes, The Muslim Women (Protection of Rights on Marriage) Billand The Citizenship (Amendment) Bill to lapseLok Sabha passes the Jallianwala Bagh National Memorial (Amendment) Bill, 2018Parliament passes Personal Laws (Amendment) Bill, 2019 ending leprosy as grounds of divorceRajasthan govt. passes a bill to provide 5% quota in jobs and educational institutions to Gujjars and 4other communities1,986 migrants of all religions from Pakistan and Afghanistan were granted Indian citizenship in thelast three years: Govt.AP Govt. to give free smartphones to all members of women’s self-help groups in the state to enablethem to avail various government services on digital platformTamil Nadu: Swami Vivekananda statue unveiled in Raj Bhavan in Chennai

Economy & CorporateDeal negotiated by NDA to procure 36 Rafale fighter jets was 2.86% cheaper than the UPA’s 2007offer: CAGDefence Ministry issues ‘expression of interest’ to shortlist companies for procurement of 111 NavalUtility Helicopters (NUH)Lok Sabha passes bill to ban unregulated deposit schemesRajya Sabha passes Interim Budget and Finance Bill 2019-20 on the last day of the Budget session ofParliament; was passed by Lok Sabha on Feb 11AYUSH Ministry launches e-AUSHADHI portal for online licensing of Ayurveda, Siddha, Unani andHomoeopathy drugsIndia’s first Centre for Textiles Trends Forecasting to be set up in the national capitalFormer Railway Board chairman Ashwani Lohani appointed chairman and managing director (CMD)of Air India

WorldGlobal unemployment rate at 5% in 2018, to decline to 4.9% in 2019: ILO’s ‘World Employment andSocial Outlook Trends 2019’World Radio Day observed on February 13 with theme as ‘Dialogue, Tolerance and Peace’

SportsMP Govt. to give reward of Rs 1 crore for sportsmen participating in international competitionsEngland (277, 361/5 declared) beat West Indies (154, 252) by 232 runs in 3rd and final Test in St Lucia,lose series 1-2

Current Affairs – News Headlines: February 15, 2019India

J&K: 43 CRPF personnel martyred in a suicide attack on a CRPF convoy on Srinagar-Jammuhighway in Pulwama districtCBDT Chairman Sushil Chandra appointed as the new Election CommissionerUnion Cabinet approves signing of 3 MoUs with Saudi Arabia for strengthening cooperation in thefields of tourism, customs and investment

sri vishnu charan | [email protected] |

https://t.me/PDF4Examshttps://t.me/IAS201819 https://t.me/PDF4Exams

https://t.me/TheHindu_Zone_official

Page 12: Current Affairs -February 1-15, 2019 · 15-02-2019  · 10, theme: ‘Sadak Suraksha-Jeevan Raksha’ National Testing Agency (NTA) launches mobile app through which students can

Current Affairs [PDF] -February 1-15, 2019

© 2019 GKToday | All Rights Reserved | https://www.gktoday.in 12

Goa: Former deputy CM and current BJP MLA Francis D’Souza dies at 64 in PanajiAP: TDP Lok Sabha member from Anakapalli, M Srinivasa Rao, quits the party and joins oppositionYSR CongressAP: UAE-based Tabreed to build India’s first district cooling system in state’s new capital Amaravati

Economy & CorporateWholesale Price Index (WPI)-based inflation declines to 10-month low of 2.76% in January 2019Centres increases Minimum Selling Price (MSP) of sugar by Rs 2 to Rs 31 per kgMSP is the rate below which the mills cannot sell sugar in the open market to wholesalersLarsen & Toubro lnfotech announces acquisition of Germany’s Nielsen+Partner for EUR 28 millionRajnath Singh flags off inaugural flight from Lucknow to Najaf in IraqHP India launches laptops HP Spectre Folio and HP Spectre x360 in IndiaGoAir CEO Cornelis Vrieswijk resigns

WorldNASA announces end of its Opportunity rover’s mission, 15 years after its arrival on Mars

SportsEva de Goede (Women’s) of Netherlands and Arthur Van Doren (Men’s) of Belgium namedInternational Hockey Federation’s (FIH) Players of the Year for 2018

February 1, 2019Lithium-ion cell technology Transfer by ISRO

ISRO has selected 10 companies for the Lithium-ion cell technology Transfer after the examination ofRequest for Qualification which contained a brief description of the qualification aspects, technologytransfer process, timelines and other relevant details.Firms Selected for Technology TransferThe firms selected by ISRO for the technology transfer are:

Amara Raja Batteries Limited, Chittoor.Bharat Electronics Limited, Pune.Carborundum Universal Limited, Kochi.Exicom Tele-Systems Limited, Gurgaon.GOCL Corporation Limited, Hyderabad.Jyoti CNC Automation Limited, Rajkot.National Aluminium Co Limited, Bhubaneswar.Sukhbir Agro Energy Limited, New Delhi.Tata Chemicals Limited, Mumbai.Thermax Limited, Pune.

Lithium-ion cell technologyThe Lithium-ion cell technology has been developed by Vikram Sarabhai Space Centre (VSSC). After thesuccessful deployment of indigenous lithium-ion batteries in various missions of ISRO, the VSSC will nowtransfer the technology to the industries to establish production facilities for producing lithium-ion cellsto cover the entire spectrum of power storage requirements of the country.The progress in Li-ion battery technology research has made it the favourite power source for electric andhybrid electric vehicles owing to its high voltage, high energy density, long life cycle and high storagecharacteristics. Li-ion batteries find wide application in electronic gadgets, telecommunication andindustrial applications as well as in aerospace.

Human Space Flight Centre InauguratedThe Human Space Flight Centre (HSFC) which would be the core of ISRO’s future manned missions wasinaugurated at the ISRO headquarters in Bengaluru.The Human Space Flight Centre will implement the first development flight of Gaganyaan with thesupport of existing ISRO Centres. The Human Space Flight Centre would be responsible for end-to-endmission planning, development of engineering systems for crew survival in space, crew selection andtraining and also pursue activities for sustained human space flight missions.

sri vishnu charan | [email protected] |

https://t.me/PDF4Examshttps://t.me/IAS201819 https://t.me/PDF4Exams

https://t.me/TheHindu_Zone_official

Page 13: Current Affairs -February 1-15, 2019 · 15-02-2019  · 10, theme: ‘Sadak Suraksha-Jeevan Raksha’ National Testing Agency (NTA) launches mobile app through which students can

Current Affairs [PDF] -February 1-15, 2019

© 2019 GKToday | All Rights Reserved | https://www.gktoday.in 13

GaganyaanGaganyaan which is India’s first manned space mission has been accorded highest priority by ISRO andthe plan is to have the first unmanned mission in December 2020 and second for July 2021. The mannedmission will happen in December 2021.The Gaganyaan project is headed by R. Hutton, who was the Polar Satellite Launch Vehicle (PSLV)Director. The Gaganyaan project takes the Indian astronauts into space to a height of 350-400 km abovethe Earth and orbits around the planet for at least a week. The Indian astronauts would be conductingexperiments in the space

North East Circuit under Swadesh Darshan InauguratedUnion Minister for Tourism, Shri. K.J. Alphons inaugurated a project under North East India Circuit of Swadesh Darshan Scheme of Ministry of Tourism.Infrastructure development activities like Tourist Information Centre, Meditation Centre, Organic EcoTourism centre, Log Huts, Zip Line, Flower Exhibition Centre, Walkways, Souvenir Shops, Cafeteria, RainShelters, Wayside Amenity, Last Mile Connectivity, Parking, Public Toilets etc, have been developed underthis project.The circuit is the first ever project in Sikkim under the scheme and encompasses Rangpo– Rorathang-Aritar- Phadamchen- Nathang-Sherathang- Tsongmo- Gangtok-Phodong- Mangan- Lachung-Yumthang-Lachen- Thangu-Gurudongmer- Mangan- Gangtok-Tuminlingee- Singtam.Swadesh Darshan SchemeSwadesh Darshan scheme is one of the flagship scheme of the Ministry of tourism for the development ofthematic circuits in the country in a planned and prioritised manner. The project is aimed at thedevelopment of quality infrastructure in the country with the objective of providing better experience andfacilities to the visitors on one hand and on other hand fostering the economic growth.The 13 thematic circuits under the Swadesh Darshan scheme are North East India Circuit, BuddhistCircuit, Himalayan Circuit, Coastal Circuit, Krishna Circuit, Desert Circuit, Tribal Circuit, Eco Circuit,Rural Circuit, Spiritual Circuit, Ramayana Circuit, Heritage Circuit and Wildlife Circuit.Swadesh Darshan Scheme is a 100 per cent centrally funded scheme and also provides for leveragingvoluntary funding under Corporate Social Responsibility (CSR) initiatives of Central Public SectorUndertakings and corporate sector.

Interim Budget 2019: HighlightsThe highlights of the Interim Budget 2019 presented by the Finance Minister are listed below:

6,000 rupees yearly direct support to farmers through PM-KISAN Programme.Pension of 3,000 rupees for unorganised sector workers earning up to 15,000 rupees through a megaPension Scheme – Pradhan Mantri Shram- Yogi Maandhan.Present Income Tax rates to continue and full tax rebate up to an income of 5 lakh rupees forindividual taxpayers. Standard deduction raised to 50,000, a hike of 10,000 for salaried class.Simplification of Direct tax system and returns to be processed in 24 hours with immediate refunds.90 per cent GST payers can file quarterly returns.Two per cent interest rebate for Small and Medium Enterprises to get an incremental loan of onecrore rupees.Constituting group of Ministers to examine GST burden on home buyers.Digitalization of Customs for export and import transactions.Allocations towards Health care, MGNREGA, SC/ST welfare programmes, Pradhan Mantri GramSadak Yojana, Development of infrastructure in North-East has been increased substantially.Rashtriya Kamdhenu Aayog for genetic upgradation of the cow.Separate Department of Fisheries.Electricity connection to all willing families by next month.Allocations to Defence budget crosses three lakh crore rupees for the first time.More than three lakh 38 thousand shell companies deregistered after demonetization.10 point vision for 2030 to realize India’s social economic potential.India to be a 10 trillion dollars economy in 13 years.

sri vishnu charan | [email protected] |

https://t.me/PDF4Examshttps://t.me/IAS201819 https://t.me/PDF4Exams

https://t.me/TheHindu_Zone_official

Page 14: Current Affairs -February 1-15, 2019 · 15-02-2019  · 10, theme: ‘Sadak Suraksha-Jeevan Raksha’ National Testing Agency (NTA) launches mobile app through which students can

Current Affairs [PDF] -February 1-15, 2019

© 2019 GKToday | All Rights Reserved | https://www.gktoday.in 14

Government e-Marketplace – GeM to be extended to all Central Public Sector Enterprises.Container cargo movement to the North-East through Brahmaputra river.Allocation for infrastructure development in the North East increased by 21 per cent.AIIMS to be established in Haryana.Digital connectivity for more than one lakh more villages.National Centre on Artificial Intelligence Centre to be established.Tightening of Cinematograph Act to check piracy.Single window clearance for shooting films for Indian filmmakers.Indigenous development of semi-high speed Vande Bharat Express trainThe fiscal deficit pegged at 3.4 per cent of GDP.

The Budget was keenly watched as the country is heading for a general election in 2019.International conference on Guru Padmasambhava in New Delhi

An International Conference on 8th century Himalayan sage Guru Padmasambhava was held in New Delhi.The conference was organised as part of events to commemorate 50-years of formalization of diplomaticties between India and Bhutan.Guru PadmasambhavaGuru Padmasambhava was born in India and travelled all across Himalayan region in the 8th century tospread Buddhism and Buddhist teachings.Guru Padmasambhava is highly revered in Bhutan. There is an image or painting of the GuruPadmasambhava in every Bhutanese home or temple.Guru Padmasambhava was also known as Guru Rinpoche is widely venerated as a “second Buddha” byadherents of Tibetan Buddhism in Tibet, Nepal, Bhutan, the Himalayan states of India, and elsewhere.Guru Padmasambhava is also considered to be the founder of Nyingma tradition, oldest of the four majorschools of Tibetan Buddhism.India-Bhutan Diplomatic RelationsThe diplomatic relations between India and Bhutan were established in 1968. India established a specialoffice of India in Thimphu in 1968. Prior to this, Indian diplomatic relations with Bhutan were looked afterby Indian Political Officer in Sikkim.The India-Bhutan bilateral relations are governed by the Treaty of Friendship and Cooperation signed in1949 between the two countries. The treaty was revised in February 2007. The India-Bhutan FriendshipTreaty together with reflecting the contemporary nature of our relationship lays the foundation for theirfuture development in the 21st century. India and Bhutan are celebrating the Golden Jubilee of theestablishment of formal diplomatic relations between the two countries.

Interim Budget 2019: Changes in Income Tax RegimeThe Union Minister of Finance Piyush Goyal presented the interim budget. The changes proposed relatedto the income tax regime are listed below:

The Finance Minister has increased the tax slab for zero tax to Five Lakh rupees and the standarddeduction introduced in the 2018 budget has been increased to Fifty thousand rupees from currentforty thousand.

Standard DeductionStandard Deduction refers to deduction allowed as per the Income Tax irrespective of the expenses met or theinvestment made by the individual. An individual is not required to disclose any investment proofs or expense bills forthis purpose, the Standard Deduction is allowed at a standard rate.

Additionally, those individuals with gross income up to 6.5 lakh rupees would be exempted frompayment of tax if they make investments in provident funds and prescribed equities.The TDS threshold for home rent has been increased to 2.4 lakh from Rs 1.8 lakh.Interest accruals up to Rs 40,000 in post offices and banks have been made tax free.The budget also increased the Income-tax relief on notional rent from unsold houses extended to 2years from 1 year.The exemptions under the capital gain tax have been increased to two crores. Capital gains

sri vishnu charan | [email protected] |

https://t.me/PDF4Examshttps://t.me/IAS201819 https://t.me/PDF4Exams

https://t.me/TheHindu_Zone_official

Page 15: Current Affairs -February 1-15, 2019 · 15-02-2019  · 10, theme: ‘Sadak Suraksha-Jeevan Raksha’ National Testing Agency (NTA) launches mobile app through which students can

Current Affairs [PDF] -February 1-15, 2019

© 2019 GKToday | All Rights Reserved | https://www.gktoday.in 15

exemption to be available on 2 house propertiesIT returns would be processed within 24 hours and returns will be paid immediately.

The Minister also announced that within a span of two years all assessment and verification of IT returnswill be done electronically by an anonymised tax system without any intervention by officials.

Interim Budget 2019: What’s there for AgricultureThe Union Minister of Finance Piyush Goyal presented the interim budget. The various provisions in thebudget related to the Agricultural sector are listed below:

The budget announced a farm support scheme for farmers owning up to 2 hectares of lands.These farmers will get Rs 6,000 per year in three equal instalments effective from December 1, 2018.The Finance Minister has set aside Rs 75,000 crore for the scheme.The farm support scheme is expected to benefit 12 crore small and marginal farmers.The allocation for the Rashtriya Gokul Mission has been increased to Rs 750 crore for increasing theproduction and productivity of cows.2% interest subvention for farmers pursuing animal husbandry and fisheries through Kisaan creditcards.A separate Department for fisheries would be created.Farmers adversely affected by natural calamities would get 2% interest subvention and additional3% interest subvention upon timely repayment.Budget announced setting up of Rashtriya Kamdhenu Aayog to upscale sustainable geneticupgradation of cow resources and to enhance production and productivity of cows.The Rashtriya Kamdhenu Aayog would also be responsible for effective implementation of laws andwelfare schemes for cows.

The budget has made some big announcements but the critics argue that the long-term plans are mereslogans with no clear road map.

Revision of GDP EstimatesThe Ministry of Statistics and Programme Implementation has released the First Revised Estimates ofNational Income, Consumption Expenditure, Saving and Capital Formation, 2017-18. The highlights of therevised estimates are:

The forecast for GDP growth for 2017-18 was increased to 7.2% from the earlier estimate of 6.7%.The actual growth rate in 2016-17 has been increased to 8.2% from the 7.1% estimated earlier.

The Ministry of Statistics and Programme Implementation clarifies that the revision of the estimates for2015-16 and 2016-17 are on the account of the availability of the latest data on agricultural production,industrial production, government expenditure (replacing the Revised Estimates with Actual for 2016-17)and also more comprehensive data available from various source agencies like the MCA and the NABARDand State/Union Territory Directorates of Economics and Statistics.Criticisms against the RevisionThe Economists criticise the revision because of the following reasons:

The numbers do not match up to the ground realities.The data corresponding to the demonetisation year of 2016-17 shows strong growth in sectors thatwere widely agreed to have been badly hit by the exercise.The main factor for the revision was the increase in private final consumption expenditure whichwas increased by 1 percentage point. This was inconsistent with the idea of people having less cashto make purchases.The other main driver of the upward revision was the construction sector which was revisedupwards by 4.7 percentage points. The earlier analyses had indicated that construction which has alarge informal sector component was adversely affected similar to other informal sectors due todemonetisation.Other drivers for the upward revision in 2017-18, mining and quarrying sector and the publicadministration sector had data that is compiled by the government itself. Hence they should nothave undergone such a vast revision. .

sri vishnu charan | [email protected] |

https://t.me/PDF4Examshttps://t.me/IAS201819 https://t.me/PDF4Exams

https://t.me/TheHindu_Zone_official

Page 16: Current Affairs -February 1-15, 2019 · 15-02-2019  · 10, theme: ‘Sadak Suraksha-Jeevan Raksha’ National Testing Agency (NTA) launches mobile app through which students can

Current Affairs [PDF] -February 1-15, 2019

© 2019 GKToday | All Rights Reserved | https://www.gktoday.in 16

With 2019 being a election year, critics also cry foul play on part of the government to project a happypicture before the general elections.

Prompt Corrective Action framework restrictions lifted for Three BanksThe Reserve Bank of India (RBI) has lifted the Prompt Corrective Action (PCA) framework operationalcurbs on Bank of India (BoI), Bank of Maharashtra (BoM) and Oriental Bank of Commerce (OBC).These public sector banks are out of the prompt corrective action (PCA) framework. This will aid inmaking marked improvements in the capital positions and asset quality.The PCA restrictions were lifted after these banks provided a written commitment that they wouldcomply with the norms of minimum regulatory capital, net NPAs (Non-performing Assets) and leverageratio on an ongoing basis. These Banks have also apprised RBI of the structural and systemicimprovements they have put in place.Prompt Corrective Action (PCA) FrameworkPrompt Corrective Action (PCA) framework has been issued by the RBI to maintain the sound financialhealth of banks. The RBI will initiate certain structured and discretionary actions for the bank under thePCA when the Banks breach any of the three key regulatory trigger points:

Capital to risk-weighted assets ratioNet non-performing assetsReturn on assets.

The PCA framework is aimed at nudging the banks to take corrective measures in a timely manner, inorder to restore their financial health.

Best Practices in Budget FormulationThe Transparency International conducted a survey to analyse the best practices followed by the states inthe Budget Formulation.About the SurveyThe survey was based on 4 parameters which include public disclosure, budgetary process, post-budgetfiscal management and efforts to make the budget more transparent and citizen-friendly. The findings ofthe survey are:

The top slot in the survey was occupied by Assam.Assam was followed by Andhra Pradesh and Odisha in the ranking of best practices followed bystates in the Budget formulation.Meghalaya, Manipur and Punjab figured in the bottom list.The report notes that Assam is the only state out of the 29 states and 2 UTs, which has published aCitizens Budget in the public domain.Also, the government of Assam is the only government that has conducted budget awarenesscampaigns across 17 districts.

Transparency InternationalTransparency International is an international non-governmental organization based in Berlin, Germany.Founded in 1993 by a group of individuals who decided to take up a stance against corruption, today fromvillages in rural India to the corridors of power in Brussels, Transparency International gives voice to thevictims and witnesses of corruption. The Corruption Perception Index of Transparency International isthe most widely used indicator of corruption worldwide.

Defence Ministry approves acquisition of Milan-2T Anti-Tank MissilesThe Defence Acquisition Council headed by Defence Minister Nirmala Sitharaman has approved theacquisition of the Rs 1,200 crore Milan-2T anti-tank missiles. While the requirement was of 70,000different types of anti-tank guided missiles (ATGM) and 850 different launchers, the DAC has approvedthe purchase of 5,000 Milan 2T anti-tank guided missiles from France.Defence Acquisition CouncilThe Defence Acquisition Council (DAC) headed by Union Minister of Defence was constituted to ensure afair defence procurement planning process. The Council aims to ensure expeditious procurement of therequirements of the Armed Forces in terms of capabilities sought and time frame prescribed by optimallyutilizing the allocated budgetary resources.

sri vishnu charan | [email protected] |

https://t.me/PDF4Examshttps://t.me/IAS201819 https://t.me/PDF4Exams

https://t.me/TheHindu_Zone_official

Page 17: Current Affairs -February 1-15, 2019 · 15-02-2019  · 10, theme: ‘Sadak Suraksha-Jeevan Raksha’ National Testing Agency (NTA) launches mobile app through which students can

Current Affairs [PDF] -February 1-15, 2019

© 2019 GKToday | All Rights Reserved | https://www.gktoday.in 17

The functions of the DAC include:Principle approval authority for 15 Years Long Terms Integrated Perspective Plan for DefenceForces.Categorization of the acquisition proposals relating to ‘Buy’, ‘Buy & Make’ and ‘Make’.Addressing issues relating to Single vendor clearance.Decision regarding ‘offset’ provisions in respect of acquisition proposals above Rs. 300 crores.Decisions regarding Transfer of Technology under ‘Buy & Make’ category of acquisition proposals.Field Trial evaluation.

Milan-2T Anti-Tank MissileAn anti-tank missile is aimed at destroying the vehicles that are heavily armoured. The features of theMilan-2T Anti-Tank Missile are:

Milan is a portable medium-range, anti-tank missile produced by Euromissile, based in Fontenay-aux-Roses in France.The Milan munitions consist of the missile in a waterproof launch tube.While MILAN 2 has a single shaped charge warhead for use against very thick and compositearmour, MILAN 2T is armed with a tandem charge for use against reactive armour.

The DAC has given approval for the procurement of Milan-2T Anti-Tank Missile to meet the urgentrequirements since the indigenous anti-tank missile Nag being developed by Defence Research andDevelopment Organisation (DRDO) is still under development.

Startup India-WhatsApp Grand ChallengeThe Facebook-owned messaging app WhatsApp has announced Startup India-WhatsApp Grand Challengeto encourage entrepreneurs and small businesses in India.About the challengeThe challenge is being hosted in association with Invest India. The top 5 winners would receive a cashprize worth $250,000 (roughly Rs. 1.8 crores).The challenge is aimed at encouraging the entrepreneurs who have innovative ideas, business models thatsolve for a local India problem, making a large scale socio-economic impact.The themes for this Start-up challenge are healthcare, rural economy, financial and digital inclusion,education and citizen safety.The applications will be evaluated by an independent evaluation committee and out of which the best 30ideas will be shortlisted for the subsequent round and further 10 in the following round.The selected 10 ideas would participate at the live pitch day during the grand finale and the top 5 would beawarded grants worth $250,000 (roughly Rs. 1.8 crores).Invest IndiaInvest India is a not for profit entity established under the Department for Promotion of Industry andInternal Trade, Ministry of Commerce and Industry in 2009.It is a national investment promotion and facilitation agency focussing on sector-specific investortargeting and development of new partnerships to enable sustainable investments in India.

Aero India 2019The 12th edition of Aero India, Aero India 2019, will be held at Air Force Station at Yelahanka in Bengalurufrom February 20 to 24. Aero India 2019 aims to provide a significant platform in bolstering businessopportunities in the International aviation sector.

Aero IndiaAero India Exhibition/Show is organised every two years. Aero India has carved a niche for itself globally as apremier aerospace exhibition, with eleven successful editions organised since 1996. The show aims at giving a fillipto the rapidly growing economy, defence production and participation of private players.

Aero India 2019The Aero India 2019 has a tagline “The Runway to a Billion Opportunities” and has a logo inspired by theTejas Light Combat Aircraft (LCA). The Ministry of Defence has planned following events during the AeroIndia 2019:

sri vishnu charan | [email protected] |

https://t.me/PDF4Examshttps://t.me/IAS201819 https://t.me/PDF4Exams

https://t.me/TheHindu_Zone_official

Page 18: Current Affairs -February 1-15, 2019 · 15-02-2019  · 10, theme: ‘Sadak Suraksha-Jeevan Raksha’ National Testing Agency (NTA) launches mobile app through which students can

Current Affairs [PDF] -February 1-15, 2019

© 2019 GKToday | All Rights Reserved | https://www.gktoday.in 18

Start-up DayThe Start-up day would witness a start-up showcase event on February 21 to tap into the highly motivatedentrepreneurial talent pool of the country. The showcase event is aimed at providing the platform forinteractions between the domestic and international start-up community, key policymakers in the Indiandefence sector, and CEOs of leading Indian and global start-ups.

Womens’ DayWomen’s Day would be organised On February 23 to showcase achievements made by women in theaerospace sector on February 23. The Women’s Day would witness felicitation of women achievers,unveiling of the brochure highlighting achievements of women, experience sharing by leading women inAero-Space Sector and flying display by women pilots, paratroopers etc.

Drone OlympicsDrone Olympics provides an opportunity for all the UAV manufacturing enthusiasts to showcase thecapabilities of the Drones they are manufacturing.

Technology Day/Students DayTechnology Day would be observed on February 22 to provide an opportunity for the students involved inthe aerospace sector, both civil and defence to showcase their projects.

Photography contestA Photography contest would be held on the theme ‘Flying Objects’.

The Future of Rail Report: Key FactsThe Future of the Rail Report was prepared by the International Energy Agency (IEA) in association withthe International Union of Railways (UIC).Findings of the ReportThe report provides a specific focus on India, The important findings of the Future of Rail Report are:

Indian railways will account for nearly 40% of total global rail activity by 2050.The Investments in Indian urban rail infrastructure is estimated to reach nearly $190bn by 2050.The report notes that India can save up to $64bn on fuel expenses by that time.The report predicts bright prospects for Indian Railways.

The report notes that the rail sector carries 8% of passengers across the world and 7% of global freightmovement but it utilises 2% of the total transport energy demand in the world, signifying its efficiency.International Energy Agency (IEA)The International Energy Agency (IEA) is an intergovernmental organization established by theOrganisation for Economic Co-operation and Development (OECD) in 1974. IEA even works non-membercountries, especially China, India, and Russia.Initially, IEA was focused on responding to physical disruptions in the supply of oil, as well as serving asan information source on statistics about the international oil market and other energy sectors. Itsmandate has been widened to focus on the “3Es” of effectual energy policy: energy security, economicdevelopment, and environmental protection.International Union of Railways (UIC)The International Union of Railways (UIC) is an international body of the rail transport industry. The UICwas created on 17 October 1922 with the aim of standardising industry practices. At present, UIC had 194members across 5 continents.

February 2, 2019Comparing PM-KISAN with Rythu Bandhu scheme of Telangana

The union government announced the PM-KISAN farm support scheme to address the farm sector incrisis. Similar initiatives have already been announced by states. Here’s a comparison of the centre’s farmsupport scheme PM-KISAN with similar schemes from various states.PM-KISAN and Rythu Bandhu of TelanganaThe Centres Pradhan Mantri Kisan Samman Nidhi (PM-KISAN) would provide an annual grant of Rs 6,000to farmers. Below is the comparison with Rythu Bandhu of Telangana:

PM-KISAN has a cap of two hectares to make a farmer eligible for the scheme. But the Rythu Bandhuof Telangana posses no restrictions on the extent of land one should own. However, there is a

sri vishnu charan | [email protected] |

https://t.me/PDF4Examshttps://t.me/IAS201819 https://t.me/PDF4Exams

https://t.me/TheHindu_Zone_official

Page 19: Current Affairs -February 1-15, 2019 · 15-02-2019  · 10, theme: ‘Sadak Suraksha-Jeevan Raksha’ National Testing Agency (NTA) launches mobile app through which students can

Current Affairs [PDF] -February 1-15, 2019

© 2019 GKToday | All Rights Reserved | https://www.gktoday.in 19

restriction on the acreage to 50 acres in the Rabi season for Rythu Bandhu payouts.The capping of acreage at five acres under PM-KISAN would be disadvantageous to farmers in therain-fed areas, where the farmers, even if they have more land, would still require financialassistance.While the centre’s farm support is ₹6,000 in three equal instalments, farmers get Rs 8,000 in twoinstalments ahead of the Kharif and Rabi in Telangana and the government has already promised toincrease it to Rs 10000 in the current year.Since the intricacies of the PM-KISAN are yet to be declared, it is not clear whether the PM-KISANwould include tenant farmers in the scheme. Rythu Bandhu incorporates even the tenant farmersunder the scheme.

KALIA and Krishak BandhuThe KALIA scheme of Odisha offers annual assistance of Rs 12,500 each to each to farmers and KrishakBandhu scheme of West Bengal offers an aid of Rs 5000 to farmers.Of all, the KALIA scheme of Odisha looks more comprehensive since it covers the landless agriculturallabourers too and also seeks to provide help to vulnerable agricultural households, besides promisinginterest-free crop loan.The schemes of Telangana, Odisha and West Bengal have also incorporated the component of insurance.The payouts in Odisha and West Bengal are yet to happen.

Enhancing Agribusiness of Cooperatives in MaharashtraThe Cooperatives Department of Maharashtra has decided to appoint managers at six major cities acrossthe country to enhance agribusiness between cooperatives in Maharashtra and other States.Improving Agri-BusinessMaharashtra has a surplus of products such as onion, tomato, grapes, pomegranate, mangoes and raisins.These are traded in other states and in the process the traders accumulate the huge profits while thefarmers are paid low due to the surplus production in the state.To bridge the mismatch and ensure a better price for farmers and reduced burden on consumers, theMaharashtra government would appoint Managers at the cities Guwahati, Kolkata, Chennai, New Delhi,Chandigarh and Jaipur to look for business opportunities in other States.Strengthening the CooperativesThe Maharashtra government would strengthen the farmers cooperatives in the state to revive andenhance the collection and trading capabilities of cooperative institutions. The strengthening ofcooperatives would be through two step prolonged strategy:

Old cooperatives, which have been in the State since the 1950s, are being revived with the help of thenew strategy and funding from institutions such as the World Bank.Provide an air freight subsidy on a pilot basis to carry high-value agricultural produce to the NorthEast, Sikkim and Jammu & Kashmir, which will build up the appetite and demand from localmarkets.

The cooperatives will sell supply fruits and vegetables directly to similar cooperatives institutions in otherStates. The profits would be retained by the cooperatives owned by the farmers.

Sleep Deprivation and Alzheimer’s RiskThe study published by the Researchers of the Washington University of the US published in the journalScience has found that sleep deprivation may increase the risk of developing Alzheimer’s disease.

Alzheimer’s DiseaseAlzheimer’s disease is a chronic neurodegenerative disease that usually starts slowly and worsens over time and it isthe cause of 60–70% of cases of dementia. As the disease advances, symptoms can include problems withlanguage, disorientation (including easily getting lost), mood swings, loss of motivation, not managing self care,and behavioural issues and gradually bodily functions are lost, ultimately leading to death.

Findings of the StudyThe important findings of the study include:

Sleeplessness accelerates the spread of toxic clumps of tau through the brain. Tau is a harbinger ofbrain damage and a decisive step along the path to dementia.

sri vishnu charan | [email protected] |

https://t.me/PDF4Examshttps://t.me/IAS201819 https://t.me/PDF4Exams

https://t.me/TheHindu_Zone_official

Page 20: Current Affairs -February 1-15, 2019 · 15-02-2019  · 10, theme: ‘Sadak Suraksha-Jeevan Raksha’ National Testing Agency (NTA) launches mobile app through which students can

Current Affairs [PDF] -February 1-15, 2019

© 2019 GKToday | All Rights Reserved | https://www.gktoday.in 20

Lack of sleep alone helps drive the disease and good sleep habits may help preserve brain health.Tau is found in the brain even among the healthy people, but under certain conditions, it will clumptogether into tangles that injure nearby tissue and may lead to cognitive decline.A sleepless night can result in increasing of the tau levels to rise by about 50 per cent.

TauTau is a protein routinely released during waking hours by the normal business of thinking and doing, andthen the release is decreased during sleep allowing tau to be cleared away. Sleep deprivation interruptsthe normal cycle As a result, tau protein will build up and it is more likely that the Tau protein will startaccumulating into harmful tangles.

INSTEX : Iran Trade Promotion PlanINSTEX acronym of Instrument in Support of Trade Exchanges is a project of the three governments ofUK, France and Germany to bypass US sanctions on Iran.INSTEXINSTEX is a special payment system which will help to save the Iran nuclear deal by allowing Tehran tokeep trading with EU companies despite Washington re-imposing sanctions.INSTEX is registered at Paris with an initial 3,000 Euros in the capital and a supervisory board withmembers from France and Germany and chaired by the UK. INSTEX will support legitimate Europeantrade with Iran, focusing initially on the sectors most essential to the Iranian population such aspharmaceutical, medical devices and agri-food goods. INSTEX is primarily aimed at small and medium-sized companies.In future, INSTEX will also be opened to third countries wanting to trade with Iran. INSTEX to beoperational requires Iran to set up a parallel structure of its own.Protecting against US SanctionsINSTEX would receive payments from companies that want to trade with Iran, either by receiving waiversfor oil imports or permissible trade in goods like food and medicine. As a result, there would be no directtransfer of funds between Iran and European companies. This would theoretically insulate firms from U.S.penalties.

Classification of Denotified, Nomadic and Semi-Nomadic CommunitiesThe interim Budget 2019 for the second time has set the ball rolling for the Classification of Denotified,Nomadic and Semi-Nomadic Communities.Provisions in the BudgetThe Budget has made following announcements related to the denotified, Nomadic and Semi-NomadicCommunities:

Formation of a committee under the Niti Aayog, which would identify and classify these tribes.Constitution of a Welfare Board under the Ministry of Social Justice and Empowerment for thepurpose of implementing welfare and development programmes for denotified, nomadic and semi-nomadic communities.

Earlier attempt for classificationNational Commission for Denotified, Nomadic And Semi-Nomadic Tribes headed by Dada Idate hadprepared state-wise lists. But this list drew a lot of criticisms including from the researchers who wereworking with the commission. It was criticised that the Idate commission lacked the scientific datanecessary to bring out a comprehensive classification of denotified, Nomadic and Semi-NomadicCommunities.Vulnerability of these CommunitiesDenotified, Nomadic and Semi-Nomadic Communities are considered to be the most deprived sections ofIndian society. These communities are hard to reach and less visible. The nomadic and semi-nomadiccommunities move from place to place in search of livelihood. As a result, they are frequently left out.Hence to address the issues related to these communities there is a need for special attention. The RenkeCommission and the Idate Commission have done commendable work in studying the various problemsfaced by these communities and recommending measures to address the issues more comprehensively.

Budgetary allocations to the Ministry of Home AffairsThe enhanced allocation for the Ministry of Home Affairs in the interim Budget 2019 is a reflection of the

sri vishnu charan | [email protected] |

https://t.me/PDF4Examshttps://t.me/IAS201819 https://t.me/PDF4Exams

https://t.me/TheHindu_Zone_official

Page 21: Current Affairs -February 1-15, 2019 · 15-02-2019  · 10, theme: ‘Sadak Suraksha-Jeevan Raksha’ National Testing Agency (NTA) launches mobile app through which students can

Current Affairs [PDF] -February 1-15, 2019

© 2019 GKToday | All Rights Reserved | https://www.gktoday.in 21

government’s focus on internal security. The Budgetary allocation for the Ministry of Home Affairs stoodat Rs 1,03,927 crore.Enhanced allocations for the Ministry of Home AffairsThe allocation of Rs 1,03,927 crore is Rs 10,477 crore more than that for the previous year and the revisedestimates for the Ministry of Home Affairs was pegged at Rs 99,034 crore. The enhanced allocation istowards following areas:

The allocation for improving police infrastructure went up to Rs 5,117 crore from Rs 4,750crore continuing the emphasis on strengthening the police infrastructure to effectively address thediverse internal security challenges haunting India.The budgetary allocation under the sub-head of ‘Mission for Protection and Empowerment forWomen’ increased to Rs 1,330 crore from previous year’s revised budgetary estimate of Rs 1,156crore in recognition of the need to effectively address the crimes against women and to build acredible deterrence by strengthening the proscriptive mechanisms.The allocation for border infrastructure and management was a shade lower, at Rs 2,000 crore, fromprevious year’s revised estimate of Rs 2,001 crore.

Enhanced allocations towards the Ministry of Home Affairs are a timely intervention especially inimproving police infrastructure and protection of women.

PM Kisan Samman NidhiThe interim budget 2019 has announced a farm income support scheme PM Kisan Samman Nidhi (PM-KISAN). The features of the PM Kisan Samman Nidhi scheme are:

The Centre will transfer Rs 6,000 every year to small and marginal farmers.The scheme will cost Rs 75,000-crore to the exchequer.The scheme aims to address unrest among cultivators who have suffered due to adverse weatherand weak prices.Rs 6,000 will be transferred directly into the bank accounts of the beneficiary farmers in threeinstalments of Rs 2,000 each.The scheme will benefit 120 million farmers with cultivable land of up to 2 hectares.

Analysis of the SchemeThe promised transfer of Rs 6,000 every year amounts to an average of Rs 500 per month. The NationalBank for Agriculture and Rural Development (Nabard) in its rural financial inclusion survey 2015-16 hadestimated farmer’s average monthly income from cultivation at Rs 3,140 in its rural financial inclusionsurvey 2015-16. Therefore income support of an average of Rs 500 per month comes out to be 16% of afarmer’s average monthly income from cultivation.

Estimated farmer’s average monthly incomeFor the most marginal farmer with 0.01 hectare, the cash transfer is comparable with the estimatedmonthly income of Rs 566 from cultivation, but it is relatively less attractive for those with more land.

sri vishnu charan | [email protected] |

https://t.me/PDF4Examshttps://t.me/IAS201819 https://t.me/PDF4Exams

https://t.me/TheHindu_Zone_official

Page 22: Current Affairs -February 1-15, 2019 · 15-02-2019  · 10, theme: ‘Sadak Suraksha-Jeevan Raksha’ National Testing Agency (NTA) launches mobile app through which students can

Current Affairs [PDF] -February 1-15, 2019

© 2019 GKToday | All Rights Reserved | https://www.gktoday.in 22

Revenue Mobilisation: Interim Budget 2019The interim budget has announced a slew of welfare measures for farm and rural economy, middle class,realty and housing and the unorganised sector. The burden on the exchequer due to the farm incomesupport scheme PM-KISAN itself comes around Rs 75000 crores. This necessitates the government toexplore the avenues to find the corpus to pay for these schemes.Revenue MobilisationThe government is exploring the following avenues to fund welfare schemes:

Large dividend transfers by the Reserve Bank of India and PSUs to balance the Budget deficit afterfunding the welfare schemes.The government is expecting Rs 82,911 crore through dividend from banks, financial institutions andthe RBI in 2019-20.In 2018-19 the government estimates receipts of Rs 74,140 crore from banks, financial institutionsand the RBI, much higher than the budget estimate of Rs 54,817 crore.The government is estimating Rs 53,200 crore as PSU dividend in 2019-20.The government is expecting to raise anywhere between Rs 12,000 crore and Rs 20,000 crorethrough the CPSE buyback route.

Challenges in Revenue MobilisationThe revenue from the Goods and Service Tax (GST) for the most part of the year has lagged behind the Rs 1lakh crore monthly target. The government’s efforts at disinvestments are also not yielding desiredresults. The government’s planned stake sales in state-run firms are still short of the target.As a result, the government is heavily dependent on the dividends from state-run firms, financialinstitutions and the RBI to fund its additional expenditure. The government has pegged dividends at Rs1.36 lakh crore in 2019-20 which is a 14 per cent rise from an already elevated dividend collection of Rs 1.19lakh crore in 2018-19.

10 dimensions of Vision 2030The Union Government in the interim budget 2019 unveiled its vision for 2030, listing 10 dimensionsto create an India where poverty, malnutrition, littering and illiteracy would be a matter of the past.Vision 2030The 10 dimensions of Vision 2030 listed in the interim budget 2019 are:Physical & Social InfrastructureBuilding next-generation infrastructure in all sectors comprising roads, railways, seaports, airports andinland waterways together with social infrastructure is the first dimension to provide ease of living.Digital IndiaBuilding a “Digital India” that reaches every corner of the economy and every citizen.Clean & Green IndiaAn India that drives electric vehicles, with renewables becoming a major source of energy, bringing downimport dependence and increasing energy security for our people is the vision of Clean & Green India.Rural IndustrializationExpanding of rural industrialization using modern industrial technologies, based on the ‘Make in India’approach, using grassroot MSMEs and startups across the country.Clean RiversClean Rivers with safe drinking water to all Indians using micro-irrigation techniques is the fifthdimension.Oceans & CoastlineExploitation of the Blue Economy, to ensure better standards and quality of life for a large number ofpeople living in the coastal areas. Exploiting the potential of India’s long coastline for becoming thestrength of the economy.SpaceIndia becoming the launch-pad of satellites for the World and placing an Indian astronaut into space by2022 (Project Gaganyaan) is the seventh dimension.

sri vishnu charan | [email protected] |

https://t.me/PDF4Examshttps://t.me/IAS201819 https://t.me/PDF4Exams

https://t.me/TheHindu_Zone_official

Page 23: Current Affairs -February 1-15, 2019 · 15-02-2019  · 10, theme: ‘Sadak Suraksha-Jeevan Raksha’ National Testing Agency (NTA) launches mobile app through which students can

Current Affairs [PDF] -February 1-15, 2019

© 2019 GKToday | All Rights Reserved | https://www.gktoday.in 23

Self-sufficiency in Food ProductionAttaining self-sufficiency in food and improving agricultural productivity with an emphasis on organicfood.HealthA healthy India, with a distress-free and comprehensive wellness system for all, is the ninth dimension.Minimum Government, Maximum GovernanceMinimum Government, Maximum Governance with proactive, responsible, friendly bureaucracy andelectronic governance is the tenth dimension.

FDI to India since 2014The Union Finance Minister Piyush Goyal in the interim budget 2019 stated that India has receivedmassive foreign direct investment worth USD 239 billion in the last five years.FDI: From Where to Where?The Union Finance Minister attributed this increased FDI inflow to a stable and predictable regulatoryregime, growing economy, strong fundamentals and liberalisation of the FDI policy allowing most FDI tocome through the automatic route. The important aspects of these FDI inflows are:

The sectors which received maximum foreign inflows include services, computer software andhardware, telecommunications, trading, construction, automobile, and power.Mauritius, Singapore, Netherlands, the US and Japan were the top source of FDI’s for India.

Automatic Route and Approval Route of FDIThe automatic and approval routes are designed with an aim to have a monitoring over the investment activities andto ensure the robustness of the economy is not compromised due to outside capital.Under the Automatic Route, neither the investor nor the Indian company requires any approval from the ReserveBank or Government of India for the investment. The sectors for the Automatic Route are specified under the FDIpolicy updated from time to time.Under the Approval Route, the foreign investor or the Indian company is required to obtain prior approval from theGovernment of India agencies or bodies specified.

The relaxation of the foreign investments norms in several sectors, including single-brand retail, defence,airlines and food processing had a contributory effect in increasing the FDI inflows to the country.

Fiscal Deficit: Interim Budget 2019The present government headed by Prime Minister Narendra Modi presented an uncompromising stanceon fiscal deficit in the beginning. In its first budget, the government had stated that “we cannot go onspending today which would be financed by taxation at a future date”. The government emphasised on“inter-generational equity” and not leaving behind “a legacy of debt for our future generations”highlighting its commitment for adhering to the fiscal deficit norms.Change in StanceThere was a deviation from this uncompromising stance on Fiscal Deficit during the second half of thegovernment’s tenure:

The fiscal deficit for 2017-18 deepened from the targeted 3.2% to 3.5% of GDP.The interim Budget has put the fiscal deficit target for 2018-19 at 3.4% of GDP, marginally higherthan the budgeted 3.3%.The targets under the Fiscal Responsibility Management Act also saw a revision over the years withthe original 3% target set for 2016-17 now being pushed to 2020-21.

The Medium Term Fiscal Policy-cum-Fiscal Policy Strategy Statement has attributed the fiscal slippage inthe last two years mainly on the lower-than-projected revenues from GST and the government having toprovide support to farmers on account of low crop prices. The critics attribute these fiscal slippages toaftermath impacts of the twin shocks of demonetisation and GST.The policy further states that the accrual of the full benefit of GST reforms and revenues is expected totake some more time and the stabilisation phase is expected to continue in 2019-20 too. Further, the fullfiscal impact of the income support scheme for farmers will also be felt in 2019-20.

Defence Budget 2019

sri vishnu charan | [email protected] |

https://t.me/PDF4Examshttps://t.me/IAS201819 https://t.me/PDF4Exams

https://t.me/TheHindu_Zone_official

Page 24: Current Affairs -February 1-15, 2019 · 15-02-2019  · 10, theme: ‘Sadak Suraksha-Jeevan Raksha’ National Testing Agency (NTA) launches mobile app through which students can

Current Affairs [PDF] -February 1-15, 2019

© 2019 GKToday | All Rights Reserved | https://www.gktoday.in 24

The Interim Budget 2019 has increased the allocations to the Defence Ministry by 6.87 per cent to Rs 3.18lakh crore against last year’s allocation of Rs 2.98 lakh crore.Allocations to Defence MinistryThe Defence Ministry budget outlay includes:

Of the total allocation of Rs 3.18 lakh crore, Rs 1,08,248 crore has been set aside for capital outlay forthe year 2019-2020 to purchase new weapons, aircraft, warships and other military hardware.Rs 1,12,079 crore has been set aside for payment of pensions.The defence budget is around 1.44 per cent of the GDP.Of the total capital outlay of Rs 1,08,248 crore, the Army was granted Rs 29,447 crore, the Navy wasgiven Rs 23,156 crore and the Indian Air Force got an allocation of Rs 39,302 crore.The revenue expenditure which includes expenses on payment of salaries and maintenance ofestablishments has been pegged at Rs 2,10,682 crore as against Rs 1,88,118 crore for 2018-19.The capital outlay for armed forces is 32.19 per cent of the government’s total capital expenditure ofRs 3,36,293 crore.

The allocation to the defence ministry for capital expenditure is said to be inadequate given the kind ofmodernisation our armed forces are envisaging.

Pradhan Mantri Shram Yogi MandhanThe Union Finance Minister Piyush Goyal announced the Pradhan Mantri Shram Yogi Mandhan scheme inthe interim budget 2019 for workers in the unorganised sector.About the SchemeThe features of the Pradhan Mantri Shram Yogi Mandhan scheme are:

Under the scheme, an assured monthly pension of Rs 3,000 per month will be provided to workers inthe unorganised sector after 60 years of age.To avail of the scheme, workers will have to contribute a minimal Rs 100 month per month.The scheme is expected to benefit 10 crore workers.This new pension scheme will run alongside the existing Atal Pension Yojana, which guaranteesreturns post-retirement.

The budget also increased gratuity limit from Rs 10 lakh to Rs 30 lakh from the next fiscal.The scheme is touted as to become the world’s biggest pension scheme for the unorganised sector in fiveyears. The scheme is designed as a tribute to the workers in the unorganised sector who contributearound 50 per cent of the country’s Gross Domestic Product (GDP).

Rishi Kumar Shukla appointed CBI DirectorIPS officer Rishi Kumar Shukla has been appointed as the chief of the Central Bureau of Investigation (CBI)for a fixed tenure of two years. Rishi Kumar Shukla is currently serving as the chairman of MadhyaPradesh Police Housing Corporation in Bhopal.The appointment of the Rishi Kumar Shukla was approved by the Appointments Committee of the cabinetbased on the recommendation of the selection committee headed by the Prime Minister. The CBI Directorsearch committee headed by Prime Minister has the Chief Justice of India and the Leader of Opposition asmembers.Supreme Court ObservationsThe appointment of the CBI director gains significance in the backdrop of comments made by theSupreme Court which had said that it was “averse” to the arrangement of an interim CBI Director and theCentre should “immediately” appoint a regular chief of the probe agency.The Supreme Court stated that the post of CBI Director is sensitive and important, and it is not good tokeep an interim director of the agency for a longer period. The Supreme Court sought to know why thegovernment has not made the appointment yet.

2 February: World Wetlands DayWorld Wetlands Day was celebrated on February 2nd with the theme of “Wetlands and Climate Change”.The theme signifies the importance of healthy and intact wetlands to one of the most pressing challengesof our times, climate change. The theme has been chosen to initiate action against the drainage ofwetlands.

sri vishnu charan | [email protected] |

https://t.me/PDF4Examshttps://t.me/IAS201819 https://t.me/PDF4Exams

https://t.me/TheHindu_Zone_official

Page 25: Current Affairs -February 1-15, 2019 · 15-02-2019  · 10, theme: ‘Sadak Suraksha-Jeevan Raksha’ National Testing Agency (NTA) launches mobile app through which students can

Current Affairs [PDF] -February 1-15, 2019

© 2019 GKToday | All Rights Reserved | https://www.gktoday.in 25

The theme Wetlands and Climate Change is aimed at increasing awareness of the importance of wetlands,such as swamps, marshes, mangroves or peatlands, to help us cope with and mitigate global warming.Significance of WetlandsThe importance of Wetlands is listed below:

Almost 90% of disasters are water-related and affects 60% of humanity that lives along coastlines byflooding and tsunamis. Wetlands are key to climate change mitigation.Wetlands function as a natural and extremely efficient carbon sink. For Example, peatlands whichcover only 3% of land mass, absorb and store twice the amount of carbon as all the world’s forestscombined.Wetlands act as a buffer for climate catastrophes that help communities be resilient against theimmediate impacts of climate change.

Why February 2nd is celebrated as World Wetland Day?The World Wetlands Day aims to bring attention towards the protection of the fragile wetlands which arethreatened by human activity. The rate of disappearance of Wetlands is 1% which is at a higher rate thandeforestation. Wetlands are also are exposed to draining and burning for agricultural enterprise and ruraldevelopment, as well as rising sea levels.World Wetlands Day is celebrated on 2 February 1971 to commemorate the signing of the Convention onWetlands, called the Ramsar Convention at the Iranian city of Ramsar on the shores of the Caspian Sea.The two-fold objectives of the Ramsar Convention are Conservation and sustainable utilization ofwetlands and stop the encroachment and loss of wetlands.

February 4, 2019Indian Foreign Exchange Reserves

The RBI data shows that Indian Foreign Exchange Reserves have been increased by USD 1.497 billion toreach USD 398.178 billion for the week to January 25. This increase has been attributed to a jump incore currency assets.Why the Foreign Exchange Reserves frequently fluctuate?The Foreign Exchange Reserves are expressed in terms of the US dollars are subject to variations due tothe appreciation/depreciation of non-US currencies such as the euro, pound and yen held in the reserves.Trends in Indian Foreign Exchange ReservesIndia’s Foreign Exchange Reserves fluctuations are listed below:

In the week previous to January 25, the foreign exchange reserves had dropped by USD 671 millionto USD 396.68 billion.The foreign exchange reserves of India had touched a record high of USD 426.028 billion in the weekto April 13, 2018. Since then it is mostly declining.The decline is largely attributed to the selling of the dollar by the RBI to contain rupee volatility.The central bank now holds 566.23 tonnes of the gold and purchase of 8.46 metric tonnes of gold wasmade in the fiscal year ending June 2018.In its annual Report, RBI had stated that the purchase of gold was made to diversify theforeign currency assets.

Foreign Exchange ReservesForeign Exchange Reserves are the reserve assets held by a central bank in foreign currencies. They areused to back liabilities on their own issued currency as well as to influence monetary policy. Thesereserves act as a buffer during the challenging times to the economy.The components of India’s FOREX Reserves which is expressed in terms of US dollars include foreigncurrency assets (FCAs), Gold Reserves, Special Drawing Rights (SDRs) and RBI’s Reserve position withInternational Monetary Fund (IMF). FCAs constitute the largest component of Indian Forex.

UNSC adopts resolution to extend sanctions against Central African RepublicThe United Nations Security Council resolution 2454 has been adopted unanimously. The features of theresolution 2454 are:

They extend the sanctions against the Central African Republic (CAR) for a year.The resolution renews an arms embargo, travel ban and asset freeze till January 31, 2020.

sri vishnu charan | [email protected] |

https://t.me/PDF4Examshttps://t.me/IAS201819 https://t.me/PDF4Exams

https://t.me/TheHindu_Zone_official

Page 26: Current Affairs -February 1-15, 2019 · 15-02-2019  · 10, theme: ‘Sadak Suraksha-Jeevan Raksha’ National Testing Agency (NTA) launches mobile app through which students can

Current Affairs [PDF] -February 1-15, 2019

© 2019 GKToday | All Rights Reserved | https://www.gktoday.in 26

The mandate of the panel of experts, which helps the Security Council with the implementation ofthe sanctions, until Feb 29, 2020, has been extended.The establishment of key benchmarks in various sectors that could help the Security Council toreview the arms embargo by April 30, 2019.

Sanctions against the Central African RepublicThe Central African Republic has been haunted by violence between Muslims and Christians sincepredominantly Muslim, Seleka rebels overthrew the Christian president in March 2013 and seized power.Anti-Balaka militias, mostly Christians, fought back, resulting in thousands of deaths and displacement.As a result, many fled to the country’s north and across the border into Chad and Cameroon. There was anincitement of violence based on religion and ethnicity.The UNSC unanimously adopted the Security Council Resolution 2127 in 2013 as a response to thebreakdown of law and order which saw increasing violence and widespread human rights abuses in theCentral African Republic and it is being renewed subsequently.The Resolution 2127 imposed several measures including a year-long embargo on the supply of arms,military equipment and related assistance to non-state actors in the CAR.

Surajkund International Crafts Mela-2019The Surajkund International Crafts Mela is organized every year by the Surajkund Mela Authority &Haryana Tourism in collaboration with Union Ministries of Tourism, Textiles, Culture and ExternalAffairs.The state of Maharashtra is the Theme State and Thailand is the Partner Nation Country for theSurajkund International Crafts Mela-2019.Objectives of the Surajkund International Crafts MelaThe Surajkund International Crafts is organised, managed and run Mela at Surajkund Faridabad with aview

To promote handicrafts, handlooms with the aid of craftsmen invited from all over the countryTo identify languishing and lesser-known crafts and to introduce them to patrons.To display crafts and loom techniques by organising demonstration sections in the Mela grounds.To undertake the promotion of export of handlooms and handicrafts.To set up an environment in which rural crafts traditions could be displayed and to project thetraditional rural ambience of a typical village near Delhi for travellers who may not have the time ormeans to visit an Indian village.

The Mela also aims to be a custodian of the heritage crafts involving the use of traditional skills that arefading away due to cheap machine-made imitations, and a special section is earmarked for showcasing ofthese heritage crafts.The theme state concept was conceptualized broadly with a view to promote and focus on art, craft andcuisine of each state of India every year.

Refugee wins the Australia’s richest literary prizeBehrouz Boochani, an Iranian asylum-seeker detained in Papua New Guinea under Australian asylum lawshas won the Victorian Prize for Literature. Behrouz Boochani was awarded for his work “No Friend But theMountains: Writing from Manus Prison”.The astonishing fact is that Boochani wrote the work on his phone and sent it to Omid Tofighian bit-by-bitin text messages. Omid Tofighian is the book’s translator who worked with Boochani over five years tobring the stories to life.The book “No Friend But the Mountains: Writing from Manus Prison” narrates experiences and fearBoochani felt in the guarded camp, which was shuttered last year after a local court ruling and theasylum-seekers moved elsewhere on the island.Australia Detention PolicyThe Australia Detention Policy deports asylum-seekers who try to enter the country by boat to ManusIsland or Nauru in the Pacific for processing, with those found to be refugees barred from resettlingin Australia.The international community is criticising Australia for its attempt to use remote Pacific islands such as

sri vishnu charan | [email protected] |

https://t.me/PDF4Examshttps://t.me/IAS201819 https://t.me/PDF4Exams

https://t.me/TheHindu_Zone_official

Page 27: Current Affairs -February 1-15, 2019 · 15-02-2019  · 10, theme: ‘Sadak Suraksha-Jeevan Raksha’ National Testing Agency (NTA) launches mobile app through which students can

Current Affairs [PDF] -February 1-15, 2019

© 2019 GKToday | All Rights Reserved | https://www.gktoday.in 27

Nauru as a ‘dumping ground’ for people seeking asylum and protection by boat to Australia.This offshore processing policy of Australia is giving rise to a completely avoidable health crisis andneedlessly destroying lives due to serious levels of mental illness, trauma, depression, self-harm, sexualassault and suicides.

India’s Mithali Raj Becomes First Woman Cricketer to Play 200 ODIsIndia’s One-day International (ODI) captain Mithali Raj has become the first woman to play 200 ODImatches with her appearance in the match against New Zealand. The record for largest number of gamesplayed by a Woman Cricketer was already in the name of Mithali Raj. Mithali Raj had achieved themilestone when she played her 192 ODI of the career, going past Charlotte Edwards’ record of 191appearances.The 36-year-old Mithali Raj is playing the game at the international circuit for almost twenty years andshe is also the fourth-highest among the cricketers as a whole, as Mithali is just behind Sachin Tendulkar(463), Sanath Jayasuriya (445) and Javed Miandad (233).Journey of Mithali RajMithali Raj made her ODI debut back on 25 June 1999 against Ireland in Milton Keynes. Mithali Raj is theleading run scorer in the ODIs among women with 6622 runs. Of the 213 matches India has played sinceher debut, she has played in all of them barring 13. Mithali Raj also holds the record for leading a countryin ODIs most number of times with 123 matches.

22nd AIIMS to come up in HaryanaThe 22nd AIIMS would come up in Haryana. The announcement was made regarding this at the InterimBudget 2019. The AIIMS would be established at Manethi village in Haryana’s Rewari district.All India Institute of Medical Sciences (AIIMS)All India Institute of Medical Sciences (AIIMS) is the autonomous institutions set up under the act ofparliament. The All India Institute Of Medical Sciences Act, 1956 provides for the establishment of AIIMS as Institute of National Importance.Objectives of AIIMS

To develop a pattern of teaching in undergraduate and postgraduate medical education in all itsbranches so as to demonstrate a high standard of medical education to all medical colleges and otherallied institutions in India.To bring together in one place educational facilities of the highest order for the training of thepersonnel in all important branches of the health activity.To attain self-sufficiency in postgraduate medical education.

Functions of AIIMSUndergraduate and postgraduate teaching in medical and related physical biological sciences.Nursing and Dental education.Innovations in education.Producing medical teachers for the country.Research in medical and related sciences.Health care : preventive, promotive and curative; primary, secondary & tertiary.Community based teaching and research

AIIMS Institutions aims to develop patterns of teaching in Undergraduate and Post-graduate MedicalEducation in all its branches so as to demonstrate a high standard of Medical Education in India and tobring together in one place educational facilities of the highest order for the training of personnel in allimportant branches of health activity; and to attain self-sufficiency in Post-graduate Medical Education.

GST collection increases to one lakh crore in JanuaryThe data from the Ministry of Finance shows that goods and services tax collections stood at Rs. 1,02,503crore in the month of January. Of the Rs. 1,02,503 crore GST revenue, Central GST is Rs 17,763 crore, StateGST (SGST) is Rs 24,826 crore, Integrated GST (IGST) is Rs 51,225 crore (including Rs. 24,065 crorecollected on imports) and Cess is Rs 8,690 crore.Rs 18,344 crore has been allocated to CGST and Rs 14,677 crore to SGST from IGST as a regularsettlement. The total revenue for the central government and state governments after regular settlement

sri vishnu charan | [email protected] |

https://t.me/PDF4Examshttps://t.me/IAS201819 https://t.me/PDF4Exams

https://t.me/TheHindu_Zone_official

Page 28: Current Affairs -February 1-15, 2019 · 15-02-2019  · 10, theme: ‘Sadak Suraksha-Jeevan Raksha’ National Testing Agency (NTA) launches mobile app through which students can

Current Affairs [PDF] -February 1-15, 2019

© 2019 GKToday | All Rights Reserved | https://www.gktoday.in 28

in December 2018 is Rs 36,107 crore for CGST and Rs 39,503 crore for SGST.The revenue has increased from Rs 94,725 crore in December 2018, and 14 per cent higher than January2018 collection of Rs 89,825 crore.Trends in GST collection

Based on data from the Ministry of FinanceFor the third time in current fiscal, the revenues from Goods and Services Tax (GST) has crossed Rs 1 lakhcrore mark. Earlier in April and October, the collections had surpassed Rs 1 lakh crore. GST collectionstood at Rs 1.03 lakh crore in April, Rs 94,016 crore in May, Rs 95,610 crore in June, Rs 96,483 crore in July,Rs 93,960 crore in August, Rs 94,442 crore in September, Rs 1,00,710 crore in October, Rs 97,637 crore inNovember and Rs 94,725 crore in December 2018.

US, Russia suspend INF TreatyWith the formal announcement by the United States about the withdrawal from the INF treaty, Russia hasfollowed the suit and declared the suspension of the treaty. Defence analysts fear about the arms race dueto the suspension of the treaty.Intermediate-Range Nuclear Forces TreatyThe features of the Intermediate-Range Nuclear Forces (INF) Treaty signed by President Ronald Reaganand Soviet General Secretary Mikhail Gorbachev in December 1987 were:

The treaty prohibited the United States and the Soviet Union from possessing, testing and deployingground-launched cruise and ballistic missiles of ranges between 500 and 5,500 kilometres (300 to3,400 miles).The treaty covers all types of ground-launched cruise and ballistic missiles whether their payload isconventional or nuclear.The treaty exempted the air-launched and sea-based missile systems in the same range.

Under the treaty, both US and Russia had destroyed 846 and 1,846 missiles respectively.How the INF treaty aided in Diffusing Tensions?Due to the limited range, Short flight times and unpredictable flight patterns, It was difficult to detect theshort and medium ranges missiles. As a result, there was a threat of nuclear war in Europe which issandwiched between Russia and US.The missiles were designed chiefly to fight a theatre nuclear war in Europe. It exacerbated crisisinstability and increased the chances of an accidental nuclear war.Hence the destruction of these missiles under the provisions of the INF treaty was highly beneficialtowards enhancing both regional and global security.What would happen now?Both Russia and US would indulge in the development of these short and medium ranges missiles. There

sri vishnu charan | [email protected] |

https://t.me/PDF4Examshttps://t.me/IAS201819 https://t.me/PDF4Exams

https://t.me/TheHindu_Zone_official

Page 29: Current Affairs -February 1-15, 2019 · 15-02-2019  · 10, theme: ‘Sadak Suraksha-Jeevan Raksha’ National Testing Agency (NTA) launches mobile app through which students can

Current Affairs [PDF] -February 1-15, 2019

© 2019 GKToday | All Rights Reserved | https://www.gktoday.in 29

could be an arms race with EU joining the race as a third entity. Since these missiles are mobile, hard todetect, nuclear-capable and can reach European cities, they have hardly any warning time at all so theyreduce the threshold for any potential use of nuclear weapons in a conflict. This would lead to instabilityand trust deficit.

Iran successfully test fires Hoveizeh Cruise MissileIran announced the successful test flight of Hoveizeh long-range cruise missile on February second whichmarks the 40th anniversary of 1979 Islamic revolution.The Hoveizeh Cruise Missile is part of the Soumar family of cruise missile and has a range of over 1,350 km(840 miles). It is designed to use against ground targets. The Hoveizeh missile needs a very short time forits preparedness and can fly at a low altitude and is manufactured by the Aerospace IndustriesOrganization of Iran.1979 Islamic RevolutionThe democratically elected prime minister in Iran was thrown out and the Shah was restored to thethrone with the help of American CIA. In the times of cold war, the US wanted to keep the Shah in poweras a bulwark against the Soviet Union.Iran was reaping enormous benefits from oil production and the gap was widening between the wealthyand the poor. The recession in 1975 led to tension between the classes.In October 1977, the son of the Shia cleric Ayatollah Khomeini died of heart attack and the rumours spreadthat he had been murdered by the SAVAK secret police. Shah was battling for life due to cancer at thetime. Shah had his Information Minister published an article in the leading newspaper that slanderedAyatollah Khomeini was a tool of British neo-colonial interests and a man without faith.This angered people and the theology students in the city of Qom exploded in angry protests. To suppressthe protests the shah came down heavily on the protestors and what followed as a massacre at Qom.Further, there were series of protests across Iran against the misadventures of Shah and hisadministration which culminated in the revolution of 1979 where the monarch of Iran, Mohammad RezaShah Pahlavi was overthrown and his government was replaced with an Islamic republic under the GrandAyatollah Ruhollah Khomeini, a leader of one of the factions in the revolt.

PM inaugurates development projects in Jammu and KashmirThe Prime Minister Narendra Modi inaugurated various development works during his visit to the stateof Jammu and Kashmir. The developmental works include:

Laying of the foundation stones of two All India Institutes of Medical Sciences (AIIMS) in Vijaypurand Awantipora.Laying of the foundation stone of northern regional centre campus of Indian Institute of MassCommunication in Jammu.Launching of the University of Ladakh, which will be the first-ever university in the Ladakh region ofthe state.Launching of various projects under the Rashtriya Uchchatar Shiksha Abhiyan.Laying of the foundation stones for three model degree colleges in Kishtwar, Kupwara andBaramulla in Jammu and Kashmir.Declaration of 100-per cent electrification of households in Jammu and Kashmir under the centralgovernment’s Saubhagya Scheme.Laying the foundation stone for 624-MW Kiru hydroelectric project in Kishtwar which is a run-of-the-river project across the Chenab.Inaugurating the 9-MW Dah hydroelectric project.Inauguration of the 220-KV Srinagar-Alusteng-Drass-Kargil-Leh transmission system.Inauguration of the 400-KV D/C Jalandhar-Samba-Rajouri-Shopian-Amargarh(Sopore) transmission line.Laying the foundation stone of 1,640-metre span double-lane bridge over the Chenab river in Sajwal.

Many of these projects are undertaken under the Prime Minister’s Development Package was announcedas a reconstruction plan for J&K.

sri vishnu charan | [email protected] |

https://t.me/PDF4Examshttps://t.me/IAS201819 https://t.me/PDF4Exams

https://t.me/TheHindu_Zone_official

Page 30: Current Affairs -February 1-15, 2019 · 15-02-2019  · 10, theme: ‘Sadak Suraksha-Jeevan Raksha’ National Testing Agency (NTA) launches mobile app through which students can

Current Affairs [PDF] -February 1-15, 2019

© 2019 GKToday | All Rights Reserved | https://www.gktoday.in 30

February 5, 2019February 4: World Cancer Day

Every year World Cancer Day is observed on February 4. The day is observed to save millions ofpreventable deaths each year through education, raising awareness and by pressing governments andindividuals across the world to take action against the deadly disease.I Am and I WillOn the World Cancer Day, the Union for International Cancer Control launched a new 3-year campaignwith the theme: “I Am and I Will”. The campaign is aimed to increase public-facing exposure andengagement to inspire the change that is required to build a cancer-free future for the world.Burden of CancerThe global burden of cancer is estimated to have increased to 18.1 million new cases and 9.6 million deathsin 2018. Approximately one in five men and one in six women worldwide develop cancer in their lifetime,and one in eight men and one in 11 women die from the disease.In India around 2.25 million cases with over 1 lakh new cases being registered every year. The disease ledto nearly 7 lakh deaths in 2018. The Indian Council of Medical Research (ICMR) estimates that India islikely to register over 17 lakh new cases and report over 8 lakh deaths by 2020.If some serious interventions are not made to alter the course of the disease, the global burden is expectedto rise to close to 30 million new cases by 2040.

Braille Voter Slips for Visually Impaired and BlindThe Election Commission of India will be issuing Braille voter slips for the visually impaired and blindvoters in the Lok Sabha elections due this year. The Braille voter slips were already issued during therecently held assembly polls.The initiative is part of the strategic framework for ‘accessible elections’, The Election commission of Indiaalso plans to provide Braille election photo identity cards.The Electronic voting machines already have the Braille feature. The ballot paper pasted on the machinewith name and symbol of candidates is Braille enabled.

BrailleBraille is a script with raised dots that can be read with the fingers by people who are blind or who have low vision.It is a tactical reading and writing system used by blind and visually impaired people who cannot access print material.

Strategic Framework on Accessible ElectionsThe Strategic Framework on Accessible Elections is a compilation of guidelines issued by ElectionCommission of India and the Department for Empowerment of Persons with Disabilities, Ministry ofSocial justice and Empowerment, Government of India.The guidelines aim to build on sensitization through education and training, community involvement ofdifferent sections of society, effective partnership with institutions and originations and creation offacilities to cater to the specific needs of Persons with Disabilities (PwDs) with a view to increasing theirparticipation.

Manipur filmmaker to return Padma Shri in protest against Citizenship Amendment BillRenowned Filmmaker from Manipur Aribam Syam Sharma has decided to return his 2006 Padma Shriaward in protest against the Citizenship (Amendment) Bill, 2016.Why the Citizenship (Amendment) Bill, 2016 has become controversial?The citizenship amendment bill 2016 aims to amend the Citizenship Act of 1955. The controversialfeatures of the bill are

The bill allows the Hindus, Sikhs, Buddhists, Jains, Parsis and Christians from Bangladesh,Afghanistan and Pakistan to apply for Indian citizenship.The bill relaxes the 11-year cutoff to six years out of 14, for immigrants of the six religions from thethree countries.

The bill is criticised for being communal as it grants citizenship on the basis of religion. It is also arguedthat the bill is against the basic structure of the constitution as the Supreme Court has upheld secularismas a basic structure of the constitution. It is said that the bill also violates Article 14 which promotes

sri vishnu charan | [email protected] |

https://t.me/PDF4Examshttps://t.me/IAS201819 https://t.me/PDF4Exams

https://t.me/TheHindu_Zone_official

Page 31: Current Affairs -February 1-15, 2019 · 15-02-2019  · 10, theme: ‘Sadak Suraksha-Jeevan Raksha’ National Testing Agency (NTA) launches mobile app through which students can

Current Affairs [PDF] -February 1-15, 2019

© 2019 GKToday | All Rights Reserved | https://www.gktoday.in 31

equality.Why the heat against the bill is more in North East?The issue of illegal immigrants is a sensitive issue in the North-East. Often the states have witnessedclashes and communal riots due to tensions between immigrants and the locals.The bill is aimed at providing citizenship to illegal immigrants. The North-Eastern states share borderswith Bangladesh. It is said that due to porous borders between India and Bangladesh, lakhs of illegalimmigrants have crossed over to India and are majorly concentrated in these northeastern states.The North-Eastern States have often witnessed protests seeking identification of these illegal immigrantsand deporting them. Assam Accord and National Registry of Citizens were aimed at curbing the menace ofthe illegal immigrants.The people of North East fear that if the bill is implemented their struggle for years would be underminedand there would be a heavy Social, Economical and political burden on the North-East.

Judicial Complex Inaugurated at AmaravatiT h e C h i e f J u s t i c e o f I n d i a R a n j a n G o g o i a n d A n d h r a P r a d e s h C h i e f M i n i s t e r N .Chandrababu Naidu inaugurated the judicial complex in Amaravati. The judicial complex will house theinterim High Court for Andhra Pradesh. The Chief Justice of India also laid the foundation stone for theconstruction of a permanent High Court to be built in Nelapadu of Guntur district.The construction of new high court was necessitated after the bifurcation of Andhra Pradesh in 2014. TheHigh Court which was common for both Andhra Pradesh and Telangana till 2018 and recently theseparate high courts were notified by the union government after the order from the Supreme Court.Judicial complex in AmravatiThe Judicial Complex in Amravati was built by Andhra Pradesh Capital Region DevelopmentAuthority within a short span of eight months. The total construction cost is estimated around Rs 173crores. As a temporary arrangement, the complex will house the High Court of Andra Pradesh.Once the construction of the new high court at Nelapadu of Guntur district is completed, the complex willhouse the city civil court.Originally the first High Court of Andra Pradesh was inaugurated on July 5, 1954, in Guntur and wasmoved to Hyderabad only in October 1956.

Tourism Sector in IndiaUnion Minister for Tourism K J Alphons delivering the inaugural speech at the 2nd ASEAN-India YouthSummit highlighted the contribution of the of the tourism sector to the GDP as below:

India’s tourism sector generated USD 234 billion revenue in 2018 and the sector registered a growthof over 19 per cent.India was ranked third in the tourism sector, according to the 2018 report of the World Travel andTourism Council (WTTC).The global revenue growth from tourism sector was five per cent and Indian tourism sector grew by19.4 per cent.87 per cent of revenue was contributed by domestic tourists and 13 per cent foreign tourists.The earnings from foreign tourists were about USD 27 billion.The revenue from the foreign tourists grew by about 14 per cent in India compared to global growthof 7 per cent.Spiritual tourism contributed around 60-70 per cent of the total domestic tourists.

ASEAN-India Youth SummitThe ASEAN-India Youth Summit aims to provide a platform for youth leaders of both India and ASEAN toengage with each other to develop a resilient and symbiotic relationship between countries.The ASEAN-India Youth Summit is organised by India Foundation, in collaboration with ASEANSecretariat and Ministry of External Affairs, Government of India (MEA).The first India-ASEAN Youth Summit was held in 2017 and the theme was Shared Values, CommonDestiny. The second India-ASEAN Youth Summit was held in Guwahati with the theme Connectivity:Pathway to Shared Prosperity.

sri vishnu charan | [email protected] |

https://t.me/PDF4Examshttps://t.me/IAS201819 https://t.me/PDF4Exams

https://t.me/TheHindu_Zone_official

Page 32: Current Affairs -February 1-15, 2019 · 15-02-2019  · 10, theme: ‘Sadak Suraksha-Jeevan Raksha’ National Testing Agency (NTA) launches mobile app through which students can

Current Affairs [PDF] -February 1-15, 2019

© 2019 GKToday | All Rights Reserved | https://www.gktoday.in 32

Road Safety Week 2019The Road Safety Week was initiated by the Ministry of Road Transport & Highways with an aim to raisepublic awareness about traffic rules and ultimately reduce casualties due to road accidents. CharitableOrganizations, Non-Government Organizations and Private firms across the country join hand with thegovernment to provide logistic support to the Road Safety Week Campaign.The Road safety week is observed annually to make people more aware of the traffic rules and to persuadethem to follow rules while on road.Road Safety Week 2019India is celebrating the 30th Road Safety Week from 04th February (Monday) to the 10th February(Sunday). The following activities are organised as part of the Road Safety Week 2019:

Road safety leaflets including roses, chocolates and flowers are distributed to the travellers on theroad.Commuters are sensitized about the methods and necessities of the road safety means they mustunderstand the use of helmets or seat belts while driving on the road or anywhere.Various painting and drawing competitions, road safety announcements, exhibitions, road rules test,girls scooter rally to encourage the use of helmets, debates on road safety at the All India Radio,workshops, seminar and etc activities are organized.Free medical checkup camps and driving training workshops are organized for the drivers toencourage them towards road safety.Road safety quiz competitions are also organized to promote people about road safety.Traffic safety games including card games, puzzles, board games and etc are organized to educateschool children about road safety.

The road accidents are mainly attributed to behavioural tendencies such as habitual flouting of trafficrules like- not wearing a helmet, not wearing a seat belt, jumping traffic lights, over speeding, drunkendriving etc. Road Safety Week is aimed at making people aware of the perils of not following traffic rulesand the emotional and financial trauma that it could bring to their families.

National Testing Agency takes measures to Acquaint AspirantsThe National Testing Agency (NTA) which is vested with the responsibility of the conduction ofcompetitive exams has taken the following measures to acquaint aspirants with the computer-basedexaminations:

A mobile application has been launched through which students can practice or take mock tests ontheir smartphones.The National Testing Agency has also launched a network of more than 4,000 Test Practice Centres(TPCs).

These measures are aimed at ensuring the aspirants especially those from rural areas are not atdisadvantage due to the mode of the exam.National Testing AgencyNational Testing Agency is a premier, specialist, autonomous and self-sustained testing organization toconduct entrance examinations for admission/fellowship in higher educational institutions.The National Testing Agency is entrusted with the responsibility to address the challenge of assessing thecompetence of candidates through an efficient, transparent and error-free process.Functions of the National Testing AgencyThe functions of the National Testing agency are:

To identify partner institutions with the necessary infrastructure from the existing schools andhigher education institutions to facilitate the conduct of online examinations without adverselyimpacting their academic routine.To set-up a question bank for all subjects using modern techniques.To establish a strong R&D culture as well as a pool of experts in different aspects of testing.To help individual colleges and universities in the field of testing and to provide training andadvisory services to institutions in India.To provide quality testing services to the academic institutions in India.

sri vishnu charan | [email protected] |

https://t.me/PDF4Examshttps://t.me/IAS201819 https://t.me/PDF4Exams

https://t.me/TheHindu_Zone_official

Page 33: Current Affairs -February 1-15, 2019 · 15-02-2019  · 10, theme: ‘Sadak Suraksha-Jeevan Raksha’ National Testing Agency (NTA) launches mobile app through which students can

Current Affairs [PDF] -February 1-15, 2019

© 2019 GKToday | All Rights Reserved | https://www.gktoday.in 33

To develop a state of the art culture of testing in India by using domestic and internationalexpertise. To collaborate with international organizations like ETS to achieve the same.To undertake any other examination that is entrusted to it by the Ministries/Departments ofGovernment of India/State Governments.To undertake the reforms and training of school boards as well as other bodies where the testingstandards should be comparable with the entrance examinations.

The National Testing Agency will be chaired by an eminent educationist and the governing council of theNational Testing agency is headed by the Secretary, Department of Higher Education under the Ministryof Human Resource and Development.The National Testing agency is successfully conducting Joint Entrance Examination (JEE) main forundergraduate engineering admissions and National Eligibility cum Entrance Test (NEET) for medicaladmissions.

Foreign Direct Investment to IndiaThe Ministry of Commerce and Industry has released the data related to the Foreign Direct Investment(FDI) inflows during April-September 2018-19. The important aspects from this data:

The Foreign direct investment (FDI) into India was declined by 11 per cent to USD 22.66 billionduring April-September period of 2018-19.The FDI inflows during April-September 2017-18 stood at USD 25.35 billion.The FDI which attracted large FDI are services (USD 4.91 billion), computer software and hardware(USD 2.54 billion), telecommunications (USD 2.17 billion), trading (USD 2.14 billion), chemicals (USD1.6 billion), and automobile industry (USD 1.59 billion).Singapore with the FDI of USD 8.62 billion inflow was the largest source of FDI during April-September 2018-19.Singapore was followed by Mauritius (USD 3.88 billion), the Netherlands (USD 2.31 billion), Japan(USD 1.88 billion), the US (USD 970 million), and UK (USD 845 million).

The Foreign Direct Investment growth witnessed a five-year low growth of 3 per cent at $ 44.85 billion in2017-18. The decline in the growth rate of FDI could adversely affect the country’s balance of paymentsand may also impact the value of the rupee.

Fiscal Deficit for April-DecemberThe data shows that the fiscal deficit touched 112.4 per cent of the full-year budget target of Rs 6.24 lakhcrore at the end of December. The important data highlights related to the fiscal deficit are:

The gap between Government’s expenditure and revenue stood at Rs 7.01 lakh crore during April-December of the current financial year which ends in March.The deficit was 113.6 per cent of the Budget Estimate by the end of December.The government had earlier budgeted to cut the fiscal deficit to 3.3 per cent of GDP or Rs 6.24 lakhcrore in 2018-19, from 3.53 per cent in the previous financial year.The interim budget for 2019-20 revised the fiscal deficit upwards marginally to 3.4 per cent of GDPor over Rs 6.34 lakh crore, on account of the additional outlay of Rs 20,000 crore for funding incomescheme for small farmers.The revenue receipts of the government totalled Rs 10.84 lakh crore or 62.8 per cent of BE in 2018-19till December, compared with 66.9 per cent during the same period last year.The revenue receipts stood at 66.9 per cent during the same period last year.The earlier estimate to mobilise Rs 17.25 lakh crore revenue during the current fiscal has beenrevised upwards to over Rs 17.29 lakh crore in the 2019-20 interim budget.Tax revenue was 63.2 per cent of budget estimates compared with 73.4 per cent in the comparableperiod of the previous year.The total expenditure of the government at December-end was Rs 18.32 lakh crore or 75 per cent ofbudget estimates.

The increase in the fiscal deficit has been attributed to lower revenue collections.

sri vishnu charan | [email protected] |

https://t.me/PDF4Examshttps://t.me/IAS201819 https://t.me/PDF4Exams

https://t.me/TheHindu_Zone_official

Page 34: Current Affairs -February 1-15, 2019 · 15-02-2019  · 10, theme: ‘Sadak Suraksha-Jeevan Raksha’ National Testing Agency (NTA) launches mobile app through which students can

Current Affairs [PDF] -February 1-15, 2019

© 2019 GKToday | All Rights Reserved | https://www.gktoday.in 34

Azim Premji awarded EY Lifetime Achievement AwardAzim Premji Chairman of Wipro Ltd will be felicitated with the Ernst & Young Lifetime AchievementAward. Ernst & Young, famously known as EY is a multinational professional services firm and is one ofthe “Big Four” accounting firms.Motivational Journey of Azim PremjiAzim Premji was born on July 24, 1945, to an affluent Gujarati entrepreneur. He inherited Wipro, avegetable oil company at the age of 21 after the sudden demise of his father.Azim Premji was pursuing Electrical Engineering from Stanford University, USA when he took over thereins of his family business in 1966. He was written off to carry the herculean task and was publiclyadvised to sell his shareholding and give it into more mature hands. This further strengthened his resolveto make Wipro a success story.Under his able leadership, Wipro diversified from hydrogenated cooking fats to bakery fats, ethnicingredient based toiletries, hair care soaps, baby toiletries, lighting products and hydraulic cylinders.The expulsion of IBM from the Indian market led to a vacuum and Wipro ventured into software servicestaking advantage. The rest is history. He led Wipro from a hydrogenated cooking fats company to apioneer in providing integrated business, technology and process solutions on a global delivery platform.Wipro Technologies is the largest independent R&D service provider in the world and is ranked amongthe top 100 technology companies globally.In 2005 Premji was conferred the Padma Bhushan and Padma Vibhushan in 2011. Premji has also pledgedto donate USD 2 billion for improving school education in India, the first of its kind by any Indianbillionaire.

Motor Vehicles (Amendment) Bill, 2017The Union Minister of Road Transport and Highways, Nitin Gadkari has stated that the CentralGovernment expects a 20 per cent decline in road accidents from the current five lakh a year andParliament’s nod to pending legislation Motor Vehicles (Amendment) Bill will also help curb the accidents.Features of the BillThe features of the Motor Vehicles (Amendment) Bill, 2017 are:

The bill seeks to redress the difficulties faced in obtaining driving licenses without the help of toutsby taking the process online. Tests for driving licences will be automated, and learner’s licences willbe issued online.The bill provides for an increase of fines for breaking road rules.The new law states that driving licence issued to a person under the age of 30 is valid till the personturns 40. For those who receive licences between the ages of 30 and 50, the licence will remain validfor 10 years. If the licence is issued between 50 and 55 years, it will be valid until the person turns 60,and above 55 years, licences will carry a five-year validity.The bill defines aggregators as a digital intermediary or market place for a passenger to connectwith a driver for the purpose of transportation and makes it optional for the states to follow centralguidelines related to the aggregators.The bill leaves the regulation of aggregators should be left to states.The bill also removes the cap on payments to be made under third-party insurance proposed in the2016 bill.The bill provides for the recall of vehicles if the defective vehicle is a danger to the environment, thedriver or other road users. The manufacturer would be required to reimburse all buyers with the fullcost of the vehicle, replace the defective vehicle, and if necessary pay a fine as specified by thegovernment.

The Motor Vehicles (Amendment) Bill, 2017 has been passed by the Lok Sabha and is pending in the RajyaSabha.

UK Home Secretary approves extradition of Vijay Mallya to IndiaThe UK home secretary has ordered the extradition of Vijay Mallya over allegations of £1bn fraud. The UKhome secretary noted that Vijay Mallya, the first to be declared Fugitive Economic Offender is accused inIndia of conspiracy to defraud, making false representations and money laundering offences.

sri vishnu charan | [email protected] |

https://t.me/PDF4Examshttps://t.me/IAS201819 https://t.me/PDF4Exams

https://t.me/TheHindu_Zone_official

Page 35: Current Affairs -February 1-15, 2019 · 15-02-2019  · 10, theme: ‘Sadak Suraksha-Jeevan Raksha’ National Testing Agency (NTA) launches mobile app through which students can

Current Affairs [PDF] -February 1-15, 2019

© 2019 GKToday | All Rights Reserved | https://www.gktoday.in 35

The liquor baron is still defiant and he has said that he would appeal against the extradition.Vijay Mallya and his ExtraditionThe judge hearing Mallya’s extradition request noted that Mallya had misrepresented how loans fromIndian banks were used. The judge described Mallya as a glamorous, flashy, famous, bejewelled,bodyguarded, ostensibly billionaire playboy who charmed and cajoled these bankers into losing theircommon sense and persuading them to put their own rules and regulations to one side.The Indian investigation agencies allege that Mallya has misled Indian Banks about the fortunes of hisfailing Kingfisher airline, before laundering the cash to fund his Formula One team and other projects. TheEnforcement Directorate is investigating the tycoon’s £977m debts linked to the airline, which went bustin 2012.

Madras High Court Rules against Negative Marking in Competitive ExamsThe Madras High Court has ruled against negative marking in competitive examinations saying they arebad in law.Observations of the Madras High CourtThe Madras High Court while hearing a petition filed by an IIT JEE aspirant who failed to clear the Mainsdue to negative marking has made the following observations:

The Madras High Court has accepted the petitioner’s argument that negative marking is notprevailing anywhere else in the world.The court said that negative marking acts only as “a bolt in the brain development” of students andprevents them from making intelligent guesses.Deducting marks for wrong answers would not in any way help in analysing the intelligence,aptitude or knowledge of the candidates.Every candidate could not be expected to know all answers for sure. In such circumstances, thepractice of negative marking would hamper brain development and create a fear psychosis amongstudents.The court rejected the CBSE counsel’s argument that in Indian context the practice is necessary.

The judge stated that “Intelligent guessing is an art. It is very useful in our life. One cannot be sure aboutall things at all times. An individual will come across a situation where he/she has to decide an issue notmerely based on his knowledge but with little guessing. While intelligent guessing requires an amount ofprior knowledge on the subject, wild guessing is a decision taken just like that”The Madras High Court ruled not to give negative marks for wrong answers and directed the CBSE tocommunicate the order to the National testing Agency which conducts the JEE (Main) exam.

Conservation and Promotion of Medicinal PlantsThe National Medicinal Plants Board (NMPB) under the Ministry of AYUSH is implementing followingschemes to encourage farming/cultivation, conservation, processing and promotion of medicinal plantsthroughout the country:National AYUSH Mission (NAM)The large scale farming/cultivation of medicinal plants is being supported under the ‘Medicinal Plants’component of the National AYUSH Mission (NAM). The scheme provides support for:

Cultivation of prioritized medicinal plants on farmer’s land.Establishment of nurseries for the supply of quality planting material.Post-harvest management.Primary processing, marketing infrastructure etc.

Conservation, Development and Sustainable Management of Medicinal PlantsThe central sector scheme Conservation, Development and Sustainable Management of Medicinal Plantsprovides project-based support for following activities:

In-situ conservation through the development of Medicinal Plants Conservation and DevelopmentAreas (MPCDAs).In-situ/Ex-situresource augmentation.Ex-situ conservation through the establishment of herbal gardens.Livelihood linkages with Joint Forest Management Committees (JFMCs) / Panchayats / Van

sri vishnu charan | [email protected] |

https://t.me/PDF4Examshttps://t.me/IAS201819 https://t.me/PDF4Exams

https://t.me/TheHindu_Zone_official

Page 36: Current Affairs -February 1-15, 2019 · 15-02-2019  · 10, theme: ‘Sadak Suraksha-Jeevan Raksha’ National Testing Agency (NTA) launches mobile app through which students can

Current Affairs [PDF] -February 1-15, 2019

© 2019 GKToday | All Rights Reserved | https://www.gktoday.in 36

Panchayats / Biodiversity Management Committees (BMCs) / Self Help Groups (SHGs).IEC activities like Training / workshops / Seminars/ Conferences etc.Research & Development.Promotion of marketing and trade of medicinal plants produce.

Voluntary Certification Scheme for Medicinal Plants ProduceThe scheme is aimed to encourage Good Agricultural Practices (GAPs) and Good Field Collection Practices(GFCPs) in medicinal plants and enhance the quality and safety of their produce.As per information of the Botanical Survey of India (BSI), an organization under the Ministry ofEnvironment, Forests & Climate Change estimated more than 8,000 species of medicinal plants are foundin India.

Exercise RahatThe disaster relief exercise ‘Exercise Rahat’ will be demonstrated in Jaipur, Kota and Alwar in Rajasthanon Feb 11-12.About the ExerciseThe important aspects related to the ‘Exercise Rahat’ are:

On behalf of the Indian Army, Jaipur based Sapta Shakti Command will conduct the JointHumanitarian Assistance and Disaster Relief Exercise, Exercise Rahat.Exercise Rahat is being conducted in coordination with NDMA to synergise efforts for humanitarianassistance and disaster relief operations.The joint exercise will see participation from Armed Forces, National Disaster ManagementResponse Mechanism (NDMRM), State Disaster Management Authority of Rajasthan and DistrictLMAs.The exercise will be conducted simultaneously in three places, beginning at Jaipur in the form of atabletop exercise and at Kota and Alwar.During the exercise, on-ground capability and coordination amongst various stakeholders will bedemonstrated.

As a prelude to the exercise, a curtain raiser has been organized at Jaipur Military station on 4th February2019.

February 6, 2019Schemes of National Minorities Development and Finance Corporation

The National Minority Development Finance Corporation (NMDFC) is implementing various schemes forthe socio-economic development of the ‘backward sections’ amongst the notified minorities. The schemesare being implemented through the State Channelising Agencies (SCAs) nominated by the respective StateGovernments/UT Administration.Categorisation of BeneficiariesThe NMDFC categorises the beneficiaries under two categories, They are:

Credit Line-1: For those with the annual family income 98,000 for rural areas & Rs.1.20 lacs for urbanareas.Credit Line-2: For those with higher annual family income eligibility criterion of up to Rs.6.00 Lacs.

Schemes of NMDFCCredit SchemesThe various credit schemes like Term Loan, Micro Finance, Education Loan and Mahila Samridhi Yojanaare introduced to enhance the livelihood opportunities by promoting income generation activities.

Mahila Samridhi Yojana:Skill development training is imparted to a group of women in women-friendly trades under the scheme and astipend @ Rs.1,000 is available for each woman during the training period.During the period of training, the women are formed into Self Help Group, followed by infusion of micro-creditmaximum up to Rs.1.00 lacs per member for the purpose of using the skill developed during the training, for incomegeneration activities.

Promotional SchemesNMDFC is implementing promotional schemes like Vocational Training & Marketing Support for with

sri vishnu charan | [email protected] |

https://t.me/PDF4Examshttps://t.me/IAS201819 https://t.me/PDF4Exams

https://t.me/TheHindu_Zone_official

Page 37: Current Affairs -February 1-15, 2019 · 15-02-2019  · 10, theme: ‘Sadak Suraksha-Jeevan Raksha’ National Testing Agency (NTA) launches mobile app through which students can

Current Affairs [PDF] -February 1-15, 2019

© 2019 GKToday | All Rights Reserved | https://www.gktoday.in 37

preference to women beneficiaries.Vocational Training SchemeUnder the Kaushal Se Kushalta scheme skill development training programs of 200 to 250 hrs durationare organized and Stipend of Rs.1,000 per candidate per month is provided. The schemesguarantee placement of minimum 70% candidates trained under the scheme.Marketing Assistance SchemeThe Marketing Assistance Scheme envisages promoting sale & marketing of their products atremunerative prices through participation /organizing exhibitions at State/District level.

Steps to Strengthen the functioning of the NMDFCThe following steps have been taken to assist the National Minority Development Finance Corporation(NMDFC) for effective implementation of its schemes:

The Ministry of Minority Affairs is providing grant-in-aid assistance for strengthening theinfrastructure & operational capability of State Channelising Agencies (SCAs), for effectiveimplementation of NMDFC schemes.A new Annual Family Income eligibility criterion of up to Rs.6.00 lacs per annum has beenintroduced for greater coverage of persons from the targeted minority communities.Quantum of loans under the Term Loan scheme increased from Rs.10.00 lacs to Rs.30.00 lacs, whileunder Micro Finance scheme, it has been increased from Rs.0.50 lacs to Rs.1.50 lacs per SHGmember.Under the Education Loan scheme, the quantum of loan has been increased from Rs.5.00 lacs toRs.20.00 lacs for domestic courses & from Rs.10.00 lacs to Rs.30.00 lacs for courses abroad.Self Declaration/Self Certification/Self Attestation of documents has been adopted in case of ReligionCertificate, Family Income, Residence Proof, Mark Sheet, etc.Transfer of loan directly in Bank Account of Beneficiary through National Electronic Funds Transfer(NEFT)/ Real Time Gross Settlement (RTGS) Insurance of beneficiary and their assets to safeguardagainst any untoward incident.

National Minorities Development and Finance Corporation (NMDFC)The National Minorities Development and Finance Corporation (NMDFC) is a not for profit companyunder the companies act 1956. It works under the aegis of Union Ministry of Minority Affairs and wasconstituted in 1994.

Minority Communities in IndiaThe notified Minorities under the National Commission for Minorities Act, 1992, are Muslims, Christians, Sikhs,Buddhists & Parsis. Jain community was also added into the list of notified Minority Communities in January 2014.

NMDFC aims to provide concessional finance to the notified Minorities for the self-employment/ incomegeneration activities.

Steps to Address Cross-Border InfiltrationThe cross-border infiltration and illegal immigration are posing serious security threats to both internaland external security. The government has undertaken following steps to arrest the cross-borderinfiltration:

Multipronged approach to check infiltration by patrolling; sharing of intelligence and conduct ofjoint operations; erection of border fencing; installation of border floodlights; use ofwatercrafts/boats and floating Border Out Posts (BOPs) for domination of riverine area of theborder; liaison with police, intelligence agencies, local government agencies; laying nakas;establishment of observation posts have been undertaken.Surveillance equipment like Night Vision Devices (NVDs); Hand Held Thermal Imagers (HHTIs);Long Range Recce Observation Systems (LORROS), etc have been deployed for effective technicalsurveillance.Mobile Vehicle Check Posts (MVCPs), Area Domination Patrols (ADPs), short and long-range patrolsalong the border are being launched on regular basis.Review of vulnerability mapping of BOPs from the point of infiltration is carried out.

sri vishnu charan | [email protected] |

https://t.me/PDF4Examshttps://t.me/IAS201819 https://t.me/PDF4Exams

https://t.me/TheHindu_Zone_official

Page 38: Current Affairs -February 1-15, 2019 · 15-02-2019  · 10, theme: ‘Sadak Suraksha-Jeevan Raksha’ National Testing Agency (NTA) launches mobile app through which students can

Current Affairs [PDF] -February 1-15, 2019

© 2019 GKToday | All Rights Reserved | https://www.gktoday.in 38

Based on threat perception and security requirements, the fencing/ floodlighting has been erectedalong India-Pakistan and India Bangladesh borders.The constructions of border roads are expedited keeping in view the administrative and operationalrequirements of the Border guarding forces.

Even then due to hostile, mountainous terrain and porous nature of the border incidents of theinfiltrations are still reported.

Measures for the protection of Sentinelese TribeThe Sentinelese Tribe are indigenous people inhabiting North Sentinel Island in the Bay of Bengal in Indiaand are considered one of the world’s last uncontacted peoples. They are considered as ParticularlyVulnerable Tribal Group (PVTG).Laws to Safeguard the interests of Sentinelese TribeLaws promulgated for the protection of interests of Sentinelese Tribe are:

A &N Islands (PAT) Regulation 1956.Scheduled Castes and the Scheduled Tribes (Prevention of Atrocities) Act, 1989.Restrictions under Foreigner (Restricted Area) Orders, 1963.Visa Manual Conditions/Passport Act 1920.Indian Forest Act, 1927.Wildlife (Protection) Act, 1972.

Other Steps to safeguard their interestsThe major steps taken towards protection of their interests include:

The entire North Sentinel Island along with 5 km coastal sea from high water mark is notified as atribal reserve.The Sentinelese are still in isolation practising primordial hunting and gathering way of life. TheGovernment has adopted an ‘eyes-on and hands-off’ practice to protect and safeguard theSentinelese tribe.A protocol for the circumnavigation of the North Sentinel Island has been notified. The ships andaircraft of Coast Guard and boats of Marine Police make sorties around North Sentinel to keepsurveillance.The coastal sea up to a fixed extent of 1 Km to 5 Km abutting the tribal territory has also beennotified as a tribal reserve so that marine resources like fish, turtle etc are available exclusively forthe Particularly Vulnerable Tribal Groups (PVTGs).

The Sentinelese tribe is at the verge of extinction. The 2011 census estimates their numbers at a mere 50.RBI penalises UCO Bank for Non-Compliance

The Reserve Bank of India (RBI) has imposed Rs 2 crore penalty on UCO Bank for non-compliance ofinstructions issued by RBI on the collection of account payee instruments and those on frauds-classification and reporting.RBI directions on Fraud Classification and ReportingThe Reserve Bank of India (RBI) advises banks from time to time about the major fraud-prone areas andthe safeguards necessary for the prevention of frauds. RBI also forwards the details of frauds of aningenious nature not reported earlier so that banks could introduce necessary safeguards by way ofappropriate procedures and internal checks. It is continuous process banks report to the Reserve Bank fullinformation about frauds and the follow-up action taken thereon.Since the delays in reporting of frauds and the consequent delay in alerting other banks about the modusoperandi and issue of caution advice against unscrupulous borrowers could result in similar frauds beingperpetrated elsewhere, Banks are strictly required to adhere to the timeframe fixed in this circular forreporting fraud cases to RBI failing which banks would be liable for penal action as prescribed underSection 47(A) of the Banking Regulation Act, 1949.To ensure uniformity in reporting, frauds have been classified as under, based on the provisions of theIndian Penal Code:

Misappropriation and criminal breach of trust.Fraudulent encashment through forged instruments, manipulation of books of account or through

sri vishnu charan | [email protected] |

https://t.me/PDF4Examshttps://t.me/IAS201819 https://t.me/PDF4Exams

https://t.me/TheHindu_Zone_official

Page 39: Current Affairs -February 1-15, 2019 · 15-02-2019  · 10, theme: ‘Sadak Suraksha-Jeevan Raksha’ National Testing Agency (NTA) launches mobile app through which students can

Current Affairs [PDF] -February 1-15, 2019

© 2019 GKToday | All Rights Reserved | https://www.gktoday.in 39

fictitious accounts and conversion of property.Unauthorised credit facilities extended for reward or for illegal gratification.Negligence and cash shortages.Cheating and forgery.Irregularities in foreign exchange transactions.Any other type of fraud not coming under the specific heads as above.

The UCO Bank which is under the prompt corrective action (PCA) framework of RBI has reported a netloss of Rs 633.88 crore for the first quarter of this financial year.

SBI UK chief honoured ‘Freedom of the City of London’The State Bank of India’s UK head Sanjiv Chadha has been honoured with the ‘Freedom of the City ofLondon’ award in recognition of his contribution to promoting the bilateral relations. Sanjiv Chadha wasnominated for the honour by Lord Mayor of London Peter Estlin and Sheriff Vincent Keaveny.Sanjiv Chadha and His contributionsSanjiv Chadha took up the position at the helm of the 98-year-old London office of the State Bank of Indiain 2014. He was instrumental in expanding the bank’s presence across the UK, and in boosting widerbilateral ties in financial services.Sanjiv Chadha was an excellent voice leading the Indian banking community and overseeing significantinvestment into the City.Freedom of the CityFreedom of the City is an honour bestowed by a City of London Corporation upon a valued member of thecommunity, or upon a visiting celebrity or dignitary.The genesis of the ‘Freedom of the City of London’ is traced to the medieval time of around 1237. Thehonour enabled recipients to carry out their trade. The previous Indian recipient of the honour isJawaharlal Nehru, the first Prime Minister of India.

Nikkei India Services Business Activity Index: Key FactsThe seasonally adjusted Nikkei India Services Business Activity Index report makes the followingobservations about the Services Business Activity in India:

The index witnessed a drop for the second straight month in January to 52.2 from 53.2 in December.The fall indicates a softer expansion in output.The growth was supported by favourable public policies, enhanced capacities and greater demand.The upturn was curbed by competitive pressures and election uncertainty.The report notes that a sharp and accelerated rise in manufacturing production counteracted theslowdown in activity growth across the service economy.The report notes that Business activity growth in the Indian service sector cooled further at thestart of 2019, amid the weakest upturn in new work since last September.Optimism regarding the outlook was sustained and the companies have continued to hire. The jobcreation was at a three-month highEven though the inflation rates remained mild by historical standards, the costs rose to a greaterextent than in December.There was a moderate increase in sales that was the weakest in four months. The slowdown wascentred on the domestic market as new orders from abroad grew to the greatest extent since lastSeptember.

Expansion rates in the Indian service sector have been at similarly modest levels for the past four monthsand the data for January extends the trend. There is some sign that growth may run out of steam, in theshort-term at least, as seen by the weakest improvement in demand for four months and relativelysubdued optimism.

Asia LPG SummitThe second edition of the Asia LPG Summit was inaugurated by Union Minister for Petroleum & NaturalGas at New Delhi. The two-day summit with the theme ‘LPG – Energy for Life’ will deliberate over seriesof subjects including how India’s successful Pradhan Mantri Ujjwala Yojana can become a viable model forother countries to replicate particularly those striving to provide clean cooking fuel to its people.

sri vishnu charan | [email protected] |

https://t.me/PDF4Examshttps://t.me/IAS201819 https://t.me/PDF4Exams

https://t.me/TheHindu_Zone_official

Page 40: Current Affairs -February 1-15, 2019 · 15-02-2019  · 10, theme: ‘Sadak Suraksha-Jeevan Raksha’ National Testing Agency (NTA) launches mobile app through which students can

Current Affairs [PDF] -February 1-15, 2019

© 2019 GKToday | All Rights Reserved | https://www.gktoday.in 40

The summit has been organized jointly by the major Indian oil marketing companies (OMCs) Indian Oil,Hindustan Petroleum and Bharat Petroleum together with the World LPG Association (WLPGA).World LPG AssociationWorld LPG Association (WLPGA) is the authoritative voice of the global LPG industry representing the fullLPG value chain. WLPGA was established in 1987 and granted Special Consultative Status with the UnitedNations Economic and Social Council in 1989.Mission of WLPGA

Demonstrate the benefits of LPG and inform, educate and influence all stakeholders.Support the development of LPG markets.Promote compliance with standards, good business and good safety practices.Identify innovation and facilitate knowledge transfer.

The WLPGA brings together over 250 private and public companies operating in more than 125countries involved in one, several or all activities of the industry. The Association aims to add value to thesector by driving premium demand for LPG, while also promoting compliance with good business andsafety practices.WLPGA has developed long-standing partnerships with other international bodies such as variousdepartments of the UN, the World Health Organization (WHO), Regional Development Banks and anumber of well-respected international NGOs such as the Partnership on Clean Indoor Air (PCIA) andE+Co.

Thailand names Siamese Fighting Fish its National Aquatic AnimalThe government of Thailand has approved the proposal to name Siamese fighting fish as the NationalAquatic Animal. The decision for based on the recommendation of the National Identity Committee ofThailand which promotes Thai cultural pride forwarded its endorsement of the fighting fish.The government has approved the proposal owing to the cultural and historical significance of theSiamese fighting fish for Thailand. The Siamese fighting fish was chosen as it’s a native, unique species tothe kingdom’s waters and an important animal for Thailand’s economy.Siamese Fighting FishThe Siamese fighting fish commonly known as the betta is a popular fish in the aquarium trade. TheSiamese fighting fish is native to the Mekong basin of Thailand, Laos, Cambodia, Vietnam and mostlyfound at Chao Phraya river in Thailand.The Siamese fighting fish was registered as intangible cultural heritage by the Cultural Ministry in 2013.The designation of Siamese fighting fish as Thailand’s national aquatic animal could help boost bothconservation efforts and commercial breeding.The IUCN status of the Siamese fighting fish is Vulnerable. The threats to the Siamese Fighting Fishinclude Human intrusions & disturbances, Natural system modifications through the construction ofDams Presence of Invasive and other problematic species, genes & diseases, Pollution due to domestic &urban wastewater, Industrial & military effluents and Agricultural & forestry effluents.

Venezuela CrisisThe Venezuela crisis is deepening with passing time. European Union, Australia, US, New Zealand haverecognised Juan Guaidó, the opposition leader as the president.Juan Guaidó, the opposition leader, has said that President Nicolás Maduro is illegitimate and has assertedhimself as Venezuela’s interim president.What’s the crisis?Both Juan Guaidó and Nicolás Maduro have debatable claims to legitimacy. As a result, the whole issue hasbecome messy.Venezuela has been grappling with crisis spiral for years with growing political discontent further fuelledby skyrocketing hyperinflation, power cuts and shortages of food and medicine. The recent crisis hauntingVenezuela is Who is the President?How did the Presidential Crisis begin?On 23rd January the leader of the legislature, Juan Guaidó, declared himself acting president and said hewould assume the powers of the executive branch from there onwards. This was a challenge to President

sri vishnu charan | [email protected] |

https://t.me/PDF4Examshttps://t.me/IAS201819 https://t.me/PDF4Exams

https://t.me/TheHindu_Zone_official

Page 41: Current Affairs -February 1-15, 2019 · 15-02-2019  · 10, theme: ‘Sadak Suraksha-Jeevan Raksha’ National Testing Agency (NTA) launches mobile app through which students can

Current Affairs [PDF] -February 1-15, 2019

© 2019 GKToday | All Rights Reserved | https://www.gktoday.in 41

Nicolás Maduro, who had been sworn into a second six-year term in office just two weeks previously.President Maduro condemned this as a ploy by the US to oust him.Nicolás Maduro was first elected as President with a thin margin of 1.6 percentage votes in April 2013after the death of his predecessor, Hugo Chávez. Nicolás Maduro was re-elected to the office of presidentfor a second six-year term in highly controversial elections in May 2018, which most opposition partiesboycotted.After being re-elected Nicolás Maduro announced that announced he would serve out his remaining firstterm and only then be sworn in for a second term.The National Assembly leader Juan Guaidó argues that because the election was not fair. Articles 233 and333 of Venezuela’s constitution empower the head of the National Assembly takes over as acting presidentunder such circumstances. Juan Guaidó has staked the claim to be the acting president, as he was the headof the legislature.

Elimination of Tuberculosis from IndiaIndia accounts for about a quarter of the global TB burden. India has set the target to eliminateTuberculosis in India by 2025. To evolve a strategy in this regard, India has put in place National StrategicPlan 2017 – 2025 which sets out the government plans of how the elimination of TB can be achieved.

What does the Elimination of a disease stand for?Elimination of a disease is defined by the World Health Organisation (WHO) as there should be less than 1 case ofTB for a population of a million people.

Step Forwards in 2018As a step towards eliminating TB, India has made following progress in 2018:

In 2018, 5, 36,752 TB patients have been notified from the private sector. There has been a 40%increase in TB notification from the private sector as compared to 2017.44,517 (8%) TB patients were given anti-TB drugs from the programme.As per State reports till 25.01.2019, 40% notified TB patients, 35% treatment supporters and 8%private practitioners have been paid incentives through DBT.

Elimination of TuberculosisThe government has envisioned following strategic steps to eliminate TB by 2025:

A partnership with the private sector has been envisaged under Joint Effort for Elimination ofTuberculosis (JEET) for the elimination of TB by 2025, five years ahead of the global schedule.To increase the reporting from the private sector, public-private support agency approach has beeninitiated.Indian Medial Association has been roped in for large scale sensitization and advocacy with privatepractitioners with standardised digital material.States have been supplied anti-TB drugs to provide for TB patients notified from private sectorbased on demand.achieveStates have been guided to keep programme provided anti-TB drugs at private practitioner’s clinicor pharmacyTo ensure the TB patients are not deprived of DBT benefit flexibility to provide the benefit throughthe existing bank account of a blood relative has been given.States have also been advised to facilitate opening of zero balance accounts for TB patients, ifnecessary, under the PradhanMantri Jan DhanYojana (PMJDY) and Indian Postal Bank.

The Minister of State for Health and Family Welfare, Shri Ashwini Kumar Choubey has stated inparliament that government is committed to achieve the target of TB elimination by 2025 and notconsidering any revision in targets.

Combating HIVIndia has made significant progress in the fight against HIV/AIDS in recent years. The following stepshave been initiated by the government to strengthen the fight against HIV:

A 360-degree multimedia campaign along with mass media supported by outdoor media such ashoardings, bus panels, information kiosks, folk performances and exhibition vans to create

sri vishnu charan | [email protected] |

https://t.me/PDF4Examshttps://t.me/IAS201819 https://t.me/PDF4Exams

https://t.me/TheHindu_Zone_official

Page 42: Current Affairs -February 1-15, 2019 · 15-02-2019  · 10, theme: ‘Sadak Suraksha-Jeevan Raksha’ National Testing Agency (NTA) launches mobile app through which students can

Current Affairs [PDF] -February 1-15, 2019

© 2019 GKToday | All Rights Reserved | https://www.gktoday.in 42

awareness on HIV/AIDS has been initiated.Training and sensitization programmes for Self-Help Groups, Anganwadi workers, ASHA, membersof Panchayati Raj Institutions and other key stakeholders are also being organised to spreadawareness about treatment and other facilities.Awareness activities are being conducted amongst High-Risk Groups including Female SexWorkers, Men having Sex with Men, Injecting Drug Users, Hijra/ Transgenders and BridgePopulation like truck drivers, migrants etc. as part of Targeted Intervention projects.HIV & AIDS (Prevention and Control) Act, 2017 has been notified to address the challenges related tothe disease in a comprehensive way.The act addresses various issues like discrimination against People Living with HIV at theworkplace, an education setting, health setting and public places and also provides for a robustgrievance redressal mechanism where compliant against discrimination would be disposed byOmbudsman at the state level.Steps have been taken by the government to track the patients through HIV diagnosis from care totreatmentPatients diagnosed with HIV are put on Anti Retro Viral Treatment irrespective of their CD4 Count.Mission Sampark has been launched to reach out to all those who are Left to Follow Up and are to bebrought under Antiretroviral therapy (ART) services.Counsellors are tasked to ensure tracking of HIV positive patients through telephone and homevisits in the case of those who drop out between diagnosis and treatment to maintain theiradherence to treatment.

The Union Ministry of Health and Family Welfare launched the National Strategic Plan 2017-24 aimed ateradicating HIV/AIDS by 2030.

Implementation of PMJAYPradhan Mantri – Jan Arogya Yojana (PM JAY) is a scheme of the government under Ayushman Bharat toreduce the financial burden on poor and vulnerable groups arising out of catastrophic hospital episodesand ensure their access to quality health services.Implementation of PM JAYThe following steps have been undertaken for the effective implementation of the PM JAY:

The State Governments have been given the flexibility to implement PM JAY either throughinsurance companies, or directly through trust/society, or in a mixed mode. National Health Agency (NHA) has been set up by the Government to implement PM JAY throughState level Health Agencies.A robust IT system has been put in place for effective implementation of the Scheme.Central Grievance Redressal Management System has been set up for receiving grievances. Asystem has been put in place to acknowledge, record, escalate & resolve grievances as per well-defined process through a three-tier grievance redressal structure.A multi-prong approach has been adopted by putting in place a fraud control mechanism.

PM-JAY scheme would provide poor, deprived rural families and identified occupational categories ofurban workers’ families as per the latest Socio-Economic Caste Census (SECC) data (approx. 50 crorebeneficiaries) with health benefit cover of Rs Five Lakhs per family per year at free of cost. The healthbenefits under the scheme include more than 1,350 medical packages covering surgery, medical anddaycare treatments, cost of medicines and diagnostics.

Know India ProgrammeKnow India Programme (KIP) is a flagship initiative for Diaspora engagement. The programme aims tofamiliarize Indian-origin youth (18-30 years) with their Indian roots and contemporary India.Features of the ProgrammeThe important features of the Know India Programme are:

Know India Programme is a three-week orientation programme for diaspora youth.The programme aims to promote awareness on different facets of life in India and the progressmade by the country in various fields e.g. economic, industrial, education, science & technology,

sri vishnu charan | [email protected] |

https://t.me/PDF4Examshttps://t.me/IAS201819 https://t.me/PDF4Exams

https://t.me/TheHindu_Zone_official

Page 43: Current Affairs -February 1-15, 2019 · 15-02-2019  · 10, theme: ‘Sadak Suraksha-Jeevan Raksha’ National Testing Agency (NTA) launches mobile app through which students can

Current Affairs [PDF] -February 1-15, 2019

© 2019 GKToday | All Rights Reserved | https://www.gktoday.in 43

communication & information Technology and culture.The programme provides a unique forum for students & young professionals of Indian origin to visitIndia, share their views, expectations & experiences and to develop closer bonds with contemporaryIndia.The programme has been in existence since 2003 and is open to PIO youth from all over the world.The programme is open to youth of Indian origin (excluding non-resident Indians) with preferenceto those from Girmitiya countries (Mauritius, Fiji, Suriname, Guyana, T&T, Jamaica etc).The programme mandates a minimum qualification of graduation or being enrolled for graduationto be part of the scheme.

Familiarising IndiaThe programme incorporates following aspects to familiarise Youths about Indian Society and culture:

Understanding of India’s political system, economy, society, and developments in various sectorsetc.Visiting places of historical, cultural, religious importance.Familiarisation with art, music and culture of India.Visiting industrial sites.Visiting a village.Interaction with non-profit organizations.Meetings with senior leadership/officials in India.Visiting a select state in India for 10 days.

The Ministry of External Affairs implements the programme in association with the chosen partner state.Categorisation of Farmers

In agriculture Census, the operational holdings are categorised in five size classes as shown below:Marginal: Below 1.00 hectare.Small: 1.00-2.00 hectare.Semi- Medium: 2.00-4.00 hectare.Medium: 4.00-10.00 hectare.Large: 10.00 hectare and above.

The India Rural Development Report 2012-13 prepared by the IDFC Rural Development Network notesthat Small farms are more efficient, especially in cultivating labour-intensive crops or tending livestock,but land holdings are too small to generate sufficient household income.To improve the condition of Small and Marginal farmers and to double the income of farmers by 2022,Government is realigning its interventions from the production-centric approach to farmers’ income-centric initiatives, with focus on better and new technological solutions.The interventions include implementation of schemes like, Pradhan Mantri Krishi Sinchai Yojana(PMKSY), Paramparagat Krishi Vikas Yojana (PKVY), Soil Health Card, Neem Coated Urea, Rainfed AreaDevelopment under National Mission for Sustainable Agriculture (NMSA), Pradhan Mantri Fasal BimaYojana (PMFBY), National Agriculture Market scheme (e-NAM), National Food Security Mission (NFSM),National Mission on Oilseeds & Oil palm (NMOOP), Mission for Integrated Development of Horticulture(MIDH), Rashtriya Krishi Vikas Yojana (RKVY), National Mission on Agriculture Extension & Technology(NMAET).

February 7, 2019Delhi to Launch Zero Fatality Corridor

Delhi Government has launched Zero Fatality Corridor with an aim to curb the number of accidents. Theannouncement was made by the Minister of Transport, Delhi during the opening ceremony of the RoadSafety Week.About the Zero Fatality CorridorThe important facts related to the Zero Fatality Corridor are:

A 3km stretch between Bhalswa chowk and Burari chowk on the outer ring road has been chosen asa case study in this regard.The selected stretch has four blackspots namely Burari chowk, Bhalswa chowk, Mukundpur chowk,

sri vishnu charan | [email protected] |

https://t.me/PDF4Examshttps://t.me/IAS201819 https://t.me/PDF4Exams

https://t.me/TheHindu_Zone_official

Page 44: Current Affairs -February 1-15, 2019 · 15-02-2019  · 10, theme: ‘Sadak Suraksha-Jeevan Raksha’ National Testing Agency (NTA) launches mobile app through which students can

Current Affairs [PDF] -February 1-15, 2019

© 2019 GKToday | All Rights Reserved | https://www.gktoday.in 44

and Jahangirpuri bus stand.This 3km stretch will be observed for scientific assessment of accidents, road engineering, road-userengagement.Experts will also ensure that safety and security measures such as police enforcement and rapidemergency care are made available on this stretch.The initiative is a collaborative approach of the transport, health, education, public worksdepartment, together with Delhi traffic police to ensure that road fatalities in Delhi are reduced.

Data shows that around 67 fatal road accidents occurred on this road over a span of two years starting2016. The Zero Fatality Corridor project aim is to reduce the fatalities to near zero levels on this particularstretch. After evaluating the impact of the initiative, the government aims to replicate this model in otherparts of the city. The initiative is part of the plan of action to reduce fatalities due to road crashes by 30per cent by 2020.

India becomes Second Largest LPG Consumer and ImporterIndia is now the second largest consumer and importer of the LPG in the world. This is attributed togovernment initiatives to push for cleaner alternatives to traditional cooking fuels such as firewood andcow dung.Import of LPGThe import of LPG grew by 12.5 per cent over the past five years to 12 million metric tons (13 million tons)in 2018-19. India now stands at the second position when it comes to the import of LPG surpassing Japan.India is at second place behind China. The exporters of LPG to India include Saudi Arabia, Qatar, theUnited Arab Emirates, Kuwait and Iran.Consumption of LPGThe number of active LPG consumers in India has grown at a compounded annual growth rate (CAGR) of15 per cent – from 14.8 crore in 2014-15 to 22.4 crore in 2017-18.Increase in population combined with LPG penetration in rural areas has resulted in an average growth of8.4 per cent in LPG consumption, making India the second largest consumer of LPG in the world at 22.5million tonnes.The Ministry of Petroleum and Natural Gas estimates that LPG consumption would grow to 30.3 milliontonnes by 2025 and 40.6 million tonnes by 2040.The flagship scheme of the government Pradhan Mantri Ujjwala Yojana (PMUY) has made a significantcontribution towards enhancing the penetration of LPG, especially in rural areas. The renewed targetsunder the Pradhan Mantri Ujjwala Yojana (PMUY) aims to provide LPG connections to 8 crorehouseholds before March 31, 2020.

Strengthening the fight against Mental Illness in IndiaThe Mental illness is the most neglected and tabooed aspect of Indian health care system. To address theburden of mental disorders, the Government of India has undertaken the following steps:

National Mental Health Programme (NMHP) is under implementation since 1982. District MentalHealth Programme (DMHP) is under implementation in 517 districts of the country for detection,management and treatment of mental disorders/ illness.Funds are provided to each district under the DMHP for sensitization training of Community HealthWorkers and elected representatives of the community at District Hospital/ Community HealthCentre for awareness generation regarding early signs and community health-seeking behaviour formental illness.The Mental Healthcare Act, 2017 provides that the appropriate Government shall take all measuresto ensure that the Government Officials including police officers and other officers of theGovernment are given periodic sensitization and awareness training.Three Central Mental Health Institutions namely National Institute of Mental Health and NeuroSciences, Bangalore, Lokopriya Gopinath Bordoloi Regional Institute of Mental Health, Tezpur andCentral Institute of Psychiatry, Ranchi have been strengthened for augmenting the humanresources in the areas of mental health and to provide quality mental health services in the country.To increase the number of qualified mental health professionals in the country, the Government,

sri vishnu charan | [email protected] |

https://t.me/PDF4Examshttps://t.me/IAS201819 https://t.me/PDF4Exams

https://t.me/TheHindu_Zone_official

Page 45: Current Affairs -February 1-15, 2019 · 15-02-2019  · 10, theme: ‘Sadak Suraksha-Jeevan Raksha’ National Testing Agency (NTA) launches mobile app through which students can

Current Affairs [PDF] -February 1-15, 2019

© 2019 GKToday | All Rights Reserved | https://www.gktoday.in 45

under the NMHP, is implementing manpower development schemes for the establishment ofCentres of Excellence and strengthening/ establishment of Post Graduate (PG) Departments inmental health specialities.

The Ministry of Health and Family Welfare is regularly reviewing and monitoring the functioning of thethree Central Mental Health Institutions and the Institutions supported under manpower developmentschemes of NMHP.

Cinematograph Amendment Bill 2019The Union Cabinet headed by Prime Minister Narendra Modi has approved the CinematographAmendment Bill 2019. The amendment bill aims to amend the Cinematograph Amendment act 1952. Thefeatures of the amendment bill are:

The amendment bill makes film piracy offences punishable with imprisonment up to three years andfines that may extend to 10 lakh or both.The amendment states that any person, who without the written authorisation of the copyrightowner, uses any recording device to make or transmit a copy of a film, or attempts to do so, or abetthe making or transmission of such a copy, will be liable for such a punishment.Section 7 of the Cinematograph Act, 1952 deals with who can watch and exhibit which films andpenalties for violating terms and conditions related to the exhibition of board-certified films.The amendment bill adds a new subsection (4) to section 7 of the Cinematograph Act, 1952 with thedefinition of piracy and the penal provisions for the same.

The Cinematograph Amendment Bill, 2019 aims to tackle film piracy by including the penal provisions forunauthorised camcording and duplication of films. The bill when passed will build a credible deterrencewhich would increase industry revenues, boost job creation, fulfil important objectives of India’s NationalIntellectual Property policy and will give relief against piracy and infringing content online.

Parmanu Tech 2019The Parmanu Tech conference was organised by the Ministry of External Affairs and Department ofAtomic Energy (DAE). The keynote address at the ‘Parmanu Tech 2019’ conference was delivered by DrJitendra Singh, Minister of State for Atomic Energy and Space.Highlights of the SpeechThe Highlights of the Speech are:

India has covered a long journey since the inception of Atomic Energy programme by Dr HomiBhabha based on the peaceful use of nuclear energy.India has achieved a unique place in the international community for progress in space technologyas well as nuclear energy.India has always used technology for constructive use and never for destructive purpose.Most of the societal applications of nuclear energy are not much known to the people.Focused efforts are required to create awareness about the benefits of nuclear energy and to allaythe fears about radiation and its harms.Nuclear energy will be a big and cost-effective source of energy in future when the other sources ofpower are gradually depleting.

Sessions in Parmanu Tech 2019During the conference sector-specific sessions were held on the following topics:

Health care: Nuclear Medicine and Radiation therapy – Care to cure.Food Preservation, Agricultural & Industrial Applications: From farms to factories – Serving theNational Cause.Showcasing India’s capabilities in Nuclear Energy: Energy Security with environmentalresponsibility – the Onward March.

The Parmanu Tech conference is aimed at showcasing the societal applications that the Department ofAtomic Energy is pursuing.

Amendments to the Banning of Unregulated Deposit Schemes Bill, 2018The Union Cabinet headed by Prime Minister has approved the amendments to the Banning ofUnregulated Deposit Schemes Bill, 2018. The amendments introduced based on the recommendations of

sri vishnu charan | [email protected] |

https://t.me/PDF4Examshttps://t.me/IAS201819 https://t.me/PDF4Exams

https://t.me/TheHindu_Zone_official

Page 46: Current Affairs -February 1-15, 2019 · 15-02-2019  · 10, theme: ‘Sadak Suraksha-Jeevan Raksha’ National Testing Agency (NTA) launches mobile app through which students can

Current Affairs [PDF] -February 1-15, 2019

© 2019 GKToday | All Rights Reserved | https://www.gktoday.in 46

the Standing Committee on Finance strengthen its objective to effectively tackle the menace of illicitdeposit-taking activities in the country, and prevent such schemes from duping poor and gullible people oftheir hard earned savings.Features of the BillThe salient features of the bill are:

The Bill bans Deposit Takers from promoting, operating, issuing advertisements or acceptingdeposits in any Unregulated Deposit Scheme. The Bill ban unregulated deposit-taking activitiesaltogether, by making them an offence. The existing legislative-cum-regulatory framework whichonly comes into effect ex-post with considerable time lags;The Bill creates three different types of offences, namely, running of Unregulated Deposit Schemes,fraudulent default in Regulated Deposit Schemes, and wrongful inducement in relation toUnregulated Deposit Schemes.The Bill provides for severe punishment and heavy pecuniary fines to act as a deterrent.The Bill provides for repayment of deposits in cases where such schemes nonetheless manage toraise deposits illegally.The Bill provides for attachment of properties/assets by the Competent Authority, and subsequentrealization of assets for repayment to depositors.Timelines have been provided for attachment of property and restitution to depositors.The Bill enables the creation of an online central database, for collection and sharing of informationon deposit-taking activities in the country;The Bill also defines “Deposit Taker” and “Deposit” comprehensively.

Definition of “Deposit Taker” and “Deposit” under the BillDeposit Takers include all possible entities (including individuals) receiving or soliciting deposits, except specificentities such as those incorporated by legislation;Deposit is defined in such a manner that deposit-takers are restricted from camouflaging public deposits as receipts,and at the same time, not to curb or hinder acceptance of money by an establishment in the ordinary course of itsbusiness.

The Bill adopts best practices from State laws and entrusts the primary responsibility of implementingthe provisions of the legislation to the State Governments.

Cabinet approves abolition of Ombudsman for Direct and Indirect taxesThe Union Cabinet headed by Prime Minister Narendra Modi has approved the abolition of institutions ofOmbudsman for income tax and indirect tax.The office of Income-Tax Ombudsman was set up in 2003 to deal with grievances of public related to thesettlement of complaints relating to income tax.Why did the government decide to abolish the office of Ombudsman?The following are cited as reasons for the abolition of the Ombudsman:

People are preferring online grievance redressal mechanism.The institution failed to achieve its objectives as the number of new complaints have fallen to singledigitsThe institution of Ombudsman could not prove to be more effective than regular existing parallelchannels of grievance redressal.Other mechanisms like Centralised Public Grievance Redress and Monitoring System and AaykarSeva Kendras are being preferred by the people.

Centralised Public Grievance Redress and Monitoring SystemCentralized Public Grievance Redress and Monitoring System (CPGRAMS) is an online web-enabledsystem developed by NIC. The platform based on web technology aims to enable submission of grievancesby the aggrieved citizens from anywhere and anytime (24×7).On the basis of the grievances received Ministries/Departments/Organisations who scrutinize and takeaction for speedy and favourable redress of these grievances. CPGRAMS also facilitate tracking ofgrievances through the system generated unique registration number.

sri vishnu charan | [email protected] |

https://t.me/PDF4Examshttps://t.me/IAS201819 https://t.me/PDF4Exams

https://t.me/TheHindu_Zone_official

Page 47: Current Affairs -February 1-15, 2019 · 15-02-2019  · 10, theme: ‘Sadak Suraksha-Jeevan Raksha’ National Testing Agency (NTA) launches mobile app through which students can

Current Affairs [PDF] -February 1-15, 2019

© 2019 GKToday | All Rights Reserved | https://www.gktoday.in 47

Aaykar Seva KendrasAaykar Seva Kendras are established as part of I-T department’s ‘Sevottam’ scheme which seeks to changethe Income-tax department’s perception from a purely enforcement agency to a service provider.The Aaykar Seva Kendras help taxpayers file their returns and redress their grievances. They are meant tobe a single window solution for all tax services.

RBI lowers Repo RateThe six-member monetary policy committee (MPC) headed by RBI Governor Shaktikanta Das has loweredthe repo rate by 25 basis points to 6.25 per cent in a 4-2 vote.Important decisions by the MPCThe features of the decisions taken by MPC are:

Together with lowering the repo rate by 25 basis points to 6.25 per cent, the policy stance has beenchanged to neutral from calibrated tightening.The shift in stance to neutral provides flexibility to meet growth challenges.The change in stance also signals higher chances of more cuts in the coming months if inflationpersisted within tolerable limits.A large part of the current investment recovery has been driven by government spending and it wasnecessary to broad base the revival with a private sector boost.

Other announcements made by RBIThe limit of collateral free bank loans for farmers to Rs 1.6 lakh from Rs 1 lakhGreater operational freedom for Banks to offer interest rates to bulk deposits.The definition of “bulk deposits” has been increased to Rs 2 crore from Rs 1 crore currently.The headline inflation is likely to persist within the RBI’s tolerable level of 4 per cent.

While the decision to change the monetary policy stance was unanimous, Deputy Governor Viral Acharyaand another MPC member, Chetan Ghate, voted for status quo in interest rates, while Das and threeothers voted for a cut in interest rates.

February 8, 2019Macedonia signs accord to join NATO

Macedonia takes a major step on road to NATO membership. The conflict over the name with Greece hadbecome a damper in its membership to NATO. With the conflict with Greece taken to logical end with theconclusion of Prespa Agreement, Macedonia has signed accession papers with NATO.The signing of accession papers allows Macedonia take part in NATO ministerial meetings as an invitee.To acquire full membership, all 29 current members must ratify the accession protocol.Russia Raises ConcernsRussia has raised concerns against Macedonia becoming part of NATO. Russia has accused NATO ofdestabilising the Balkans by pushing Macedonia and Montenegro to join NATO.Russia sees Balkan nations as its sphere of influence and is against NATO or any other body led by US orEU making inroads to these Balkan countries. NATO’s membership provides a guarantee of mutualdefence, provides a welcome insurance policy against possible incursions. Russia perceives this as anattempt by the west to contain it by making inroads to the areas which Russia considers its sphere ofinfluence.North Atlantic Treaty Organisation (NATO)North Atlantic Treaty Organisation (NATO) is a military and political alliance established through theWashington Treaty. NATO aims to promote democratic values and enables members to consult andcooperate on defence and security-related issues to solve problems, build trust and, in the long run,prevent conflict.Article 5 of the Washington Treaty states that an armed attack against one of the member states wouldbe considered as an attack against all members, and other members would assist the attacked member,with armed forces if necessary.

Linking Of PAN Card with Aadhaar Mandatory for Filing IT returns : SCThe Supreme Court has upheld the section 139AA of the Income Tax Act and said that linking of PAN withAadhaar is mandatory for the filing of Income Tax returns.The Supreme Court made these observations while hearing an appeal filed by the Centre against a Delhi

sri vishnu charan | [email protected] |

https://t.me/PDF4Examshttps://t.me/IAS201819 https://t.me/PDF4Exams

https://t.me/TheHindu_Zone_official

Page 48: Current Affairs -February 1-15, 2019 · 15-02-2019  · 10, theme: ‘Sadak Suraksha-Jeevan Raksha’ National Testing Agency (NTA) launches mobile app through which students can

Current Affairs [PDF] -February 1-15, 2019

© 2019 GKToday | All Rights Reserved | https://www.gktoday.in 48

High Court order allowing two people, Shreya Sen and Jayshree Satpute, to file Income Tax returns for2018-19 without linking their Aadhaar and PAN numbers.Noting that with regard to Assessment Year 2018-19 the two petitioners had filed the Income Tax returnsin terms of the orders of the High Court and the assessment has also been completed, Supreme Court saidthat for the assessment year 2019-20, the income tax return shall be filed in terms of the judgment passedby this court.Section 139AA of the Income-tax ActSection 139AA of the Income-Tax Act 1961 was introduced by the Finance Act, 2017. The section makes itmandatory to quote Aadhaar / Enrolment ID of Aadhaar application form, for the filing of return of incomeas well as in the application form to enrol for PAN.The section also specifies that those already in possession of the PAN numbers must mandatorily linktheir PAN numbers with Aadhaar. Failure to do so would result in PAN number becoming invalid.

Banks Liable for Unauthorised Withdrawals: Kerala High CourtThe Kerala High Court has said that banks cannot be absolved from the liability to the loss caused to acustomer on account of unauthorised withdrawals made from his/her account, merely on the ground thatthe customer did not respond promptly to the SMS alert given by the bank.Observations made by the Kerala High CourtThe following observations were made by the Kerala High Court while hearing the appeal filed by theState Bank of India, challenging an order of a sub-court, Pala, allowing the plea of an NRI for the realisationof ₹2.40 lakh lost through unauthorised withdrawal by fraudsters in Brazil:

A bank has a duty to its customers to take necessary steps to prevent unauthorised withdrawalsfrom their accounts.Hence if a customer suffered a loss on account of transactions not authorised by him, the bank wasliable to the customer for the loss.It is the obligation of banks to create a safe electronic banking environment to combat all forms ofmalicious conduct, which result in the loss to their customers.The court noted that if a customer suffered a loss in connection with transactions made byfraudsters, it had to be presumed that it was on account of the failure of the bank to put in place asystem which prevented such withdrawals, and the bank was, therefore, liable for the loss caused tothe customer.The court held that if there exists a specific term in the contract between a bank and its customer tothe effect that the bank would be exonerated from the liability in connection with any unauthorisedtransaction if the customer did not respond to the SMS alerts, such notifications cannot be the basisfor determining the liability of the customer.

The argument of the SBI was that since the loss caused to the customer was not due to any action orinaction on the part of the bank, even if the withdrawals were made fraudulently by third-parties withoutthe knowledge of the plaintiff, the bank was not liable for the same and the customer should have set thecriminal law in motion in the foreign country for redressal of his grievance.

International Day of Zero Tolerance for Female Genital MutilationThe International Day of Zero Tolerance for Female Genital Mutilation is observed on February 6 everyyear. It is an annual awareness day celebrated as part of the UN’s efforts to eradicate female genitalmutilation. It was first introduced in 2003.Why February 6 was Chosen?It was on February 6, 2003, Stella Obasanjo, the First Lady of Nigeria and spokesperson for the CampaignAgainst Female Genital Mutilation, made the official declaration on “Zero Tolerance to FGM” in Africaduring a conference organized by the Inter-African Committee on Traditional Practices Affecting theHealth of Women and Children (IAC). Then the UN Sub-Commission on Human Rights adopted this day asan international awareness day.Female Genital MutilationFemale Genital Mutilation refers to procedures involving partial or total removal of the female externalgenitalia or other injuries to the female genital organs for non-medical reasons. The practice is widely

sri vishnu charan | [email protected] |

https://t.me/PDF4Examshttps://t.me/IAS201819 https://t.me/PDF4Exams

https://t.me/TheHindu_Zone_official

Page 49: Current Affairs -February 1-15, 2019 · 15-02-2019  · 10, theme: ‘Sadak Suraksha-Jeevan Raksha’ National Testing Agency (NTA) launches mobile app through which students can

Current Affairs [PDF] -February 1-15, 2019

© 2019 GKToday | All Rights Reserved | https://www.gktoday.in 49

carried out on young girls between infancy and the age of 15 due to cultural, religious or social reasons.Spotlight Initiative which is a joint project of the European Union and the United Nations aims toeliminate all forms of violence against women and girls, specifically targets sexual and gender-basedviolence, and harmful practices in Sub-Saharan Africa, which include female genital mutilation.Female genital mutilation is a gross violation of the human rights of women and girls. The practicediscriminates against women on the basis of sex and compromises the rights to health, physical integrityand life, the right to freedom from torture or cruel, inhuman or degrading treatment or punishment; andthe rights of the child.Hence to abolish this inhumane discriminatory practice, Sustainable Development Goals in 2015 calls foran end to FGM by 2030 under Goal 5 on Gender Equality.

Father Franois Laborde conferred with Légion d’HonneurFather Franois Laborde a 92-year-old priest from Bengal was conferred with Légion d’Honneur (Legion ofHonour) in recognition of his work for specially-abled and destitute children. Father Laborde has his rootsin France and is the citizen of India now.Father Laborde has set up an organisation called Howrah South Point which works for the development ofspecially-abled children, the destitute and the most deprived sections of the society.Father Franois Laborde is the third person from the state of West Bengal to have been awarded the Legionof Honor. The honour has already been bestowed upon film maestro Satyajit Ray and actor SoumitraChatterjee. Both of them also hails from the state of West Bengal.Légion d’Honneur (Legion of Honour)Légion d’Honneur (Legion of Honour) officially National Order of the Legion of Honour was instituted byNapoleon Bonaparte in 1802 as a general military and civil order of merit conferred without regard tobirth or religion provided that anyone admitted swears to uphold liberty and equality.The honour has been retained by all later French governments and regimes. It is the highest civilianaward given by the French government for outstanding service and excellence in any field regardless ofthe nationality of the recipients.

Part Two of the Darwaza Band Campaign LaunchedThe part two of Darwaza Band campaign which aims to promote the use of toilets and to sustain the opendefecation free status of villages across India has been launched.The campaign has been launched under the aegis of Swachh Bharat Mission Grameen initiative ofMinistry of Drinking Water and Sanitation.Darwaja Band literally stands for shutting the door. The campaign symbolically stands for shutting thedoor for open defecation. The campaign aims to encourage behaviour change in men who have toilets butare not using them and to encourage women to stand up for this issue in their villages and assume aleadership role.The Darwaza Band campaign is designed to encourage behaviour change through television ads, radiojingles, outdoor publicity and digital campaigns. The campaign is launched in association with WorldBank.Making India open Defecation FreeThe data from government suggests that over 50 crore people have stopped defecating in the open sincethe launch of the Swachh Bharat Mission and over 5.5 lakh villages have been declared Open DefecationFree. The national sanitation coverage is now in excess of 98 per cent as compared to 39 per cent in 2014.

GSAT-31 launched from French GuianaIndia’s communication satellite GSAT-31 was successfully launched from the Spaceport in French Guiana.GSAT-31 was launched from the launch vehicle Ariane 5 VA-247.Features of the GSAT-31The important features of the GSAT-31 are:

GSAT-31 would be placed at Geosynchronous Transfer Orbit.GSAT-31 will augment the Ku-band transponder capacity in Geostationary Orbit.GSAT-31 derives its heritage from ISRO’s earlier INSAT/GSAT satellite series and will providecontinuity to operational services on some of the in-orbit satellites.

sri vishnu charan | [email protected] |

https://t.me/PDF4Examshttps://t.me/IAS201819 https://t.me/PDF4Exams

https://t.me/TheHindu_Zone_official

Page 50: Current Affairs -February 1-15, 2019 · 15-02-2019  · 10, theme: ‘Sadak Suraksha-Jeevan Raksha’ National Testing Agency (NTA) launches mobile app through which students can

Current Affairs [PDF] -February 1-15, 2019

© 2019 GKToday | All Rights Reserved | https://www.gktoday.in 50

GSAT-31 will provide DTH Television Services, connectivity to VSATs for ATM, Stock-exchange,Digital Satellite News Gathering (DSNG) and e-governance applications.GSAT-31 will also be used for bulk data transfer for a host of emerging telecommunicationapplications.

After separation from the upper stage of Ariane-5, the two solar arrays of GSAT-31 were automaticallydeployed in quick succession and ISRO’s Master Control Facility at Hassan in Karnataka took over thecommand and control of GSAT-31.ISRO scientists will now undertake phase-wise orbit-raising manoeuvres to place the satellite inGeostationary Orbit (36,000 km above the equator) using its onboard propulsion system. The satellitewould be then placed in its final orbital configuration.

Vidarbha claim 2nd successive Ranji TrophyThe defending champions Vidarbha won the 85th Ranji Trophy title. The defending champions defeatedSaurashtra by 78 runs in the final.Score Card of the Ranji Final ClashVidarbha: 312 and 200Saurashtra: 307 and 127Aditya Sarwate of was Vidarbha was declared man of the match for his 11 wickets in the match and 49runs in Vidarbha’s second innings which helped Vidarbha clinching the title for the second time.Ranji TrophyRanji Trophy is the Indian domestic first-class cricket championship between teams representing regionaland state cricket associations. The championship is named after Ranjitsinhji Vibhaji Jadeja, who wasfamously referred to as ‘Ranji’.The cricket championship was conceived in July 1934 and the first fixture was held between Madras andMysore at the Chepauk ground in Madras. The Ranji Trophy was also donated by Ranjitsinhji VibhajiJadeja.Mumbai(Bombay) have won the tournament the most number of times with 41 wins. Vidarbha became thesixth team to successfully defend their title. The other sides to win the consecutive titles areBombay/Mumbai, Maharashtra, Delhi, Rajasthan and Karnataka.

Arundhati Scheme of AssamThe government of Assam has announced a new scheme called Arundhati to provide gold at free of cost tothe brides.Arundhati SchemeThe important features of the scheme are:

Under the scheme, the government of Assam aims to provide 1 Tola Gold, at about Rs 38K as ontoday, to brides belonging to all such communities of Assam where it is customary to provide gold atthe time of the wedding.The government has set aside Rs 300 cr has been for the implementation of the Arundhati Scheme.The scheme is named after Arundhati, wife of great sage Basistha.Arundhati considered an epitome of chastity and conjugal bliss and the government t wants to wishthe bride with joy and happiness of that of Arundhati.The benefit under Arundhati scheme can be availed upon formal registration of marriages underSpecial Marriage (Assam) Rules, 1954 & the gold will reach beneficiaries right in time for the socialmarriage.The scheme is limited for economically weaker sections, whose annual income is below Rs 5 lakh.

Gold forms an inherent part of Indian weddings. With the introduction of the Arundhati Scheme, thegovernment wants to stand with those fathers who cannot afford to gift a set of gold ornaments to theirdaughters and would to resort to borrowings and put themselves in the vicious cycle of debt.

President Trump Nominates David Malpass for the Presidency of World BankThe US President Trump has nominated David Malpass for the post of the president of the World Bank.David MalpassDavid Malpass had held senior roles in the US Treasury during the Reagan and George HW Bush

sri vishnu charan | [email protected] |

https://t.me/PDF4Examshttps://t.me/IAS201819 https://t.me/PDF4Exams

https://t.me/TheHindu_Zone_official

Page 51: Current Affairs -February 1-15, 2019 · 15-02-2019  · 10, theme: ‘Sadak Suraksha-Jeevan Raksha’ National Testing Agency (NTA) launches mobile app through which students can

Current Affairs [PDF] -February 1-15, 2019

© 2019 GKToday | All Rights Reserved | https://www.gktoday.in 51

administrations. He has also served as chief economist at Bear Stearns bank. Later he founded researchgroup Encima Global and held a number of directorships at finance firms.After the assent of Trump to the US Presidency, Mr Malpass took up a senior role at the US Treasury andwas the undersecretary for international affairs.President of World BankThe guidelines of the World Bank require the World Bank president to have:

A proven track record of leadership.Experience managing large organizations with international exposure, and a familiarity with thepublic sector.Ability to articulate a clear vision of the Bank’s development mission.A firm commitment to and appreciation for multilateral cooperation.Effective and diplomatic communication skills, impartiality, and objectivity.

Executive directors of the World Bank can nominate candidates for the post. The candidate must becitizens of one of the bank’s member countries and cannot be a bank governor, executive director oralternate. Executive directors are expected to clearly communicate and define the nomination period.If there are more than three candidates, there will be a shortlisting process to narrow the field through aninformal straw poll. The shortlisted candidates will be interviewed by the board and then the board willmake a final selection through a majority vote.To win the presidency of the World Bank, a candidate must win approval from the institution’s executiveboard, which has 25 members. The US holds a 16% share of board voting power hence more often it is theUS-backed candidate gets elected for the post of the president.

RBI Sixth bi-monthly Monetary Policy Statement 2018-19: Key FactsThe important facts related to the RBI’s sixth bi-monthly monetary policy statement for 2018-19 is listedbelow:

RBI has relaxed the CPI or retail inflation forecasts for India in FY20.Because of the low inflation forecasts, RBI has decided to trim down policy repo rate by 25 basispoints, taking the overall interest rate down to 6.25% now from previous 6.50%.The policy stance has also been changed from calibrated tightening to neutral.RBI data suggests that the 7th Pay Commission’s HRA allowance impact on the inflation indicatorhas diminished.The reverse repo rate under the liquidity adjustment facility now stands adjusted to 6.0 per cent.The marginal standing facility (MSF) rate and the Bank Rate now stand at 6.5 per cent.

Why the Inflation Forecasts were on the lower side?The lower inflation forecasts by the RBI are attributed to the following reasons:

Food inflation has continued to be on the downside with continuing deflation across several itemsand a significant moderation in inflation in cereals. Food commodities are experiencing excesssupply conditions domestically as well as internationally. Hence, the short-term outlook for foodinflation appears particularly benign, despite adverse base effects.The moderation in the fuel prices was larger than anticipated. Inflation in items of ruralconsumption such as firewood and chips, which had remained sticky and at elevated levels, hascollapsed in recent months. Electricity prices have also witnessed an unexpected moderation. Thisresulted in a softer outlook for the fuel groupThe recent unusual pick-up in the prices of health and education is seen as a one-off phenomenon.

The next meeting of the MPC for the policy review is scheduled from April 2 to 4, 2019.US approves sale of two missile defence systems for Air India One

The US President has approved the sale of two missile defence systems for Air India One. The two missiledefence systems Large Aircraft Infrared Countermeasures (LAIRCAM) and Self-Protection Suites (SPS) willenhance the security of planes flying the prime minister and the president.Enhancing the Security of Prime Minister and PresidentThe new defence systems are being acquired given the high-level threat to the prime minister and thepresident. The features of the new systems are:

sri vishnu charan | [email protected] |

https://t.me/PDF4Examshttps://t.me/IAS201819 https://t.me/PDF4Exams

https://t.me/TheHindu_Zone_official

Page 52: Current Affairs -February 1-15, 2019 · 15-02-2019  · 10, theme: ‘Sadak Suraksha-Jeevan Raksha’ National Testing Agency (NTA) launches mobile app through which students can

Current Affairs [PDF] -February 1-15, 2019

© 2019 GKToday | All Rights Reserved | https://www.gktoday.in 52

LAIRCM protect large aircraft from man-portable missiles.LAIRCAM system increases crew-warning time, decreases false alarm rates and automaticallycounters advanced intermediate range missile systems.The missile warning subsystem will use multiple sensors to provide full spatial coverage and thecounter-measures subsystem will use lasers mounted in pointer-tracker turret assemblies.The countermeasure systems will also automatically counters advanced intermediate range missilesystems with no action required by the crew.SPS will facilitate a more robust capability into areas of increased missile threats.

These defence systems would be installed at two Boeing 777 Head-of- State aircraft which carriesPresident and Prime Minister.

International IP Index 2019: Key FactsThe U.S. Chamber Chamber of Commerce’s Global Innovation Policy Center (GIPC) has released theInternational IP Index 2019. The Index aims to create a roadmap for countries that aspire to fostereconomic growth and global competitiveness through stronger IP. The Index benchmarks the IPframework in 50 global economies.The index evaluates the IP infrastructure in each economy based on 45 unique indicators, which arecritical to the growth of effective IP systems. The indicators encompass 8 categories of IP protection:patents, copyrights, trademarks, trade secrets, commercialization of IP assets, enforcement, systemicefficiency, and membership and ratification of international treaties.International IP Index 2019The important features of the International IP Index 2019 are:

The U.S., UK, and EU economies remain atop the global IP rankings, though the U.S.’s lead narroweddue to systemic challenges to the U.S patent system.Japan and Singapore also appear among the Index’s top ten.India, for the first time, broke free of the bottom ten per cent of economies measured.Majority of the economies benchmarked are building more effective foundations for IP policy.The government introduced reforms in Argentina to streamline the patenting process, increaseinternational cooperation on IP, and bolster transparency and stakeholder engagement has resultedin increasing of its overall score by 15% and its rankings from 46th in 2018 to 40th in 2019.

sri vishnu charan | [email protected] |

https://t.me/PDF4Examshttps://t.me/IAS201819 https://t.me/PDF4Exams

https://t.me/TheHindu_Zone_official

Page 53: Current Affairs -February 1-15, 2019 · 15-02-2019  · 10, theme: ‘Sadak Suraksha-Jeevan Raksha’ National Testing Agency (NTA) launches mobile app through which students can

Current Affairs [PDF] -February 1-15, 2019

© 2019 GKToday | All Rights Reserved | https://www.gktoday.in 53

sri vishnu charan | [email protected] |

https://t.me/PDF4Examshttps://t.me/IAS201819 https://t.me/PDF4Exams

https://t.me/TheHindu_Zone_official

Page 54: Current Affairs -February 1-15, 2019 · 15-02-2019  · 10, theme: ‘Sadak Suraksha-Jeevan Raksha’ National Testing Agency (NTA) launches mobile app through which students can

Current Affairs [PDF] -February 1-15, 2019

© 2019 GKToday | All Rights Reserved | https://www.gktoday.in 54

The report concludes that starting from a low global IP standard, progress has been slow andinconsistent. Yet there are obvious green shoots, seen most clearly in the adoption of broadermeasures to improve the systemic efficiency of IP rights administration and the ability of IPowners to leverage their rights to finance innovative and creative activities. Notwithstandingcontinued political threats to undermine IP rights for populist purposes, there is evidence that theworld is becoming an IP believer, inspired by the possibility of a better tomorrow.

International IP Index 2019: Key Facts about IndiaThe International IP Index 2019 released by the U.S. Chamber of Commerce’s Global Innovation PolicyCenter (GIPC) makes the following observations related to India:

India experienced its second consecutive year of growth in the IP global ranking, jumping eightplaces from 44th to 36th.The jump in the ranking is attributed to the recognition of international standards of copyrightprotection and incentives for intellectual property.The report notes that the improvement reflects important reforms implemented by Indianpolicymakers towards building and sustaining an innovation ecosystem for domestic entrepreneursand foreign investors alike.India’s overall score has also increased substantially from 30.07% (12.03 out of 40) in the 2018 indexto 36.04% (16.22 out of 45) in 2019.India’s score represents the largest gain of any country measured on the Index.The weakness of India as highlighted in the report are barriers to licensing and technology transfer,including strict registration norms, limited framework for the protection of biopharmaceutical IPrights, patentability rules outside international standards, lengthy pre-grant opposition proceedingsand previously used compulsory licensing for commercial and non-emergency situations as keyhurdles.

The report notes that the most substantial movement can be seen from India, which has surged almost 20per cent and climbed eight places in the IP Index rankings from 44th to 36th.

Factors which led to India’s stride in International IP Index 2019In the International IP Index 2019 released by U.S. Chamber of Commerce’s Global Innovation PolicyCenter (GIPC), India has surged almost 20 per cent and climbed eight places in the IP Index rankings from44th to 36th. The report cites the following factors as responsible for India’s performance:

India has taken several noteworthy steps to improve its IP system in 2018 and also performed wellon the new indicators included in the Index this year.The Government of India take steps to improve its national IP environment and these effortscontinued in 2018 and progress has been made on several important areas measured by the Index.Greater efforts were made to align and incorporate India’s IP environment with the international IPsystem.India’s accession to the WIPO Internet Treaties and subsequent agreement with the Japanese PatentOffice on a pilot PPH both stand out.India also invested considerable energy into decreasing pendency rates for patent and trademarkapplications by hiring staff and investments in resources for modernising and improving theadministrative capacities of the Office of the Controller General of Patents, Designs and Trademarks(CGPDTM).As a result, the application backlog had been reduced from over 200,000 pending applications inMarch 2017 to just over 155,000 applications by end of June 2018.

The report notes that substantial challenges persist, particularly regarding India’s patenting and IPenforcement environments nevertheless this improvement is a real accomplishment, and Indianpolicymakers should be congratulated on their successful efforts in 2018.

sri vishnu charan | [email protected] |

https://t.me/PDF4Examshttps://t.me/IAS201819 https://t.me/PDF4Exams

https://t.me/TheHindu_Zone_official

Page 55: Current Affairs -February 1-15, 2019 · 15-02-2019  · 10, theme: ‘Sadak Suraksha-Jeevan Raksha’ National Testing Agency (NTA) launches mobile app through which students can

Current Affairs [PDF] -February 1-15, 2019

© 2019 GKToday | All Rights Reserved | https://www.gktoday.in 55

Plea in Supreme Court in favour of Two Child PolicyA plea filed lawyer Ashwini Upadhyay in Supreme Court has sought the declaration of the ‘two-childnorm’ as a mandatory criterion for government jobs, aids and subsidies and urged that the law dealingwith the condition for recognition of a state or national party be suitably amended in this direction.The plea lists the following appeals:

The non-compliance of the norm should lead to the withdrawal of citizens statutory rights includingthe right to vote and contest elections.Declaring of first Sunday of every month as ‘health day’ to spread awareness against populationexplosion and providing contraceptive pills, condoms, vaccines to economically-weaker sections andfamilies below poverty line.Setting 21 years as the minimum marriageable age for all citizens.Implementation of the 24th recommendation of the National Commission to Review the Working ofthe Constitution (NCRWC) which proposes to control the population by means of education andimplementation of small family norms.The two-child norm for contesting local body elections has been adopted by states like AndhraPradesh, Gujarat, Maharashtra, Odisha, Rajasthan and Haryana has yielded very positive results inreducing the population growth in those states.People representatives are not only public servants but also lawmakers. Hence they should set anexample for others for adopting two child norms so that common people of the country can beencouraged to emulate two-child norm.

The appeal argues that even though India was the first country in the world to have a population policy, ithas not achieved much in terms of population control due to the negligence of governments.

February 9, 2019Government to set-up a Unified Authority for Regulating Entities in IFSC

The Union Cabinet headed by Prime Minister Narendra Modi has approved setting up of a unifiedauthority for regulating all financial services in international financial services centres (IFSCs) in thecountry. The government has approved a bill to set up a unified regulator.

Fact BoxThe first IFSC in India was set up at GIFT City in Gandhinagar, Gujarat.

Promoting the Ease of Doing BusinessIFSCs are set-up to bring back the financial services and transactions that are currently carried out inoffshore financial centres by Indian corporate entities and overseas branches or subsidiaries of financialinstitutions (FIs) to India.In order to ensure this, the business and regulatory environment must be comparable to other leadinginternational financial centres in the world like London and Singapore. Currently, the banking, capitalmarkets and insurance sectors in IFSCs are regulated by multiple regulators like the RBI, SEBI and IRDAIrespectively.For IFSCs to attain its objectives there is a need for inter-regulatory coordination. The establishment of aunified financial regulator for IFSCs will result in providing a world-class regulatory environment tomarket participants from the ease of doing business perspective.For the setting up of a unified regulator, the Union cabinet has approved the International FinancialServices Centres Authority Bill, 2019.

Anti-tank Missile Helina Flight TestedThe Helicopter-launched anti-tank missile Helina was successfully test fired from the Integrated TestRange in Chandipur in Balasore district of Odisha.Features of Helina-Anti Tank MissileThe important features of the Helicopter-launched anti-tank missile Helaina are:

Helina is the indigenously designed and built missile system.Helina is the air-launched variant of the Nag, a fire-and-forget ATGM with an estimated range of 4kilometers.

sri vishnu charan | [email protected] |

https://t.me/PDF4Examshttps://t.me/IAS201819 https://t.me/PDF4Exams

https://t.me/TheHindu_Zone_official

Page 56: Current Affairs -February 1-15, 2019 · 15-02-2019  · 10, theme: ‘Sadak Suraksha-Jeevan Raksha’ National Testing Agency (NTA) launches mobile app through which students can

Current Affairs [PDF] -February 1-15, 2019

© 2019 GKToday | All Rights Reserved | https://www.gktoday.in 56

Helina’s range is estimated at between 7 to 8 kilometres.Helina is guided by an infrared imaging seeker (IIR) operating in the lock-on before-launch modeand helps in further strengthening the defence capabilities of the country.The infrared imaging seeker (IIR) technology is also indigenously developed and has beendemonstrated in the NAG anti-tank guided missile system.Helina is launched from twin-tube stub wing-mounted launchers on board Hindustan AeronauticsLimited’s (HAL) Light Combat Helicopter (LCH) and Dhruv Advanced Light Helicopters (ALH).

Helina is one of the most advanced anti-tank weapons in the world. The production and induction of theHelina missiles are likely to happen in 2019.

Ladakh declared as third administrative division of J and KThe Jammu and Kashmir government ordered the creation of a separate administrative division forLadakh region. It is hailed as a landmark decision by the governor ruled state under President Rule.

Fact BoxThe State of Jammu and Kashmir had two administrative divisions of Jammu and Kashmir earlier. Ladakh would bethe third administrative division.

Ladakh Administrative DivisionThe Ladakh Administrative Division would comprise of Leh and Kargil districts and would beheadquartered in Leh. The Ladakh region was earlier part of the Kashmir division.To facilitate the functioning of the Ladakh Administrative Division a divisional commissioner and aninspector general of police would be appointed.A committee under the chairmanship of Principal Secretary to government has been formed to identifythe posts of divisional level heads of various departments that may be required for the new division;particularly their staffing pattern, roles and responsibilities and proposed location of these offices.The local governance in Kargil and Leh was taken care of by the hill development councils. The remotenessand geographical isolation of the Leh had resulted in the region remained cut off for the developmentalactivities. The administrative division is expected to fasten the pace of the development.

“Law, Justice and Judicial Power – Justice P N Bhagwati’s Approach” Book LaunchedThe book “Law, Justice and Judicial Power – Justice P N Bhagwati’s Approach” written by Mool ChandSharma was launched by the Chief Justice of India, Shri Justice Ranjan Gogoi. The first copy of the bookwas received by the President Ram Nath Kovind.The book is a collection of 24 essays celebrating Justice P N Bhagwati’s judicial work and landmarkjudgments as well as his work towards introducing public interest litigation in India.Justice P N Bhagwati’s contribution to the Indian Judicial SystemJustice Bhagwati has been referred to as the father of public interest litigation in India. He impartedidealism and simplicity to the highest court in the country wherein even a petition filed on a postcard wastaken into consideration.The PIL tradition initiated by him is an Indian contribution to the practice of law and the process of justicedelivery. It is admired by other democracies and other legal systems as well.The Chief Justice of India, Shri Justice Ranjan Gogoi referred to Justice Bhagwati as the “harbinger ofenvironmental jurisprudence” and a “judicial statesman”.Justice Bhagwati evolved constitutional ideas that are still getting shaped. It was Justice Bhagwati whoenvisaged the necessity for specialized Environmental Courts, which later manifested itself in the form ofthe National Green Tribunal. For the first time, the prison reforms were embarked upon in India due tothe efforts of Justice Bhagwati.

India to Train Civil Servants from BangladeshDuring the ongoing visit of the Abul Hassan Mahmood Ali, the Foreign Minister of Bangladesh, India andBangladesh signed a MoU for training 1800 Bangladesh Civil Servants. The MoU was signed betweenIndia’s National Centre for Good Governance (NCGG) and the Ministry of Public Administration ofBangladesh.The NCGG would be training the Bangladesh civil servants for the second time. 1500 Bangladesh Civil

sri vishnu charan | [email protected] |

https://t.me/PDF4Examshttps://t.me/IAS201819 https://t.me/PDF4Exams

https://t.me/TheHindu_Zone_official

Page 57: Current Affairs -February 1-15, 2019 · 15-02-2019  · 10, theme: ‘Sadak Suraksha-Jeevan Raksha’ National Testing Agency (NTA) launches mobile app through which students can

Current Affairs [PDF] -February 1-15, 2019

© 2019 GKToday | All Rights Reserved | https://www.gktoday.in 57

Servants have already been trained in NCGG under the first MOU signed 5 years ago.The civil servants from Bangladesh would be trained at the NCGG an institute under the Department ofAdministrative Reforms and Public Grievances (DAR&PG).Officers of the rank Additional Deputy Commissioners/ Additional District Magistrates, Upazila NirbahiOfficers, Deputy Director Local Government, Senior Assistant Secretaries, Senior AssistantCommissioners, Assistant Commissioners (Land) and Officers of equivalent rank in the Ministries fromBangladesh Civil Service (Administration) cadre would be trained in India.The Civil Servants from Bangladesh will be trained in e-Governance and service delivery, public policy andimplementation, information technology, decentralization, urban development and planning, Ethics inAdministration and challenges in the implementation of SDGs.

Great Indian Bustard is the Mascot for COP-13 on Migratory SpeciesThe Union Government has announced the Great Indian Bustard (GIB) as the mascot for the 13thConference of Parties (COP) of the UN Convention on the conservation of migratory species (CMS) to beheld in Gujarat next year.The logo, mascot and the website for the 13th Conference of Parties (COP) was launched by the UnionMinister for Environment, Forests and Climate Change Harsh Vardhan. The mascot for the event, GreatIndian Bustard has been fondly named as ‘Gibi’.Convention on the Conservation of Migratory Species of Wild Animals (CMS)CMS is an international treaty under the aegis of the United Nations Environment Programme (UNEP). Itis also known as the Bonn Convention. CMS aims to conserve terrestrial, marine and avian migratoryspecies throughout their range.CMS is only global and UN-based intergovernmental organization established exclusively for conservationand management of terrestrial, aquatic and avian migratory species throughout their range.The convention provides a global platform for deliberations on the conservation and sustainable use ofmigratory wild animals and their habitatThe convention was signed in 1979 at Bonn (hence the name Bonn Convention), Germany and entered intoforce in 1983. Its headquarters are in Bonn, Germany. Since its entry into force, the membership hasgrown steadily to include over 120 Parties from Africa, Central and South America, Asia, Europe andOceania.

Voter Verification and Information Programme LaunchedThe Voter Verification and Information Programme (VVIP) was launched by the Election Commissionduring the two-day intensive training-cum-workshop on the use of information & communicationtechnology for the conduct of general elections.During the workshop, training was imparted on the c-VIGIL mobile application. The c-VIGIL app providestime-stamped, evidence-based proof of complaints relating to violation of model code of conduct orexpenditure limit, complete with live photo/video with auto location data.Voter Verification and Information Programme (VVIP)The Voter Verification and Information Programme (VVIP) has been launched by the ElectionCommission to help citizens verify their names in the electoral rolls, register as new voters, update voterdetails and make corrections in their voter identity cards ahead of the general election due in April-May.The VVIP service can be availed by the voters through various channels including voter helplineapplication, voter helpline number 1950 and contact centres in all districts across India.PwD AppDuring the workshop, the PwD App was also launched. The App is aimed at enabling Persons withDisabilities (PwDs) to request for new registration, change in address, change in particulars and markthemselves as PwD through the use of the mobile application was also launched. By simply entering theircontact details, the booth level officer is assigned to provide them doorstep facility. PwDs can also requestfor a wheelchair during polling.

Health Ministry conducts 8th round of National Deworming Day campaignThe Health Ministry undertook the 8th round of National Deworming Day campaign across the countryon 8th February to reduce the prevalence of parasitic intestinal worms among children.

sri vishnu charan | [email protected] |

https://t.me/PDF4Examshttps://t.me/IAS201819 https://t.me/PDF4Exams

https://t.me/TheHindu_Zone_official

Page 58: Current Affairs -February 1-15, 2019 · 15-02-2019  · 10, theme: ‘Sadak Suraksha-Jeevan Raksha’ National Testing Agency (NTA) launches mobile app through which students can

Current Affairs [PDF] -February 1-15, 2019

© 2019 GKToday | All Rights Reserved | https://www.gktoday.in 58

National Deworming DayNational Deworming Day is aimed at deworming all preschool and school-age children (enrolled and non-enrolled) between the ages of 1-19 years through the platform of schools and Anganwadi Centers toimprove their overall health, nutritional status, access to education and quality of life. Albendazole tabletsare orally administered for the children during the programme.The Ministry of Health & Family Welfare, Government of India is the nodal agency for the implementationof the National Deworming Day. The National Deworming Day is implemented by the Ministry of Health& Family Welfare in association with the Department of School Education and Literacy under the Ministryof Human Resource and Development, Ministry of Women and Child Development, Ministry of DrinkingWater and Sanitation, Ministry of Panchayati Raj, Ministry of Tribal Affairs, Ministry of RuralDevelopment, Ministry of Urban Development, and Urban Local Bodies (ULBs).Bi-annual round of deworming is recommended in the States where the prevalence of Soil-TransmittedHelminths infection is more than 20% and annual round in other states. Only two States namely Rajasthanand Madhya Pradesh have reported less than 20% prevalence and have been recommended for the annualround.

61 Individuals Declared Annual Income of Over Rs 100 CroreAs per the information provided by Minister of State for Finance Pon Radhakrishnan to the Lok Sabha 61individuals who declared an income of more than Rs 100 crore during the assessment year 2017-18.Even though the numbers have increased sharply from the 38 individuals reporting income over Rs 100Crore in the previous assessment year, it is still a minuscule number in a population of 1.3 billion Indians.In the assessment year 2014-15, the number of individuals disclosing a gross total income of over Rs 100crore in a year in his/her return of income filed with the Income Tax (IT) department was 24.Even though there has been a sharp increase in the number of individuals declaring income of over 100crore, it has been said the actual numbers are predominantly more and reported numbers are less due tothe large scale underreporting.The following steps have been initiated to prevent the under-reporting:

Government is taking stringent actions under the Benami Properties Transactions Act andproperties valued at Rs 6,900 crore were under attachment by agencies.Income tax authorities had identified more than 2,000 benami transactions till December 2018which includes include deposits in bank accounts, land, apartments and jewellery and theprovisional attachment of properties had been done in over 1,800 cases.The Revenue Department under the Ministry of Finance is setting up a mechanism to ensure that allreturns are processed within 24 hours and refunds issued simultaneously to increase thecompliance in the filing of IT returns.

The government has already sanctioned Rs 4,200 crore for the upgrade of information technologyinfrastructure of Central Board of Direct Taxes (CBDT) for processing returns, refunds, faceless scrutinyand verification.

Indian Foreign Exchange Reserves increased by $2.063 billionThe data from the Reserve Bank of India (RBI) shows that the Foreign Exchange Reserves of Indiaincreased by $2.063 billion to $400.24 billion in the week to February 1. The foreign exchange reserves hadincreased by USD 1.497 billion to reach USD 398.178 billion in the previous week corresponding to January25. The increase in the value of the Foreign Exchange Reserves is attributed to the increase in foreigncurrency assets.India’s Forex ReservesThe components of India’s Forex reserves are:

The Foreign Currency Assets which forms a major component of the overall reserves increased by$1.280 billion to $373.430 billion. Foreign currency assets expressed in USD are subjected to theeffects of appreciation or depreciation of non-US currencies such as the euro, pound and yen held inthe reserves.In the reporting week, the gold reserves also increased by $764.9 million to $22.686 billion.India’s special drawing rights with the International Monetary Fund (IMF) rose by $6.2 million to

sri vishnu charan | [email protected] |

https://t.me/PDF4Examshttps://t.me/IAS201819 https://t.me/PDF4Exams

https://t.me/TheHindu_Zone_official

Page 59: Current Affairs -February 1-15, 2019 · 15-02-2019  · 10, theme: ‘Sadak Suraksha-Jeevan Raksha’ National Testing Agency (NTA) launches mobile app through which students can

Current Affairs [PDF] -February 1-15, 2019

© 2019 GKToday | All Rights Reserved | https://www.gktoday.in 59

$1.470 billion.India’s reserve position with the IMF also increased by $11.2 million to $2.654 billion.

Foreign Exchange Reserves are used to back liabilities on their own issued currency as well as to influencemonetary policy. These reserves act as a buffer during the challenging times to the economy.

PM Lays Foundation Stone for Sela Tunnel in Arunachal PradeshPM Narendra Modi laid Foundation Stone for the Sela Tunnel Project in Arunachal Pradesh across SelaPass.

Sela PassSela Pass is a high-altitude mountain pass located on the border between the Tawang and West Kameng Districtsof Arunachal Pradesh. The pass connects Tawang for rest of India.

Key Facts about the Sela Tunnel ProjectThe important facts about the Sela Tunnel project are:

The Sela pass tunnel will be constructed by the Border Roads Organisation (BRO).The announcement of the project was done in the Union Budget 2018 by then Finance Minister ArunJaitley.Project costs Rs. 687 crore and is expected to be completed in the three years.The project involves the construction of a road for a total distance of 12.04 km which consist of twotunnels of 1790 and 475 meters.The project once completed will reduce the travelling time from Tezpur to Tawang by more thanone hour.The project will also aid travellers in by-passing the dangerous snow-covered Sela top at a height of13,700 feet.

An asset to the Defence ForcesThe Sela tunnel project will provide all-weather connectivity and reduce travelling time. This aids theswift and fast movement of army troops in Tawang which is a strategically-located district located nearthe border of China. The tunnel will reduce the travel time between the Indian Army’s 4 Corpsheadquarters at Tezpur in Assam and Tawang.The tunnel project will also boost the tourism industry and economic activities in the region as well asnorth-eastern states.

PM Launches Development Works in Arunachal PradeshPM Narendra Modi launched various developmental works in the state of Arunachal Pradesh whichincludes:

DD Arunprabha – a dedicated 24×7 Satellite channel for Arunachal Pradesh.Sela Tunnel Project.Film and Television Institute in Jote.

DD ArunprabhaDD Arunprabha is the 24th satellite channel operated by Doordarshan. It will showcase the rich traditionand diversity of local culture. Together with aiding in integrating North-East with the rest of the country,the channel will bring alive the magnificence of North-East by airing content sensitive to the needs andaspirations of local population including news, travelogues, mythological shows, documentaries,magazines, telefilms, reality shows, daily shows etc.DD Arunprabha is equipped with state of the art facilities, including a Digital Satellite News Gatheringunit to provide live coverage from remote locations, for the 24×7 telecast. The seamless transmission ofDD Arunprabha will be ensured by playout facility and the earth stations established at DDK Itanagar.Film and Television Institute in JoteThe permanent campus of Film and Television Institute would be established at Jollang-Rakap (Jote),Papum Pare, Arunachal Pradesh. It would be the third Film and Television Institute of Ministry of I&B (theother two are: FTII Pune and SRFTI Kolkata) and first in the North-East. The institute will provide a boostto the talented youngsters of the North-East in the film and television sector.

sri vishnu charan | [email protected] |

https://t.me/PDF4Examshttps://t.me/IAS201819 https://t.me/PDF4Exams

https://t.me/TheHindu_Zone_official

Page 60: Current Affairs -February 1-15, 2019 · 15-02-2019  · 10, theme: ‘Sadak Suraksha-Jeevan Raksha’ National Testing Agency (NTA) launches mobile app through which students can

Current Affairs [PDF] -February 1-15, 2019

© 2019 GKToday | All Rights Reserved | https://www.gktoday.in 60

New Policy for Overseas BorrowingsThe Reserve Bank has come out with a new policy for overseas borrowings. The RBI has rationalized theearlier policy in consultation with the Government of India.Features of the New Overseas Borrowing PolicyThe features of the new policy are:

The new policy allows all eligible borrowers to raise External Commercial Borrowings (ECB) up to$750 million or equivalent per financial year under the automatic route replacing the existing sectorwise limits.To liberalise the framework ECB and rupee-denominated bonds Tracks I and II under the existingframework are merged as ‘Foreign Currency denominated ECB’ and Track III and RupeeDenominated Bonds framework are combined as ‘Rupee Denominated ECB’ to replace the currentfour-tiered structure (Track I, II and III denotes amount and maturity of funds raised).All-in cost ceiling per annum has been pegged at the benchmark rate plus 450 bps spread.The minimum average maturity period (MAMP) has been kept at three years for all ECBs,irrespective of the amount of borrowing except the borrowers specifically permitted in the circularto borrow for a shorter period.The list of eligible borrowers has been expanded to include all entities eligible to receive foreigndirect investment (FDI).The new policy allows port trusts, units in SEZ, SIDBI, EXIM Bank, registered entities engaged inmicro-finance activities, registered societies/trusts/ cooperatives and non-governmentorganisations to borrow.Negative list for which the ECB proceeds cannot be utilised includes real estate activities,investment in the capital market, equity investment, working capital purposes except foreign equityholder, repayment of Rupee loans for except foreign equity holder.

The recent changes that have been brought out in the ECB policy are likely to help wider set of eligibleborrowers i.e. corporates and other entities to avail ECBs to meet their capital needs with the UniformMinimum Average Maturity Period requirements, uniform all-in-cost ceilings and small negative end-userlist.

February 10, 2019Family Planning for Males

Due to the orthodoxy and parochial nature of Indian Society, males prefer to lay outside the ambit of thefamily planning and the onus lies on the woman. In order to arrest these tendencies the government ofIndia has undertaken the following steps:

Vasectomy Fortnight is observed across the country in the month of November to increaseawareness on male participation and promotion of male sterilization.Training of service providers in No-Scalpel Vasectomy (NSV) has been undertaken to enhance thepool of service providers.Condom boxes are set up in facilities to enable people to access condoms in privacy.The ‘Home Delivery of Contraceptive Scheme’ has been initiated to deliver contraceptives includingcondoms to the beneficiaries.The compensation for male sterilization has been substantially increased under the Enhancedcompensation scheme for sterilization.A 360-degree media campaign has been undertaken to underline the role of men in family planningto encourage men to adopt family planning methods.Under the Mission Parivar Vikas campaign due attention is paid towards extensive mobilizationactivities including promotion of male participation and assured vasectomy services.

The national average for the use of male family planning methods (Male sterilization + Condom) is 5.9 %and 13 states have a better participation rate ththe an national average.

Health and Wellness Centres- Ayushman BharatThe Ayushman Bharat scheme has two sub-components:

sri vishnu charan | [email protected] |

https://t.me/PDF4Examshttps://t.me/IAS201819 https://t.me/PDF4Exams

https://t.me/TheHindu_Zone_official

Page 61: Current Affairs -February 1-15, 2019 · 15-02-2019  · 10, theme: ‘Sadak Suraksha-Jeevan Raksha’ National Testing Agency (NTA) launches mobile app through which students can

Current Affairs [PDF] -February 1-15, 2019

© 2019 GKToday | All Rights Reserved | https://www.gktoday.in 61

PM JAY- Pradhan Mantri Jan Arogya Yojana which is a health insurance scheme for 10.74 crorefamilies identified through SECC database.Health and Wellness Centres: Delivery of the comprehensive Primary health care through Healthand Wellness centres covering prevention, promotion and ambulatory care at the primary level.

Ayushman Bharat – Health and Wellness CentresTo expand access to Comprehensive Primary Health Care (CPHC), Health and Wellness Centres (HWCs),Sub Health Centres (SHCs) and Primary Health Centres (PHCs) are being strengthened as Health andWellness Centres (HWCs) under the Ayushmann Bharat Scheme.These Health and Wellness Centres aim to provide preventive, promotive, rehabilitative and curative carefor an expanded range of services encompassing reproductive and child health services, communicablediseases, non-communicable diseases, palliative care and elderly care, oral health, ENT care, and basicemergency care.The Health and Wellness centres provide comprehensive primary health care through:

Expended Service Delivery.Continuum of Care – Telehealth/ Referral.Expanding HR- MLHP & Multiskilling.Medicines & Expanding Diagnostics.Community Mobilisation and Health Promotion.Robust IT System.Infrastructure.Partnership for Knowledge & Implementation.Financing/ Provider Payment Reforms.

The services in HWCs will be provided through a Mid-level Health Care Provider (MLHP)/CommunityHealth Officer (CHO) who is required to undergo a Certificate in Community Health through IGNOU orpublic university.

ILO Centenary Celebrations LaunchedThe ILO Centenary Celebrations in India were inaugurated by Union Minister of State (I/C) for Labour andEmployment Shri Santosh Kumar Gangwar at V.V. Giri National Labour Institute, Noida.India being a founding member has played an active role in every field of the organization. India hasratified 47 out of 189 ILO conventions during the century which includes all eight core conventions.International Labour OrganisationInternational Labour Organisation (ILO) is a tripartite organisation where representatives from thegovernment, employers and employees openly debate and create labour standards. ILO is celebrating its100th anniversary in 2019. As part of the 100th Anniversary, ILO is implementing seven centenaryinitiatives, which are:The future of work initiativeThe world of work is undergoing a major process of change. In order to understand and to respondeffectively to these new challenges, the International Labour Organization has launched a “Future of Workinitiative” in order to be able to advance its mandate for social justice.The end to poverty initiativeThe End to Poverty Centenary Initiative is designed specifically as the vehicle to take forward the ILO’swork in implementing the 2030 Agenda for Sustainable Development.The women at work initiativeThe women at work initiative aim to better understand and to address, why progress on delivering ondecent work for women has been so slow and what needs to be done towards securing a better future forwomen at work.The green initiativeThe Green Initiative aims to scale up the ILO’s knowledge, policy response and capacity to manage a justtransition toward greener economies and a sustainable future.

sri vishnu charan | [email protected] |

https://t.me/PDF4Examshttps://t.me/IAS201819 https://t.me/PDF4Exams

https://t.me/TheHindu_Zone_official

Page 62: Current Affairs -February 1-15, 2019 · 15-02-2019  · 10, theme: ‘Sadak Suraksha-Jeevan Raksha’ National Testing Agency (NTA) launches mobile app through which students can

Current Affairs [PDF] -February 1-15, 2019

© 2019 GKToday | All Rights Reserved | https://www.gktoday.in 62

The standards initiativeThe standards initiative aims to enhance the relevance of international labour standards through astandards review mechanism and to consolidate tripartite consensus on an authoritative supervisorysystem.The enterprises initiativeThe enterprises initiative aims to establish a platform for ILO engagement with enterprises which wouldcontribute to their sustainability and to ILO goals.The governance initiativeThe Governance Initiative aims to complete the reform of the ILO’s governance structures, to undertakethe evaluation of the impact of the 2008 Social Justice Declaration, and act on the findings and outcomesof the evaluation.

World Pulses Day-2019February 10, 2019, was celebrated as the first World Pulses Day. The Year 2016 was celebrated asInternational Year of Pulses to highlight the contribution of Pulses to sustainable food production andtowards food security and nutrition. Building on the positive momentum of the International Year ofPulses, the UN General Assembly passed the resolution to celebrate 10 February as World Pulses Day tokeep the positive momentum surrounding these healthy, nutritious, protein-rich, nitrogen-fixing legumesalive after the FAO’s successful 2016 International Year of Pulses Campaign.What are the Pulses?Pulses are a type of leguminous crops that are harvested solely for the dry seed and does not include cropsthat are harvested green (e.g. green peas, green beans). Dried beans, lentils and peas are the mostcommonly known and consumed types of pulses. Pulses include all varieties of dried beans, such as kidneybeans, lima beans, butter beans and broad beans. Chickpeas, cowpeas, black-eyed peas and pigeon peas arealso pulses, as are all varieties of lentils.Significance of PulsesNutritionPulses are packed with nutrients and have a high protein content, making them an ideal source of proteinparticularly in regions where meat and dairy are not physically or economically accessible. Pulses are alsolow in fat and rich in soluble fibre, which can lower cholesterol and help in the control of blood sugar.Pulses also aid in combating obesity.Food securityThe nitrogen-fixing properties of pulses improve soil fertility, which increases and extends theproductivity of the farmland. Pulses, when used for intercropping and as cover crops, promote farmbiodiversity and soil biodiversity, while keeping harmful pests and diseases at bay. This enhances foodsecurity.Climate change mitigationPulses contribute to climate change mitigation by reducing dependence on the synthetic fertilizers usedto introduce nitrogen artificially into the soil. This reduced use of synthetic fertilizers will reducegreenhouse gas emissions during their production.

MEA to Play Facilitating Role to Promote Silk in Global MarketsThe Minister of External Affairs, Sushma Swaraj announced that the Ministry of External Affairs wouldplay a facilitating role in promoting the silk in the global markets.The External Affairs Minister urged the Textile Ministry to increase the production to capture thegrowing market.Eradication of the thigh reeling practice in the tasar silk sectorThe Ministry of Textiles as part of its effort to eradicate thigh reeling practice in tasar silk sector and toensure rightful earning to the rural and tribal women reelers is distributing Buniyaad reeling machine toweavers. A target has been set to eradicate thigh reeling practice by March 2020.Mobile Application for quality certificationThe silk production capacity of India is expected to increase to 38,500 tonnes by 2020 from the currentlevel of 32,000 tonnes. Increasing production will help in reducing import as India is the second largest

sri vishnu charan | [email protected] |

https://t.me/PDF4Examshttps://t.me/IAS201819 https://t.me/PDF4Exams

https://t.me/TheHindu_Zone_official

Page 63: Current Affairs -February 1-15, 2019 · 15-02-2019  · 10, theme: ‘Sadak Suraksha-Jeevan Raksha’ National Testing Agency (NTA) launches mobile app through which students can

Current Affairs [PDF] -February 1-15, 2019

© 2019 GKToday | All Rights Reserved | https://www.gktoday.in 63

producer of silk after China and the largest consumer of silk.Hence to ensure the quality of the silk, the Ministry of Textiles has also launched a mobile application e-cocoon for quality certification in the silkworm seed sector.

Surging Silk – Accomplishment and Way ForwardThe Ministry of Textiles in association with Central Silk Board had organized an event, Surging Silk –Accomplishment and way forward on 9th February 2019 at in New Delhi.The event was presided over by Union Minister of Textiles, Smriti Zubin Irani and Union Minister ofExternal Affairs, Sushma Swaraj was the chief guest for the event.Surging Silk – Accomplishment and Way ForwardThe Surging Silk – Accomplishment and Way Forward event was conducted to:

Highlight the growth of the silk industry in India over the past 4 years.Distribution of Buniyaad Tasar Silk Reeling Machines to women reelers from tribal areas, as part oftotal eradication of the age-old practice of thigh reeling.Honouring the best achievers of the silk industry across various segments of sericulture.Awarding the best performing states.The release of melange fabric produced from silk waste.The launch of mobile application e-cocoon for quality certification in the silkworm seed sector.Releasing of a compendium on Indian silk industry which will be a comprehensive compilation onthe status and growth of sericulture and silk industry of each state, plans and programmes launchedby the government being implemented across the country and ethnic silk products supported with adirectory of important contacts points, to have first-hand information of the industry.State sericulture profile.

The Textile Ministry had made elaborate arrangements for participation from different spectrum of theTextile industry in large numbers.

PM Inaugurates Development Works at TripuraPrime Minister Narendra Modi inaugurated the development works at Tripura during his visit. Thedevelopment projects inaugurated were:

A 23.32-km railway track in southern Tripura. This 23 Km Garjee-Belonia section is built at a cost ofRs 400 crore and is part of the Rs 3,407-crore Agartala-Sabroom (115 km) national railway projectaiming to link southern Tripura’s last border town (Sabroom).This new railway line would facilitate the carriage of goods and passengers from the northeasternstates on the Bangladeshi railway network.New complex of Tripura Institute of Technology (TIT) at Narsingarh. The new complex was built at acost of Rs 103 crore. The cost was shared by the state and the Central government.Unveiled a statue of Tripura’s last ruler, Maharaja Bir Bikram Kishore Manikya at the Agartalaairport.

Maharaja Bir Bikram Kishore ManikyaMaharaja Bir Bikram Kishore Manikya (reigned 1923-47) is considered as the father of modern architecturein Tripura, as the entire planning of present-day Tripura was initiated during his rule including planningof the capital city of Tripura, Agartala.In 1939, the Maharaja reserved land for the local Tripura tribals, which was later instrumental in thecreation of the Tripura autonomous district council.He is also credited for the construction of the first airport at Agartala in Tripura, which is today thesecond busiest airport in the entire northeast.The statue of the Maharaja Bir Bikram Kishore Manikya is designed and built by the faculty and studentsof Tripura government’s Art and Craft College at a cost of Rs 5 lakh.The statue was unveiled as part of the efforts of the union government policy of honouring the unsungheroes of India, who have contributed greatly to nation-building.

Meghalaya govt to rename Dikkibandi stadium after P A SangmaThe government of Meghalaya has decided to rename the Dikkibandi stadium after P A Sangma. Thedecision was made by the state cabinet as per the recommendations made by state naming authority

sri vishnu charan | [email protected] |

https://t.me/PDF4Examshttps://t.me/IAS201819 https://t.me/PDF4Exams

https://t.me/TheHindu_Zone_official

Page 64: Current Affairs -February 1-15, 2019 · 15-02-2019  · 10, theme: ‘Sadak Suraksha-Jeevan Raksha’ National Testing Agency (NTA) launches mobile app through which students can

Current Affairs [PDF] -February 1-15, 2019

© 2019 GKToday | All Rights Reserved | https://www.gktoday.in 64

tasked with the job to name roads, and institutions after personalities. Along with the Dikkibandi stadium,a junction leading to AMPT road in Tura have been renamed after P A Sangma.P A SangmaPurno Agitok Sangma has served as the Speaker of the Lok Sabha and the Chief Minister of Meghalaya. Hebegan his political journey with the Congress party and represented the Tura constituency of Meghalayain Lok Sabha multiple times.P A Sangma served as the Chief Minister of Meghalaya from 1988 to 1990. He formed the NationalistCongress Party in 1999 along with Sharad Pawar and Tariq Anwar.He contested the elections for the post of President of India in 2012 and lost the elections to PranabMukherjee. P A Sangma died from cardiac arrest in New Delhi on 4 March 2016 at the age of 68.

February 11, 2019Exercise Cutlass Express 2019

The annual multinational maritime exercise Cutlass Express was held from 27 Jan to 06 Feb 19. Theexercise is aimed to improve law enforcement capacity, promote regional security and progress inter-operability between the armed forces of the participating nations for the purpose of interdicting illegalmaritime activity in the Western Indian Ocean.The annual Exercise Cutlass Express is sponsored by U.S. Africa Command (USAFRICOM) and conductedby Naval Forces Africa (NAVAF). Exercise Cutlass Express 2019 is the eighth iteration of the annualmultinational maritime exercise and was held in the vicinity of Djibouti, Mozambique and Seychelles.The participating nations in the exercise Cutlass Express 2019 were Canada, Comoros, Djibouti, France,India, Kenya, Madagascar, Mauritius, Mozambique, Portugal, Seychelles, Somalia, Tanzania, TheNetherlands and the United States. The Indian Navy was represented in the exercise through INS Trikand.During the exercise, Naval, Coast Guard and Marine Police personnel from a number of East Africancountries were jointly trained by mentors from USA, India, and Netherlands, with the support ofinternational organisations like the International Maritime Organisation (IMO), Combined Maritime Force(CMF) and European Naval Forces (EUNAVFOR).

Asiatic Lion Conservation ProjectThe Asiatic Lion Conservation Project was announced by the Centre and the Gujarat government at a costof Rs 97.85 crore.Asiatic Lion Conservation ProjectAsiatic Lion Conservation Project would use modern information and communication technology forconservation and protection efforts of the Great Gir Region, including GPS-based animal and vehicletracking, automated sensor grid with movement sensors, night vision capability and real-time monitoringand report generation.The project will also have a dedicated veterinary institute, “lion ambulances”, and back-up stocks ofvaccines that may be required.The government of Gujarat has envisaged a ‘Greater Gir’ that includes, other than the existing GirNational Park, sanctuaries in Girnar, Pania and Mitiyala. The conservation project also includes “habitatimprovement” measures, making more sources of water available, creating a wildlife crime cell, and a taskforce for the Greater Gir region.Conservation of Asiatic LionIn 2013 Supreme Court had ordered translocation of some lions from Gujarat to Madhya Pradesh withinsix months and the Kuno-Palpur Wildlife Sanctuary in Madhya Pradesh was identified to be the mostsuitable for reintroducing the species. But no step forwards was made in this regard.The translocation was planned as part of long-term survival plan so that the objectives of the conservationwould be best served if they could be present outside Gujarat, too, so that they are protected againstcalamities like a forest fire, a disease, or calamities. The deaths of the Asiatic lions in Gujrat last year hasagain brought the questions of conservation to the forefront.The other challenge is the increasing number of Asiatic Lions in Gujrat. As a result, several of them werenow found outside protected areas and involved in the human-animal conflict as well as in increasingcontact with domestic animals as well as feral dogs, from where they could have contracted the virus.

sri vishnu charan | [email protected] |

https://t.me/PDF4Examshttps://t.me/IAS201819 https://t.me/PDF4Exams

https://t.me/TheHindu_Zone_official

Page 65: Current Affairs -February 1-15, 2019 · 15-02-2019  · 10, theme: ‘Sadak Suraksha-Jeevan Raksha’ National Testing Agency (NTA) launches mobile app through which students can

Current Affairs [PDF] -February 1-15, 2019

© 2019 GKToday | All Rights Reserved | https://www.gktoday.in 65

India successfully tests Solid Fuel Ducted Ramjet Propelled MissileIndia has successfully undertaken the demonstration of the indigenously developed propulsiontechnology. The successful demonstration of indigenous technology is a step forward in the developmentof long-range air-to-air missiles.A Long JourneyIndia had embarked on the journey to indigenously develop the Solid Fuel Ducted Ramjet technology in2013 with estimated funding of nearly Rs 500 cr to develop the technology and demonstrate it in 5 years.The Defence Research Development Laboratory (DRDL) at Hyderabad was the lead agency for thecollaborative mission project. The first test of this indigenous technology was carried out in May 2018.The recent one was a second trail and it established a series of technologies including the ground booster,separation of the ground booster and nozzle-less booster.During the test flight, the missile was guided to high altitude to simulate aircraft release conditions andsubsequently the nozzle-less. All the mission parameters were successfully met.How Solid Fuel Ducted Ramjet propelled missile is different?The Solid Fuel Ducted Ramjet propelled missile uses the air-breathing ramjet propulsion technology,which helps propel the missile at high supersonic speeds (above Mach 2) for engaging targets at longranges.

ASCEND 2019ASCEND 2019 is an ambitious initiative of the Kerala government to position the state as an investmenthotspot. Union Minister of Commerce & Industries and Civil Aviation Suresh Prabhu was the chief guestat the opening ceremony of the day-long conclave, ASCEND 2019.ASCEND 2019ASCEND 2019 aims to make Kerala a top investment destination and empower entrepreneurs to launchtheir enterprises in a speedy and hassle-free manner. The important aspects of the ASCEND 2019 are:

‘Invest Kerala Guide’ – a compendium of administrative and policy reforms was released by theChief Minister Pinarayi Vijayan. Invest Kerala Guide provides a ready reckoner on the immensebusiness opportunities in the state and simplified procedures and protocol in making theinvestment.The two-day conclave aims to bring into focus governance that is simplified, efficient andtransparent and facilitates business and investments by clipping a plethora of irksome bureaucraticprocedures.Invest Kerala Portal would be launched to improve the investment ecosystem of the state and helpentrepreneurs from India and abroad to set up a wide array of enterprises and conduct business in acongenial environment.The two-day initiative will also display the landmark administrative and policy reforms that haveconsiderably improved Ease of Doing Business in Kerala and raised its profile among both industryleaders and young entrepreneurs for setting up their shops.K-SWIFT (Kerala Single Window Interface for Fast, Transparent Clearances) and IBPMS (IntelligentBuilding Plan Management System) will be displayed on the two-day conclave.KSWIFT is an online clearance mechanism designed by the Kerala government to simplify and speedup the issuance of clearances from Departments / Agencies for launching enterprises in the state.

The two-day conclave would be attended by the leading lights from industry and businesses.Examining the Government Data on Job Creation

The Prime Minister while addressing the Lok Sabha made the following claims about the creation:New jobs were created in the unorganised sector, which accounts for 80-85 per cent employment.PM cited an increase in the sale of commercial vehicles, infrastructure building and housingactivities as evidence for the claim.PM further said that 1.8 crore people had enrolled in the Employees’ Provident Fund (EPF) in the past15 months, and 64 per cent of them, who were below the age of 28 were first-time employees.PM also cited data showing an increase in the registration of employees under the National PensionScheme (NPS) from 65 lakh in March 2014 to 1.2 crore till October 2018.

sri vishnu charan | [email protected] |

https://t.me/PDF4Examshttps://t.me/IAS201819 https://t.me/PDF4Exams

https://t.me/TheHindu_Zone_official

Page 66: Current Affairs -February 1-15, 2019 · 15-02-2019  · 10, theme: ‘Sadak Suraksha-Jeevan Raksha’ National Testing Agency (NTA) launches mobile app through which students can

Current Affairs [PDF] -February 1-15, 2019

© 2019 GKToday | All Rights Reserved | https://www.gktoday.in 66

Why the critics claim this argument as flawed?The critics refuse to buy these data on job creation based on the following grounds:

Considering the EPF enrolments as a reflector of job creation is not correct as it can be theformalisation of informal jobs.Enrolment for the EPF does not necessarily mean that the person has got a job. It is most likely thatthe person has enrolled for the EPF for the first time though he/she had a job for a long time.As per the law, an employer with 20 people or more is required to register with the EPFO. If a firmhad 19 employees till yesterday and today 20th person joins in, then all 20 would be enrolled for thefirst time. Hence what is perceived as 20 jobs created is actually one.PM also said that new 6.35 lakh new non-corporate taxpayers such as doctors must have providedjobs in the past four years. Experts call it “incomprehensible” in the absence of the source of the data.As per data from CMIE India’s unemployment rate shot up to 7.4 per cent in December 2018 and thenumber of unemployed increased by a substantial 11 million – the highest in 15 months.

Further critics cite the alleged data from the NSSO survey which is said to be withheld by the government.It is said that the report puts the unemployment rate at 6.1 per cent in 2017-18, post-demonetisation. It isthe highest level of unemployment since 1972-73 – the period since when the jobs data is comparable.

Hindi Included as third official court language in Abu DhabiAbu Dhabi has included Hindi as the third official language to be used in its courts, alongside Arabic andEnglish. It is part of the efforts of Abu Dhabi to improve access to justice.Inclusion of HindiThe Abu Dhabi Judicial Department (ADJD) has announced that it has extended the adoption ofinteractive forms of statement of claims filed before courts by including the Hindi language alongsideArabic and English in labour cases.The inclusion of Hindi language is aimed at helping Hindi speakers to learn about litigation procedures,their rights and duties without a language barrier, in addition to facilitating registration procedures viaunified forms available through the ADJD website.The adoption of multilingual interactive forms for claim sheets, grievances and requests aim to promotejudicial services in line with the plan Tomorrow 2021 and increase the transparency of litigationprocedure.Indians in the UAEThe population of UAE is estimated to be around nine million of which 2/3rd are immigrants from foreigncountries and the Indian community in the UAE, numbering 2.6 million, constitutes 30 per cent of thetotal population and is the largest expatriate community in the country.

Chinook Helicopters to be formally inducted into Indian Air ForceThe first batch of four Chinook military helicopters manufactured by the American aerospace majorBoeing has arrived at the Mundra port in Gujarat. This first batch consignment will be ferried toChandigarh, where they will be formally inducted into the Indian Air Force in 2019.Chinook HelicoptersChinook is a multi-role, vertical-lift platform, which is used for transporting troops, artillery, equipmentand fuel. Chinook would be deployed for humanitarian and disaster relief operations and in missions suchas transportation of relief supplies and mass evacuation of refugees.The advanced multi-mission helicopter Chinook will provide the Indian armed forces with unmatchedstrategic airlift capability across the full spectrum of combat and humanitarian missions.India had placed a mega contract to Boeing for the procurement of 22 Apache helicopters and 15 Chinooksfor the Indian Air Force in September 2015. To deliver on the order, Boeing has strengthened its supplychain with over 160 partners in India. Boeing has also planned a joint venture to manufacture fuselages forApache helicopters and the annual sourcing from India stands at $1 billion.

India’s exports to China set to touch Record HighIndia’s export to China is expected to reach an all-time high in the financial year 2018-19.Increase in ExportsThe increase in the exports to China is attributed to US-China trade spat creating new opportunities forexporters and Beijing removing some trade barriers in its effort to check the growing bilateral trade

sri vishnu charan | [email protected] |

https://t.me/PDF4Examshttps://t.me/IAS201819 https://t.me/PDF4Exams

https://t.me/TheHindu_Zone_official

Page 67: Current Affairs -February 1-15, 2019 · 15-02-2019  · 10, theme: ‘Sadak Suraksha-Jeevan Raksha’ National Testing Agency (NTA) launches mobile app through which students can

Current Affairs [PDF] -February 1-15, 2019

© 2019 GKToday | All Rights Reserved | https://www.gktoday.in 67

imbalance.The growth of exports was driven by the following sectors which include marine products, organicchemicals, plastics, petroleum products, grapes and rice.Bridging the Trade DeficitIndian exports are expected to reach an all-time high in the current financial year. The exports betweenApril and December stood at $12.7 billion which is closer to last year’s exports of $13.33 billion.India feels that more efforts are required for greater market penetration in China to bridge the tradedeficit. India’s trade deficit with China stood at $63 billion in 2017-18. This $63 billion trade deficit wasmore than a third of India’s total trade deficit. The exports to China stood at $13.3 billion and imports fromChina stood at $76.38 billion in 2017-18.Acknowledging the need to take steps to check the deficit, the following steps have been initiated tobridge the deficit:

China signed three export protocols with India on rice, fishmeal and tobacco to allow imports of thethree items.An announcement regarding Chinese import quotas for sugar and rice for 2019 is expected soonwhich would aid in planning their exports well in time.India is also expecting to sign a protocol for export of Indian soyabean meals, cakes andpomegranates to China in the near future.

To make the most of the US-China trade dispute, the Department of Commerce had taken the initiative toidentify and share with Indian exporters and other stakeholders, specific lines where the US will losecompetitiveness in China and where India had an export potential.

US considers withdrawing Generalised System of Preferences from IndiaThe United States is reportedly considering withdrawal of benefits to India under its Generalised Systemof Preferences amid a widening dispute over its trade and investment policies.The US Trade Representative (USTR) is in the process of completing the review of India’s status as a GSPbeneficiary and an announcement was expected over the next two weeks.The withdrawal of the Generalised System of Preferences from India could be the strongest punitiveaction since President Donald Trump took office in 2017 vowing to reduce the US deficit with largeeconomies. The Trump administration has repeatedly called out India for its high tariffs.Generalised System of PreferencesGeneralised System of Preferences instituted in 1976 under the Trade Act of 1974 is a U.S. trade programdesigned to promote economic growth in the developing world by providing preferential duty-free entryfor up to 4,800 products from 129 designated beneficiary countries and territories.Impact on Trade with IndiaThe withdrawal of the Generalised System of Preferences from India would result in the elimination ofduty-free access for about 2,000 Indian product lines. This will hurt small businesses such as jewellery.This will adversely affect Indian exports to the US. After the withdrawal of the Generalised System ofPreferences number of goods qualifying for preferential treatment could be reduced, or the wholeprogramme could be withdrawn.

125th Constitutional Amendment BillThe government has introduced the 125th Constitutional Amendment Bill in the Parliament to increasethe financial and executive powers of the 10 Autonomous Councils in the Sixth Schedule areas of thenortheastern region.Features of the Amendment BillThe salient features of the 125th Constitutional Amendment Bill which will impact one crore tribal peoplein Assam, Meghalaya, Tripura and Mizoram are:

The amendments proposed provide for elected village municipal councils, ensuring democracy atthe grass-root level.The amendment empowers the village councils to prepare plans for economic development andsocial justice including those related to agriculture, land improvement, implementation of landreforms, minor irrigation, water management, animal husbandry, rural electrification, small scale

sri vishnu charan | [email protected] |

https://t.me/PDF4Examshttps://t.me/IAS201819 https://t.me/PDF4Exams

https://t.me/TheHindu_Zone_official

Page 68: Current Affairs -February 1-15, 2019 · 15-02-2019  · 10, theme: ‘Sadak Suraksha-Jeevan Raksha’ National Testing Agency (NTA) launches mobile app through which students can

Current Affairs [PDF] -February 1-15, 2019

© 2019 GKToday | All Rights Reserved | https://www.gktoday.in 68

industries and social forestry.The amendment bill mandates the finance commission to recommend devolution of financialresources to these autonomous councils. The Autonomous Councils are dependent on grants fromCentral ministries and the State government for specific projects.The amendment bill also reserve one-third of the seats for women in the village and municipalcouncils in the Sixth Schedule areas of Assam, Mizoram and

The bill to empower the Autonomous Councils was announced by the government last month in thebackdrop of the protests in the North-East following the passage of the Citizenship Amendment Bill, 2019,in the Lok Sabha.

World Government Summit 2019The seventh annual World Government Summit is being held in Dubai (UAE). The World GovernmentSummit is a forum dedicated to shaping the future of governments. The Summit is being attended by4,000 decision-makers from 140 countries including heads of state, ministers and business leaders.World Government SummitThe World Government Summit is a global platform dedicated to shaping the future of governmentsworldwide. The Summit:

Sets the agenda each year for the next generation of governments, focusing on how they can harnessinnovation and technology to solve universal challenges facing humanity.Provides a unique platform to showcase and exchange best practices and smart solutions to thechallenges that nations face.

About the OrganisationThe World Government Summit is a neutral, non-profit organization at the intersection of governmentand innovation. The World Government Summit functions as a knowledge exchange platform for leadersin the public and private sectors to convene and collaborate with world-renowned experts to create apositive impact for citizens worldwide.The Organisation comprises a wide network of leaders of the public and private sectors and inspires themto think collectively and creatively about disruptive intersections between government and innovation,with the ultimate goal of promoting development and prosperity.

Eight medical Equipment as Drugs by the Health MinistryThe government has notified the following eight medical equipments as Drugs after consultation with theDrugs Technical Advisory Board:

All implantable devices.CT scan.PET equipment.MRI equipment.Defibrillators.Dialysis machines.Bone marrow separators.X-ray machine.

The notification has been made under Section 3 of the Drugs and Cosmetics Act, with effect from April 1,2020. The notification brings all the implantable and diagnostic devices will come under the regulatoryframework. This is a big step forward as the majority of the medical devices were unregulated in India.Drugs Technical Advisory Board (DTAB)DTAB is the highest statutory decision-making body on technical matters related to drugs in India. It wasconstituted under the provisions of the Drugs and Cosmetics Act, 1940. DTAB is part of the Central DrugsStandard Control Organization (CDSCO) under the Ministry of Health and Family Welfare.

Sarthi Sandesh VahiniThe Sarthi Sandesh Vahini vehicles were flagged off by Uttar Pradesh Chief Minister Yogi Adityanath.The Uttar Pradesh state family welfare department has initiated the Sarthi Sandesh Vahini mission tospread awareness regarding family planning in urban and rural areas. The Sarthi Sandesh Vahini vehicleswill provide family planning information to the viewers through various documentaries and films.

sri vishnu charan | [email protected] |

https://t.me/PDF4Examshttps://t.me/IAS201819 https://t.me/PDF4Exams

https://t.me/TheHindu_Zone_official

Page 69: Current Affairs -February 1-15, 2019 · 15-02-2019  · 10, theme: ‘Sadak Suraksha-Jeevan Raksha’ National Testing Agency (NTA) launches mobile app through which students can

Current Affairs [PDF] -February 1-15, 2019

© 2019 GKToday | All Rights Reserved | https://www.gktoday.in 69

To ensure the objectives of the mission are accomplished the Uttar Pradesh government will appoint thenodal officers to supervise the campaign and ensure that several targets set by the department includingwomen and child development and education for all are being fulfilled.Tackling EncephalitisThe Uttar Pradesh government has also launched the new initiative to eradicate encephalitis.

EncephalitisEncephalitis is an uncommon but serious condition in which the brain becomes inflamed (swollen). Encephalitis canbe life-threatening and requires urgent treatment in hospital. The very young and very old are most at risk ofEncephalitis.

The mission aims to spread awareness about the disease and provide medical assistance to the needy.

GeM and CCI Sign MoUGovernment e Marketplace (GeM) and Competition Commission of India (CCI) have signed aMemorandum of Understanding to enable a fair and competitive environment in the e-Marketplace.Combating Anti-competitive practicesBoth CCI and GeM attach significant importance to the advanced analytical tools and processes foridentification of malpractices like cartelization. Hence to pool their knowledge of the public procurementdomain for detection of Anti-competitive practices GeM and CCI have signed a MoU.Government e Marketplace (GeM)GeM is a state-of-the-art national public procurement platform of Ministry of Commerce and Industries.GeM facilitates online procurement of common use Goods & Services required by various GovernmentDepartments / Organisations / PSUs.GeM provides the tools of e-bidding, reverse e-auction and demand aggregation to facilitate thegovernment users, achieve the best value for their money.GeM aims to enhance transparency, efficiency and speed in public procurement and has used technologyto remove entry barriers for bonafide sellers and has created a vibrant e-marketplace with a wide range ofgoods and services.Competition Commission of India (CCI)Competition Commission of India is a statutory body of the Government of India, responsible forenforcing the Competition Act, 2002 throughout the country and to prevent activities that have anadverse effect on competition. CCI consists of a Chairperson and 6 Members appointed by the CentralGovernment.

India-Norway Marine Pollution InitiativeIndia and Norway signed a letter of Intent for establishing the India-Norway Marine Pollution Initiative.The letter of intent was signed by the Ministry of Environment, Forests and Climate Change, Governmentof India together with the Norwegian Ministry of Foreign Affairs.India-Norway Marine Pollution InitiativeDuring the visit of the Norwegian Prime Minister’s visit to India in January, India and Norway had agreedto work more closely on oceans by signing a MoU and establishing the India-Norway Ocean Dialogue.Both the governments had agreed to share experiences and competence, and collaborate on efforts todevelop clean and healthy oceans, sustainable use of ocean resources and growth in the blue economy.The India-Norway Marine Pollution Initiative is the first Joint initiative and it takes forward thecommitments made under the MoU. The Joint initiative aims to combat marine pollution, which is one ofthe fastest growing environmental concerns.The India-Norway Marine Pollution Initiative will support local governments in implementing sustainablewaste management practices, develop systems for collecting and analyzing information about sources andscope of marine pollution and improve private sector investment.The initiative will also support beach clean-up efforts, awareness-raising campaigns and pilot projectusing plastic waste as fuel substitution for coal in cement production and developing frameworks fordeposit schemes.

sri vishnu charan | [email protected] |

https://t.me/PDF4Examshttps://t.me/IAS201819 https://t.me/PDF4Exams

https://t.me/TheHindu_Zone_official

Page 70: Current Affairs -February 1-15, 2019 · 15-02-2019  · 10, theme: ‘Sadak Suraksha-Jeevan Raksha’ National Testing Agency (NTA) launches mobile app through which students can

Current Affairs [PDF] -February 1-15, 2019

© 2019 GKToday | All Rights Reserved | https://www.gktoday.in 70

February 12, 2019Improving Research Ecosystem in India

The central government has launched the following schemes to improve the research ecosystem in India:Prime Minister’s Research FellowshipTo attract the bright and meritorious students into research within the country, the Prime Minister’sResearch Fellowship has been initiated. Students selected for the fellowship are given fellowship for 5years – Rs.70,000/- per month for the first two years, Rs. 75,000/- per month in the third year andRs.80,000 per month in 4th and 5th year with an annual research grant of Rs. 2 lakh.Setting up of Research ParksThe Government has approved setting up of Research Parks at IIT Kharagpur, IIT Bombay, IIT Delhi, IITGuwahati, IIT Kanpur, IIT Hyderabad, IIT Gandhinagar and IISc Banglore.Impacting research Innovation and Technology (IMPRINT)IMPRINT India scheme aims to provide solutions to the most relevant engineering challenges andtranslating knowledge into viable technology (products or processes) in ten selected technology domains.Uchhatar Avishkar Yojana (UAY)Uchhatar Avishkar Yojana has been launched to promote innovation of a higher order that directlyimpacts the needs of the Industry and thereby improves the competitive edge of Indian manufacturing.Smart India HackathonsSmart India Hackathons are organized on yearly basis since 2017 to find out digital solutions to variousproblems faced by the people and gives hands-on working and research opportunity for engineeringstudents.Institution’s Innovation Council (IIC)Institution’s Innovation Council (IIC) have been established in 960 Higher Educational Institutions (HEIs)for promoting the innovation ecosystem within their campuses.Trainee Teacher SchemeA Trainee Teacher Scheme has been initiated in NITs with the objective to attract, motivate and sponsorbest graduate engineers for part-time M.Tech and PhD programmes.Further schemes like Colleges with Potential for Excellence (CPE), University with Potential forExcellence(UPE), Centre with Potential for Excellence in Particular Areas(CPEPA), Special AssistanceProgramme (SAP), Major Research Project (MRP) have been initiated by the government to boost theresearch ecosystem in the country.

LAWASIA Human Rights Conference 2019The first LAWASIA Human Rights Conference was organised by LAWASIA, in association with the BarAssociation of India. The LAWASIA Human Rights Conference 2019 was held between 9 – 10 February2019.The conference aims to provide a unique opportunity for lawyers and associated professional members toexchange insights and expertise on topics of significant importance to all.The theme of the LAWASIA Human Rights Conference 2019 was “State Power, Business and HumanRights: Contemporary Challenges”. The conference explored a wide range of human rights issues ofrelevance in the Asia Pacific region, including:

Gender, Sexuality and Human Rights.New Technologies, Privacy and Mass Surveillance.Experiences and Challenges of Human Rights Lawyers.Aggressive Policing and Human Rights.Freedom of the Press in the Digital Age.Cross-border Migration & its Conflicts.Climate Change, Water Conflicts & Human Rights.Business and Human Rights.

LAWASIALAWASIA is a regional association of lawyers, judges, jurists and legal organisations and it advocates forthe interests and concerns of the Asia Pacific legal profession.LAWASIA provides a platform to promote the cross-jurisdictional exchange of legal knowledge; as a voice

sri vishnu charan | [email protected] |

https://t.me/PDF4Examshttps://t.me/IAS201819 https://t.me/PDF4Exams

https://t.me/TheHindu_Zone_official

Page 71: Current Affairs -February 1-15, 2019 · 15-02-2019  · 10, theme: ‘Sadak Suraksha-Jeevan Raksha’ National Testing Agency (NTA) launches mobile app through which students can

Current Affairs [PDF] -February 1-15, 2019

© 2019 GKToday | All Rights Reserved | https://www.gktoday.in 71

of the legal profession; and as a conduit for encouraging adherence to mutually-held principles of the ruleof law, professional integrity and the protection of human rights.LAWASIA’s central aim is to strengthen professional and business relations among law associations,lawyers, the private sector and government representatives in the Asia Pacific.

Himachal’s First Mega Food Park inaugurated in UnaThe Union Minister of Food Processing Harsimrat Kaur Badal inaugurated the Cremica Food Park in Unawhich is the first mega food park of Himachal Pradesh.Cremica Food ParkThe important features of the Cremica Food Park are:

The mega food park set up at a cost of Rs 107.34 crore will benefit people of Una district and nearbydistricts of Kangra, Hamirpur and Bilaspur.The Mega Food park provides facilities such as bulk aseptic packaging (24 MT/ hour), Frozen storage(1,000 MT), Deep Freeze, Dry Warehouse, QC Laboratory and other food processing facilities.The park has a common administrative block for office and other uses by the entrepreneurs and 3Primary Processing Centers(PPC) at Solan, Mandi, and Kangra having facilities for primaryprocessing and storage near the farms.The mega food park at Una will leverage an additional investment of about Rs 250 crores in 25-30food processing units in the park and generate a turnover of about Rs 450-500 crore annuallyThe mega food park will also provide direct and indirect employment to 5,000 persons and benefitabout 25,000 farmers.

Mega Food Park SchemeThe Ministry of Food Processing is implementing Mega Food Park Scheme to give a major boost to thefood processing sector by adding value and reducing food wastage at each stage of the supply chain with aparticular focus on perishables.These mega food parks create modern infrastructure facilities for food processing along the value chainfrom farm to market along with strong forward and backward linkages through a cluster-based approach.

National Gas GridThe Union Government has envisaged the development of the National Gas Grid. The government isimplementing various projects of about 14,239 Km gas pipelines which would add to the existing 16,788Km natural gas pipeline.Authorisation AuthorityUnder the Petroleum and Natural Gas Regulatory Board Act, 2006 Petroleum and Natural Gas RegulatoryBoard (PNGRB) has been recognised as the authority to grant authorization to the entities for thedevelopment of City Gas Distribution (CGD) network in Geographical Areas.Petroleum and Natural Gas Regulatory Board identifies the geographical areas for authorizing thedevelopment of the CGD network in synchronization with the development of natural gas pipelineconnectivity/ natural gas availability.Objectives of the National Gas GridThe objectives of the National Gas Grid are:

To remove regional imbalance within the country with regard to access for natural gas and provideclean and green fuel throughout the country.To connect gas sources to major demand centres and ensure availability of gas to consumers invarious sectors.Development of City Gas Distribution Networks in various cities for the supply of CNG and PNG.

The National Gas Grid together with providing gas connections to households will provide betterinfrastructure for automobiles using gas. The National Gas Grid will also aid in renewing of the fertilizersector and also give a boost to the Power and Automotive sector.

Defence Innovation HubsThe Innovations for Defence Excellence (iDEX) framework under the Defence Ministry is envisagingsetting up of independent Defence Innovation Hubs (DIHs) where innovators can get information aboutneeds and feedback from the Armed Services directly and create solutions for India’s major defence

sri vishnu charan | [email protected] |

https://t.me/PDF4Examshttps://t.me/IAS201819 https://t.me/PDF4Exams

https://t.me/TheHindu_Zone_official

Page 72: Current Affairs -February 1-15, 2019 · 15-02-2019  · 10, theme: ‘Sadak Suraksha-Jeevan Raksha’ National Testing Agency (NTA) launches mobile app through which students can

Current Affairs [PDF] -February 1-15, 2019

© 2019 GKToday | All Rights Reserved | https://www.gktoday.in 72

platforms. Defence Innovation Hubs are aimed at attracting more innovators to work for the defencesector in India.Guidelines for Setting up of Defence Innovation HubsThe Framework to Fund Defence Innovation Hubs under iDEX has been approved by the Board of DefenceInnovation Organisation (DIO). The framework prescribes the following minimum criterion for setting upDefence Innovation Hubs: Any Central Government recognized Incubator including but not limited to:

Department of Science and Technology (DST) recognized Incubators.Atal Innovation Mission, NITIAayog created Atal Incubation Centers (AICs) and EstablishedIncubation Centers (EICs).Ministry of MSME recognized incubators.Any other incubator recognized or funded through any Central government scheme.The incubator located in districts mentioned in the list of SME clusters hosted by the Ministry ofMSME in collaboration with UNIDO.Incubator / Hub promoted by local industry associations.

The Defence Innovation Organisation set up under iDEX has announced setting up of two DIHs in TamilNadu (Coimbatore) and Maharashtra (Nashik).Defence Innovation OrganisationDefence Innovation Organisation is a not for profit company under section 8 of the Companies Act. TheDefence Innovation Organisation is funded by Hindustan Aeronautics Limited (HAL) and BharatElectronics Limited (BEL). The headquarters of the Defence Innovation Organisation is located atBengaluru.Innovations for Defence Excellence (iDEX) schemeInnovations for Defence Excellence (iDEX) scheme aims to create an ecosystem to foster innovation andtechnology development in Defence and Aerospace by engaging Industries including MSMEs, Startups,Individual Innovators, R&D institutes and Academia and provide them grants/funding and other supportto carry out R&D which has good potential for future adoption for Indian defence and aerospace needs.The Innovations for Defence Excellence (iDEX) functions as the executive arm of the Defence InnovationOrganisation.

Indian Historian Sanjay Subrahmanyam wins the Dan David PrizeIndian historian Sanjay Subrahmanyam has won prestigious Dan David Prize of Israel for the year 2019 forhis work on inter-cultural encounters between Asians, Europeans and people of North and South Americaduring the early modern era.Sanjay Subrahmanyam shares the award in the category of ‘past time dimension’ with Prof KennethPomeranz of the University of Chicago.Dan David PrizeThe Dan David Prize is a joint international award given by the Dan David Foundation headquartered atTel Aviv University. The award was instituted by late Dan David, an international businessman andphilanthropist who envisioned a project that would extend beyond traditional academic categorisations.The Dan David Prize is awarded under three categories namely:

Past category is generally drawn from the fields of history, archaeology, palaeontology andbiography.Present from arts, media, policy and economics.Future from one of the exact or natural sciences.

For the Year 2019, the awardees were chosen from the three categories for their work:Macrohistory: Past categoryDefending democracy: PresentCombating climate change: Future

The award in the “Present” category for “Defending Democracy” has been won by Reporters withoutBorders and Prof Michael Ignatieff.The prize in the “future” category for combating climate change has been won by Christina Figueres, aCosta Rican diplomat

sri vishnu charan | [email protected] |

https://t.me/PDF4Examshttps://t.me/IAS201819 https://t.me/PDF4Exams

https://t.me/TheHindu_Zone_official

Page 73: Current Affairs -February 1-15, 2019 · 15-02-2019  · 10, theme: ‘Sadak Suraksha-Jeevan Raksha’ National Testing Agency (NTA) launches mobile app through which students can

Current Affairs [PDF] -February 1-15, 2019

© 2019 GKToday | All Rights Reserved | https://www.gktoday.in 73

To encourage and foster new generations of scholars, the awardees are required to donate 10 per cent oftheir prize money towards scholarships for graduate or post-graduate researchers in their respectivefields.

February 13, 2019Environment Ministry Releases ‘India – Spearheading Climate Solutions’

Union Minister of Environment, Forest and Climate Change has released a publication titled “India –Spearheading Climate Solutions” on 12 February, 2019. This document mentions India’s key actions takentowards combating and adapting to climate change.India’s Key Actions Towards Combating and Adapting Climate Change

India’s National Action Plan on Climate Change (NAPCC) that has eight major missions on Solar,Enhanced Energy Efficiency, Sustainable Habitat, Water, Sustaining the Himalayan Ecosystem,Green India, Sustainable Agriculture and Strategic Knowledge on Climate Change.International Solar Alliance (ISA) – It is an alliance of 121 countries, most of them sunshine countries(i.e. lying either completely or partly between the Tropic of Cancer and the Tropic of Capricorn). Itwas founded in 2015 at Paris and was initiated by India. Membership now has been extended to allthe members of the United NationsState Action Plan on Climate Change (SAPCC)FAME Scheme – It stands for Faster Adoption and Manufacturing of (Hybrid &) Electric Vehicles(FAME-India) Scheme, which was launched in 2015 under the National Electric Mobility Mission Plan(NEMMP). It aims to create the infrastructure for and promote e-mobility.Atal Mission for Rejuvenation & Urban Transformation (AMRUT) –The scheme was launched inthe year 2015 and is solely aimed for Smart CitiesPradhan Mantri Ujjwala Yojana – The scheme was launched in 2016. It aims for providing access toclean cooking fuelUJALA scheme – It stands for Unnat Jeevan by Affordable LEDs and Appliances for All and wasstarted in 2015. Its aim is to embrace energy efficient LED bulbsSwachh Bharat Mission – It is a nationwide campaign from 2014 to 2019 aiming to clean up India.National Adaptation Fund for Climate Change (NAFCC) – It was launched in 2015-16 and aims tosupport robust adaptation techniques to reduce adverse effects of climate change.

Other Initiatives Taken by GovernmentApart from the above, Government of India has taken an ambitious goal of generating 175 GW ofrenewable energy by 2022. India will leapfrog from Bharat Stage -IV to Bharat Stage-VI emission norms byApril 2020. India’s renewable energy capacity stands at more than 74 GW today including about 25 GWfrom solar. Further, India submitted its Second Biennial Update Report (BUR) to UNFCCC (United NationsFramework Convention on Climate Change) in December 2018 as per the reporting obligations under theconvention, which stated that India’s GDP came down by 21% between 2005 & 2014. It also stated thatIndia’s climate-goal for pre-2020 is well on track.

Life Size Portrait of Atal Bihari Vajpayee Unveiled in ParliamentThe President of India Ram Nath Kovind has unveiled the life size portrait of former Prime Minister AtalBihari Vajpayee at the Central Hall of Parliament on 12th February 2018. There were many other leaderspresent including Vice President Venkaiah Naidu, Prime Minister Narendra Modi, Lok Sabha SpeakerSumitra Mahajan, Leader of Opposition in the Rajya Sabha Ghulam Nabi Azad and many other UnionMinisters.Key Facts About Atal Bihari Vajpayee

Late Atal Bihari Vajpayee was born on December 25, 1924, in Gwalior in Madhya Pradesh. He passedaway on August 16, 2018.He was a member of the Indian Parliament for over four decades He was one of the foundingmembers of the Bharatiya Jan Sangh. He worked as its President from 1968 to 1972 and thenresigned in 1978. He became the first President of the BJP in 1980.He served as the Minister of External Affairs in the Cabinet of Prime Minister Morarji Desai from 26

sri vishnu charan | [email protected] |

https://t.me/PDF4Examshttps://t.me/IAS201819 https://t.me/PDF4Exams

https://t.me/TheHindu_Zone_official

Page 74: Current Affairs -February 1-15, 2019 · 15-02-2019  · 10, theme: ‘Sadak Suraksha-Jeevan Raksha’ National Testing Agency (NTA) launches mobile app through which students can

Current Affairs [PDF] -February 1-15, 2019

© 2019 GKToday | All Rights Reserved | https://www.gktoday.in 74

March 1977 to 28 July 1979He served three terms as the Prime Minister of India – first term was for a term of 13 days in 1996,second was for a period of 13 months from 1998 to 1999 and the third was for a full term from 1999 to2004He was the first non-Indian National Congress leader to serve a full-term as the Indian PrimeMinister.Under his tenure, India had conducted the Pokhran-II nuclear tests in 1998. The Kargil War was alsofought during his tenure.He had always given his best efforts to normalize and better relations with Pakistan.He was awarded the Bharat Ratna Award by the then Indian President Pranab Mukherjee in 2015.His birthday is observed as the Good Governance DayHe is also known for his poetry.PM Modi had released a commemorative coin in memory of Mr. Vajpayee on December 24, 2018 onthe occasion of his 94th Birthday

About the PortraitThe Parliament’s Portrait Committee chaired by Sumitra Mahajan decided to install the portrait in ameeting on December 18. Portraits of former PMs Jawaharlal Nehru, Indira Gandhi and Rajiv Gandhi alsoadorn the walls of Central Hall.

National Productivity Week: February 12-18, 2019National Productivity Council (NPC) is celebrating National Productivity week from February 12th to 18th,2019. This year, the theme is “Circular Economy for Productivity & Sustainability”. This theme will help NPCto meet its zero waste goal.The above theme is decided by looking into the different national objectives ofsustainable growth and commitments with respect to different sectors. Themes are always used as theyhelp in information dissemination and aid in the adoption of new processes for growth andmodernization.

What is Circular Economy?Circular Economy is based on the model of Make, Use and Return. It controls the finite stocks and reuses them byrecycling and thus balances the resource flow. The key advantages of Circular Economy are as follows:

It will increase productivity.1.It will create more jobs.2.It will reduce carbon emission thus preserving valuable raw materials.3.

Circular Economy system increases the product life span through improved design and servicing and relocatingwastes from the end of the supply chain to the beginning.

Other Initiatives taken by NPC to accomplish the goal of converting linear economy to circular economyare:

New Solid waste management rules to fulfill the Swacch Bharat objectives.1.Trained energy professionals in various industrial sectors.2.To meet 50% of NDC target, 18640 certified energy auditors have been prepared.3.Through 10-15 annual workshops from the last 12 years, 8000 boiler supervisors have been trained4.who are experts on efficiency and safety issues.

About the National Productivity CouncilNational Productivity Council is an autonomous registered society under DIPP (Department forPromotion of Industry and Internal Trade), Ministry of Commerce & Industry. It was established in theyear 1985. Its core function is to promote and adopt the new and improved culture of productivity in India.

Siddhartha Lal Named EY Entrepreneur of the Year 2018 – IndiaThe CEO and Managing director of Eicher Motors, Siddhartha Lal has been named the EY Entrepreneur ofthe Year for 2018 on 12th February 2019 in Mumbai. He was responsible for a huge turnaround of theEicher Motors after he had taken over the reins in 2006. In this process he has also resurrected themotorcycle brand Royal Enfield in India. The Chairman of Wipro Mr. Azim Premji was awarded Lifetime

sri vishnu charan | [email protected] |

https://t.me/PDF4Examshttps://t.me/IAS201819 https://t.me/PDF4Exams

https://t.me/TheHindu_Zone_official

Page 75: Current Affairs -February 1-15, 2019 · 15-02-2019  · 10, theme: ‘Sadak Suraksha-Jeevan Raksha’ National Testing Agency (NTA) launches mobile app through which students can

Current Affairs [PDF] -February 1-15, 2019

© 2019 GKToday | All Rights Reserved | https://www.gktoday.in 75

Achievement Award for his contribution in the Indian information technology industry and hisphilanthropic works.About EY Entrepreneur of the Year 2018 (EOY)The EOY is held annually in India to honour entrepreneurs who emerged as Builders of a better India withtheir creative ideas and exceptional enterprises. This year was the 20th year of EOY India Awards program.Mr. Lal will represent India at the EY World Entrepreneur of the Year Award (WEOY) in Monte Carlo from6 – 8 June 2019. There were seven other categories of EY Entrepreneur of the Year 2018 winners –

Byju Raveendran, founder and CEO, Think & Learn (start-up)1.Kishore Biyani, founder and CEO, Future Group (business transformation)2.Nirmal K Minda, CMD, Minda Industries (manufacturing)3.Ritesh Agarwal, founder and CEO, OYO Hotels and Homes (services)4.Sanjay Agarwal, MD and CEO, AU Small Finance Bank (financial services)5.Binish Chudgar, vice CMD, Intas Pharmaceuticals (Life Sciences & Healthcare)6.Rajesh Mehra, promoter and director, Jaquar group (consumer products and retails).7.Titan Company Limited MD Bhaskar Bhat won the Entrepreneurial CEO award.The winners were chosen by a eight-member jury led by Uday Kotak.The winners having unicorns and market disruptors have a revenue of more than ₹1,28,000 croreand employ over 2,50,000 people worldwide.

About EY World Entrepreneur of the Year Award (WEOY)The EY World Entrepreneur Of The Year Forum is organized each year and hosts a unique gathering ofdisruptive entrepreneurs and global business leaders from more than 50 countries to find innovativeapproaches to accelerate growth. This program was started from US since the year 1986. Nominations aredone regionally in most countries. Regional winners are then eligible for their country’s EY EntrepreneurOf The Year National Award. These country winners will then be eligible for the EY World EntrepreneurOf The Year Award. The event is held every year in Monaco.

What is a Unicorn Entrepreneur?A privately held startup company valued at over $1 billion is called a Unicorn. The coin was termed in 2013 by the byventure capitalist Aileen Lee.What is a Market Disruptor?An innovation that creates a totally new market and value network, thus disrupting the existing market and valuenetwork by displacing established market-leading firms, products, and alliances is called Disruptive Innovation. Theperson responsible for it is called the Market Disruptor.

About EYEY (Ernst and Young) is one of the largest professional services firms in the world founded in 1989 that isheadquartered in London, England, United Kingdom. Its CEO is Mr. Mark Weinberger It is one of the BigFour Accounting firms – Deloitte, EY, KPMG and PricewaterhouseCoopers (PwC)

Multinational Cobra Gold Military Exercise Starts in ThailandThe United States and Thailand are hosting the multi-nation Cobra military exercise this year. Theexercise is taking place in the northern Thai province of Phitsanulok. 10000 personnel, 29 nations aretaking part as participants or observers.About Cobra Gold Exercise

It is one of the Asia-Pacific region’s largest multinational military exercises that is held in Thailandevery year.It was first held in 1982 and its headquarters is in Bangkok, Thailand.This is a Thai-American initiative with an aim to improve coordination between the armed forces.India joined this exercise for the first time in 2016 while China was admitted for the first time in2015 but was only allowed to participate in humanitarian assistance training.This exercise improved coordination among the military in response to natural disaster. Some of theexamples are the Indian Ocean tsunami 2004, 2011 Tōhoku tsunami, Typhoon Haiya etc.This year it is the 38th edition of this exercise and it will see active participation from Singapore,

sri vishnu charan | [email protected] |

https://t.me/PDF4Examshttps://t.me/IAS201819 https://t.me/PDF4Exams

https://t.me/TheHindu_Zone_official

Page 76: Current Affairs -February 1-15, 2019 · 15-02-2019  · 10, theme: ‘Sadak Suraksha-Jeevan Raksha’ National Testing Agency (NTA) launches mobile app through which students can

Current Affairs [PDF] -February 1-15, 2019

© 2019 GKToday | All Rights Reserved | https://www.gktoday.in 76

Japan, China, India, Indonesia, Malaysia and South Korea, Thailand, and the United States.The exercise will include three components: military field training, humanitarian assistance, anddisaster relief training.India is participating in this exercise with a 14 member contingent.

What is Phitsanulok?It is the capital of Phitsanulok Province and is located in the north of Thailand is on the banks of the Nanriver. It is one of the oldest cities of Thailand.

Indian Army holds annual ‘Exercise Topchi’Indian Army on 12th February 2019 showcased its artillery firepower by using ultra light Howitzers andindigenous Swathi weapon-locating radar at the annual “Exercise Topchi”. The exercise was held at DeolaliCamp near Nashik. The aviation and surveillance capabilities were also showcased.About the Exercise Topchi 2019There were a showcase of display of rockets, missiles, surveillance and target acquisition radars, remotelypiloted aircraft and hi-tech equipments in addition to gun fire. The precision displayed while deliveringthe explosives in the target area stunned all those present. The ones who were present included Lt GenYVK Mohan, Commandant of Defence Service Staff College (DSSC), Wellington; Lt Gen R S Salaria,Commandant, School of Artillery and Colonel Commandant Regiment of Artillery and other Army officials.What is the Ultra Light Howitzer M777?It is a towed 155 mm artillery piece that is used by the ground forces of Australia, Canada, India and SaudiArabia. It is manufactured by BAE Systems’ Global Combat Systems division (London, England). It wasinducted in the Indian Army on November 9, 2018.What is the Swathi Weapon Locating Radar?Swathi Weapon Locating Radar is a mobile artillery locating phased array radar developed by India. It is acounter-battery radar designed to detect and track incoming artillery and rocket fire to determine thepoint of origin for Counter-battery fire. It has been developed by DRDO’s Bangalore based laboratory,LRDE and the Government owned Bharat Electronics Limited (BEL)What is the Deolali Camp?Deolali is a small hill-station in the Nashik district of Maharashtra. It has one of the oldest military centresin India. It was established in 1869 and is situated on the banks of the River Darna. The prestigious Schoolof Artillery is located in Deolali.

National Cancer Institute Inaugurated in HaryanaPrime Minister, Narendra Modi inaugurated National Cancer Institute (NIC) at Badhsa in Jhajjar districtof Haryana.Prime Minister also inaugurated an ESIC Medical College and Hospital in Faridabad. It is the firstEmployees’ State Insurance Corporation (ESIC) Medical College and hospital in North India. It has 510beds and includes the facilities of surgical oncology, radiation oncology, medical oncology, anaesthesia,palliative care, and nuclear medicine.

Fact BoxThere are 6 existing ESIC medical colleges at Kolkata, Faridabad, Hyderabad, Chennai, Bengaluru, and Gulbarga.

About Employees’ State Insurance Corporation (ESIC)It was established under the Employees’ State Insurance Act, 1948(ESI Act) by the Ministry of Labourand Employment, Government of India.It is an autonomous organization and it manages the funds of all employees earning ₹21,000 or lessper month.Employees’ State Insurance includes self-financing social security and health insurance scheme forIndian workers.ESIC is operating many medical, dental, nursing and paramedical schools in India.

What is the purpose of ESIC Hospital?Workers are generally exposed to sickness related to their occupation; they may even suffer fromtemporary or permanent disablement etc, which results in loss of their wages. Institutions like this negate

sri vishnu charan | [email protected] |

https://t.me/PDF4Examshttps://t.me/IAS201819 https://t.me/PDF4Exams

https://t.me/TheHindu_Zone_official

Page 77: Current Affairs -February 1-15, 2019 · 15-02-2019  · 10, theme: ‘Sadak Suraksha-Jeevan Raksha’ National Testing Agency (NTA) launches mobile app through which students can

Current Affairs [PDF] -February 1-15, 2019

© 2019 GKToday | All Rights Reserved | https://www.gktoday.in 77

the resulting physical or financial distress in such contingencies and uphold human dignity in times ofcrises.Cancer Incidence in HaryanaAccording to 2018 survey, 38% of deaths due to cancer occur in Haryana. Some of the identified reasonsare:

Rapid industrializationIndustries are disposing waste directly into the river without undergoing any sewage treatmentprocess. This gives rise to air and water-borne diseases among which skin infection and cancer aremost common.Improper sanitationExcessive use of fertilizers

The contamination due to the reasons listed above pollutes water by causing eutrophication (thephenomenon of a body of water becoming overly enriched with minerals and nutrients which induceexcessive growth of plants and algae) and the consumption of this polluted water is resulting in cancer.

February 14, 2019Egypt’s el-Sissi Elected New Chairman of African Union

The President of Egypt Abdel-Fattah el-Sissi took over the chairmanship of African Union from PaulKagame, President of Rwanda. African Union chairmen set agendas of issues to be addressed during theirtenure of one-year. El-Sissi is expected to concentrate on security and financial reform.African UnionThe Heads of State and Government of the Organisation of African Unity issued a Declaration, SirteDeclaration calling for the establishment of an African Union on 9.9.1999. The Union was launched on 26May 2001 in Addis Ababa, Ethiopia and was launched on 9 July 2002 in South Africa and the African Unionreplaced the Organisation of African Unity (OAU), established on 25 May 1963.

Africa DayAfrica Day (formerly African Freedom Day and African Liberation Day) is the annual commemoration of thefoundation of the Organisation of African Unity (OAU) (now known as the African Union) on 25 May 1963.

The African Union was established for accelerating the process of integration in the continent to enable itplay its rightful role in the global economy while addressing multifaceted social, economic and politicalproblems compounded as they are by certain negative aspects of globalisation. All UN memberstates based in Africa and on African waters are members of the AU

Registration of Marriage of Non-Resident Indian Bill, 2019The government has introduced the Registration of Marriage of Non-Resident Indian Bill, 2019 in theParliament. The bill was introduced by the Minister of External Affairs Sushma Swaraj in the Rajya Sabha.Features of the BillThe Bill aims to arrest the cases of Indian women being trapped in fraudulent marriages with non-resident Indians (NRI). The important features of the bill are:

The bill mandates the Non-Resident Indians who marry an Indian citizen or a fellow NRI abroad tocompulsorily register their marriages within 30 days.Failure to register their marriage would face the prospects of their passports being impounded oreven revoked.The bill allows courts to attach movable and immovable properties of NRIs who are declared“proclaimed offenders” for failing to appear before the law.The bill allows courts to send summonses and warrants to the accused through a speciallydesignated website to be hosted by the Ministry of External Affairs.The bill amends the Passport Act and the Code of Criminal Procedure to allow passport authoritiesto revoke or impound passport or travel documents of the accused, and allow courts to attach theirproperties.The bill clearly states that if an NRI marries an Indian citizen here, the marriage has to be registered

sri vishnu charan | [email protected] |

https://t.me/PDF4Examshttps://t.me/IAS201819 https://t.me/PDF4Exams

https://t.me/TheHindu_Zone_official

Page 78: Current Affairs -February 1-15, 2019 · 15-02-2019  · 10, theme: ‘Sadak Suraksha-Jeevan Raksha’ National Testing Agency (NTA) launches mobile app through which students can

Current Affairs [PDF] -February 1-15, 2019

© 2019 GKToday | All Rights Reserved | https://www.gktoday.in 78

as per local laws and if the marriage takes place abroad, it has to be registered with designatedofficers to be appointed in foreign countries.

The Bill proposes to offer greater protection to Indian women married to NRIs and serve as a deterrent toNRIs against harassment of their spouses.

Ghumot to be declared Goa’s heritage musical instrumentThe Art and Culture Minister of Goa, Govind Gaude has announced that Ghumot, an indigenous earthendrum will soon be notified as a heritage instrument of Goa. Ghumot is an indigenous earthen drumfashioned as a designed clay pot, with the skin of the monitor lizard stretched taut across the pot’s mouth,forming a drumhead. Ghumot is a percussion instrument widely played during Ganesh Chaturthi Aarties.

Fact BoxGoa which is the former Portuguese colony presented Ghumot as a gift to the Portuguese Prime Minister AnotnioCosta during his visit to India in 2017. Portuguese Prime Minister Anotnio Costa is is of Goan origin.

Why Ghumot was banned?The skin of monitor lizard is one of the key components of the Ghumot. Since the monitor lizard wasclassified as an endangered wildlife species and was listed in Schedule I of the Wildlife (Protection) Act1972, Ghumot was banned by the Forest Department of Goa.In recent years attempts were made to revive the heritage instrument of Goa by replacing monitor lizardskin with the skin of a she-goat. This revival had spiked the interests about the instrument.Taking forward the efforts to conserve the heritage musical instrument the government of Goa is takingsteps to declare Ghumot as the heritage musical instrument of Goa.

Rajasthan Assembly Passes Bill to Scrap Minimum Education Qualification for Civic PollsThe Legislative Assembly of Rajasthan passed the Rajasthan Panchayati Raj (Amendment) Bill, 2019 andthe Rajasthan Municipality (Amendment) Bill, 2019 to end the minimum education criterion for panchayatand civic polls candidates.What was the education criterion proposed?As per the Rajasthan Panchayati Raj Amendment Bill 2015, Candidates were:

Required to have passed Class X for contesting municipal elections, Class VIII for contestingpanchayat polls for the post of a sarpanch, and Class X for contesting Zila Parishad or PanchayatSamiti elections.Construction of toilets in their homes as mandatory for contesting polls to the panchayati rajinstitutions in the state.

Even the Supreme Court had upheld the constitutional validity of a similar law in Haryana whichmandated education qualification for candidacy in local polls. The Supreme Court had said that “it is onlyeducation which gives a human being the power to discriminate between right and wrong, good and bad”.Why the government scrapped the criterion?In the run-up to the assembly elections in 2018, the Congress Party had included in its manifesto that itwould abolish the minimum education criterion for panchayat and civic polls candidates if it comes topower.Defending the bill the government stated in assembly that Sarpanches awarded by President, VicePresident were declared ineligible due to the provisions in the previous Act. The Act was against the basictenants of the Constitution and society cannot be divided on the basis of education.The government even stated that several cases had come up where people contested the election on thebasis of fake certificates. The minimum education qualification created two sections in the society and theilliterate people had started feeling inferior.

PM Modi serves the 3 Billionth Meal of Akshaya PatraPrime Minister Narendra Modi served the three billionth meal of Akshaya Patra, an NGO that works withthe government on Mid-Day Meal Scheme. The Prime Minister Served the food to about 20 schoolchildrenwho are the beneficiaries of the mid-day meal scheme at the Vrindavan Chandrodaya Mandir campus inthe holy city of Vrindavan. Prime Minister also unveiled a ceremonial plaque to mark the serving of “3rdbillionth meal” by Akshaya Patra Foundation.

sri vishnu charan | [email protected] |

https://t.me/PDF4Examshttps://t.me/IAS201819 https://t.me/PDF4Exams

https://t.me/TheHindu_Zone_official

Page 79: Current Affairs -February 1-15, 2019 · 15-02-2019  · 10, theme: ‘Sadak Suraksha-Jeevan Raksha’ National Testing Agency (NTA) launches mobile app through which students can

Current Affairs [PDF] -February 1-15, 2019

© 2019 GKToday | All Rights Reserved | https://www.gktoday.in 79

Akshaya Patra FoundationAkshaya Patra Foundation is a Bengaluru-based not-for-profit organisation Funded by InternationalSociety for Krishna Consciousness (ISKCON). Akshaya Patra Foundation is an implementation partner forthe governments Mid-Day Meal Scheme.Akshaya Patra Foundation has served meals to more than 1.76 million children in twelve states covering14,702 schools since its inception. In the year 2016, Akshaya Patra commemorated the serving of 2 billioncumulative meals in the presence of the then President of India Pranab Mukherjee.Akshaya Patra Foundation is the world’s largest not-for-profit run organisation serving wholesome foodto over 1.76 million children every school day from over 14,000 schools across 12 states in India.

MEA Organises Training Course for Govt officials from NepalThe Ministry of External Affairs had organised a training course for the officials of the Government ofNepal on ‘Global Perspectives on Public Financial Management’ at the Institute of Government Accountsand Finance (INGAF) in New Delhi. The training programme was part of the Indian Technical andEconomic Cooperation (ITEC) programme.It was a special tailor-made training programme on Public Finance Management designed at the requestand requirement of the Ministry of Finance of Nepal for a total of 62 officials.The training was aimed at enhancing the capabilities of the officials in global practices dealing with e-payment system, accounting & cash management, public borrowing system, accounting of foreignloans/grants and repayments, pension payment system, internal audit, fiscal responsibility and budgetmanagement, etc.Indian Technical and Economic Cooperation (ITEC)The Indian Technical and Economic Cooperation (ITEC) Programme was instituted on 15 September 1964as a bilateral programme for providing assistance from the Government of India. The ITEC programmewas constituted based on the underlying belief that it was necessary to establish relations of mutualconcern and inter-dependence based not only on commonly held ideals and aspirations but also on solideconomic foundations. Technical and economic cooperation was considered to be one of the essentialfunctions of an integrated and imaginative foreign policy.The ITEC Programme is fully funded by the Government of India. Due to the various successful initiativesof ITEC, there is now a visible and growing awareness among other countries about the competence ofIndia as a provider of technical know-how and expertise as well as training opportunities, consultancyservices and feasibility studies. As a result, these programmes are generating immense goodwill andsubstantive cooperation towards India among the developing countries.

Unani Day 2019February 11 every year is observed as the Unani Day. As part of the Unani Day celebrations, variousactivities including marathons, writathon, quiz competitions, public health talks, daily health tips for thegeneral public and workshops were organised across the country.National Conference on Unani Medicine for Public HealthTo commemorate the Unani Day 2019 a two day National Conference on Unani Medicine for Public Healthwas held at New Delhi.The two-day conference had a dedicated session on lifestyle disorders and their management, regimenttherapy, mother and child care, geriatric care, integration and mainstreaming of Unani Medicine in publichealth and globalisation of Unani Medicine. Also, a ceremony was held for the distribution of AYUSHAwards for Unani Medicine during the conference.The conference shed light on the important role played by Unani Medicine in public health especially incombating NCDs, lifestyle disorders and various chronic diseases by providing cost-effective Unanitreatment and quality products for patient healthcare.The conference was hosted to provide an excellent platform for sharing of knowledge and development oflinkages between institutes and organisations engaged in the promotion and development of UnaniMedicine.Why February 11 is celebrated as Unani Day?February 11 marks the birth anniversary of Hakim Ajmal Khan. Hakim Ajmal Khan took great interest in

sri vishnu charan | [email protected] |

https://t.me/PDF4Examshttps://t.me/IAS201819 https://t.me/PDF4Exams

https://t.me/TheHindu_Zone_official

Page 80: Current Affairs -February 1-15, 2019 · 15-02-2019  · 10, theme: ‘Sadak Suraksha-Jeevan Raksha’ National Testing Agency (NTA) launches mobile app through which students can

Current Affairs [PDF] -February 1-15, 2019

© 2019 GKToday | All Rights Reserved | https://www.gktoday.in 80

the expansion and development of the native system of Unani medicine and to that end built threeimportant institutions, the Central College in Delhi, the Hindustani Dawakhana and the Ayurvedic andUnani Tibbia College better known as Tibbia College, Delhi, which expanded research and practice in thefield and saved the Unani System of Medicine from extinction in India.His efforts infused a new force and life into an otherwise decaying Unani medical system under Britishrule. In recognition of the contribution of Hakim Ajmal Khan, the Ministry of AYUSH adopted 11thFebruary as Unani Day in 2017.

PETROTECH -2019Prime Minister Narendra Modi inaugurated the PETROTECH -2019 conference and called for responsiblepricing of crude oil that balances interests of both producers and consumers highlighting the need to haveuniversal assess to clean, the affordable and equitable supply of energy.PETROTECH -2019 conference is a three-day mega event held from 10 to 12 February 2019. Theconference was held at India Expo Centre, Greater Noida, Uttar Pradesh on 11th February 2019.In the backdrop of changing global energy basket, PETROTECH-2019 offered an excellent opportunity tohydrocarbon industry, their associates and academia to interact and debate the latest advances in thehydrocarbon sector, which can be leveraged for meeting global energy requirementsAlong with PETROTECH -2019 an exhibition over 13 country pavilions and around 750 exhibitors fromover 40 countries, with exclusive areas on the Make in India and Renewable Energy theme was organised.PETROTECHPETROTECH conference is a biennial International Oil & Gas Conference. The conference provides aplatform for national and international experts in the oil & gas industry to exchange views and shareknowledge, expertise, and experiences.The 13th edition of International Oil & Gas Conference and Exhibition, PETROTECH-2019 is beingorganized by Oil & Natural Gas Corporation Limited, under the aegis of Ministry of Petroleum and NaturalGas, Government of India.

2019 Grammy AwardsThe 61st GRAMMY Awards were held on February 10, 2019, at Staples Center in Los Angeles. The winnersare:

Record of the Year: THIS IS AMERICA.Album of the Year: GOLDEN HOUR.Song of The Year: THIS IS AMERICA.Best New Artist: DUA LIPA.Best Pop Solo Performance: JOANNE (WHERE DO YOU THINK YOU’RE GOIN’?)Best Pop Duo/Group Performance: SHALLOWBest Traditional Pop Vocal Album: MY WAYBest Pop Vocal Album: SWEETENERBest Dance Recording: ELECTRICITYBest Dance/Electronic Album: WOMAN WORLDWIDE

Grammy AwardsGrammy Awards are presented by the Recording Academy to recognize achievements in the musicindustry. The Grammys held for the first time on May 4, 1959, are the second of the Big Three major musicawards held annually (between the American Music Awards in the Fall, and the Billboard Music Awards inthe Summer).

Guidelines for setting up of Dealer Owned Dealer Operated CNG stationsOn the Sidelines of the Petrotech 2019, the Union Minister for Petroleum and Natural Gas, ShriDharmendra Pradhan released the guidelines for setting up of Dealer Owned Dealer Operated (DODO)CNG stations in India.The City Gas Distribution (CGD) entities run by central public sector undertakings will execute theseguidelines for the setting up of CNG stations on DODO model in the country.New GuidelinesThe features of these new guidelines are:

sri vishnu charan | [email protected] |

https://t.me/PDF4Examshttps://t.me/IAS201819 https://t.me/PDF4Exams

https://t.me/TheHindu_Zone_official

Page 81: Current Affairs -February 1-15, 2019 · 15-02-2019  · 10, theme: ‘Sadak Suraksha-Jeevan Raksha’ National Testing Agency (NTA) launches mobile app through which students can

Current Affairs [PDF] -February 1-15, 2019

© 2019 GKToday | All Rights Reserved | https://www.gktoday.in 81

Initially, these guidelines would be implemented in over 150 districts of the country.Entire plot owned by a private owner shall be developed exclusively for setting up of CNGstation and allied commercial activities at the discretion of CGD entity.The plot owner applicant desirous of setting up CNG station will have to enter into a long termagreement with CGD entity.The CNG equipment (compressor/ cascade/ dispenser, etc.) will be installed and commissioned byCGD entity while adhering to all the statutory rules/guidelines related to fire and safety.The dealer is required to arrange all the permissions to set up CNG Station including change of landuse, necessary clearance, license, etc. at own expense and cost.The dealer shall be paid commission by the concerned CGD entity as per the sales achieved based onthe fixed dealer commission.

Objectives of the New GuidelinesThe new guidelines will act as an advisory for bringing the uniform system in line with DODO policywhich will further help in bringing hygiene and cleanliness in the environment.The guidelines aim to encourage those with possession of land to join hands with CGD entities in theirendeavour to provide clean fuel for the transport sector.Together with reducing the cost of setting up CNG filling stations for CGD entities the guidelines willprovide landowners and investors to become partners in this green drive.The new guidelines are expected to give a fillip to the CNG business in the country as the easy availabilityof CNG across the country shall encourage more people to switch to eco-friendly and economical CNG.

BAFTA FILM AWARDS 2019The British Academy of Film and Television Arts, BAFTA awards are presented in an annual award showhosted by the British Academy of Film and Television Arts (BAFTA) to honour the best British andinternational contributions to film. The BAFTA FILM AWARDS 2019 have been announced. The awardeesunder the various categories are listed below:

Best Film: RomaOutstanding British Film: The FavouriteLeading Actress: Olivia Colman, The FavouriteLeading Actor: Rami Malek, Bohemian RhapsodySupporting Actress: Rachel Weisz, The FavouriteSupporting Actor: Mahershala Ali, Green BookDirector: Alfonso Cuarón, RomaCostume Design: The FavouriteMake Up & Hair: The FavouriteOriginal Music: A Star Is BornOutstanding Debut by a British Writer, Director or Producer: Michael Pearce and Lauren Dark, BeastAnimated Film: Spider-Man: Into the Spider-VerseAdapted Screenplay: BlacKkKlansmanBritish Short Animation: RoughhouseBritish Short Film: 73 CowsCinematography: RomaDocumentary: Free SoloEditing: ViceFilm Not in the English Language: RomaOriginal Screenplay: The FavouriteProduction Design: The FavouriteSound: Bohemian RhapsodySpecial Visual Effects: Black PantherEE Rising Star Award: Letitia Wright, Black Panther/Infinity War (public vote)

The Special Award for Outstanding British Contribution to Cinema was given to Elizabeth Karlsen and

sri vishnu charan | [email protected] |

https://t.me/PDF4Examshttps://t.me/IAS201819 https://t.me/PDF4Exams

https://t.me/TheHindu_Zone_official

Page 82: Current Affairs -February 1-15, 2019 · 15-02-2019  · 10, theme: ‘Sadak Suraksha-Jeevan Raksha’ National Testing Agency (NTA) launches mobile app through which students can

Current Affairs [PDF] -February 1-15, 2019

© 2019 GKToday | All Rights Reserved | https://www.gktoday.in 82

Stephen Woolley and the Fellowship honor was presented to editor Thelma Schoonmaker.Dera Baba Nanak Land Post Designated as Immigration Centre for visiting Kartarpur Sahib

The notification issued by the Union Home ministry has designated Dera Baba Nanak land post located inPunjab’s Gurdaspur district as an authorised immigration checkpoint to exit and enter for visitingKartarpur Sahib Gurdwara in Pakistan.Kartarpur Sahib CorridorThe Kartarpur Sahib Corridor connects Kartarpur Gurdwara in Pakistan with Dera Baba Nanak shrine inIndia’s Gurdaspur district. The Kartarpur Sahib Gurdwara is located on the banks of the Ravi River, aboutthree-four km from the border in Pakistan.The proposed corridor was institutionalised to facilitate visa-free movement of Indian Sikh pilgrimsbetween Kartarpur Gurdwara in Pakistan and Dera Baba Nanak shrine in India. The visitors are requiredto obtain a permit to visit Kartarpur Sahib where the Guru Nanak Dev, the founder of Sikhism, had spentmore than 18 years of his life. Dera Baba Nanak shrine was established in 1522 by Guru Nanak Dev.As part of the trust-building exercise, the Pakistan new government had proposed to open the KartarpurSahib Corridor. Works to facilitate the opening the corridor began with Vice President M Venkaiah Naidulaying the foundation stone at the Indian side November 28 and Pakistani Prime Minister ImranKhan laying the foundation stone at the Pakistani side on November 28.

World Employment and Social Outlook Trends 2019: Key FactsThe International Labour Organisation has released the annual World Employment and Social OutlookTrends for the year 2019. The important features of the report are:

Unemployment rates were anticipated to fall to 4.9% in 2019 and are expected to hold steady into2020.The number of jobless in real terms is estimated to rise from 172 million to 174 million in that time asthe labour market expands.The unemployment rate increased from 5 to 5.6% between 2008-9, as the fallout of the near totalcollapse of major markets took hold.The report lamented the fact that it had taken nine years to recover from the shock of the 2007/8global financial crisis.The report presents concerns about the female workforce participation rate. The much lower labourforce participation of women at 48%, compared with 75% for men means that around three in five ofthe 3.5 billion people in the global labour force were men.After a period of rapid improvement that lasted until 2003, subsequent progress on closing thegender gap in participation rates had stalled.2 billion of the 3.3 billion people in the global labour market worked in informal employment whichput their economic security at risk.The poor quality of many jobs also resulted in the fact that in 2018 more than one-quarter of workersin low and middle-income countries were living in extreme or moderate poverty.The global unemployment among young people (between 15-24) stood at 11.8 per cent, higher thanother age brackets.The report notes that in line with a stable aggregate unemployment rate, the outlook for men,women and young people with regard to opportunities on the job market is also very stable andfurther adds that neither the gender disparities nor the labour market challenges faced by youngpeople are expected to abate in the coming year.Youth participation in the labour market had been declining steadily for the last 25 years and one ofthe main causative factors for this was more young people were entering further education.

The report also notes that around 114 million children aged between 5-14 involved in the global labourmarket and 73 million of them were working in hazardous conditions, according to data from 2016.

World Sustainable Development Summit 2019The World Sustainable Development Summit 2019 was inaugurated by the Vice President of India, Shri M.

sri vishnu charan | [email protected] |

https://t.me/PDF4Examshttps://t.me/IAS201819 https://t.me/PDF4Exams

https://t.me/TheHindu_Zone_official

Page 83: Current Affairs -February 1-15, 2019 · 15-02-2019  · 10, theme: ‘Sadak Suraksha-Jeevan Raksha’ National Testing Agency (NTA) launches mobile app through which students can

Current Affairs [PDF] -February 1-15, 2019

© 2019 GKToday | All Rights Reserved | https://www.gktoday.in 83

Venkaiah Naidu in New Delhi. The World Sustainable Development Summit 2019, organized by TheEnergy and Resources Institute – TERI.World Sustainable Development SummitThe World Sustainable Development Summit is the annual flagship event of The Energy and ResourcesInstitute (TERI). World Sustainable Development Summit is the sole Summit on global issues taking placein the developing world.The World Sustainable Development Summit provides a platform for global leaders and practitioners todiscuss and deliberate over climatic issues of universal importance.The World Sustainable Development Summit strives to provide long-term solutions for the benefit of theglobal community by assembling the world’s most enlightened leaders and thinkers on a single platform.World Sustainable Development Summit is continuing the legacy of Delhi Sustainable DevelopmentSummit (DSDS) which was initiated in 2001 with the aim of making ‘sustainable development’ a globallyshared goal.The Energy and Resources Institute – TERITERI – the Tata Energy Research Institute was established by the Mr Darbari S Seth in 1974. Mr Darbari SSeth was the builder of Tata Chemicals Limited. He provided a decent sum for establishing TERI’s corpusand provided encouragement, inspiration, and moral support to the Director and staff of the Institutewithout ever interfering with the operational decision-making.Over time from its inception, TERI has emerged as one of the world’s pre-eminent think tanks andresearch institutions in the field of energy, climate change and sustainability.TERI aims to usher transitions to a cleaner and sustainable future through the conservation and efficientuse of Earth’s resources and innovative ways of minimizing and reusing waste.

e-Aushadhi PortalMinister of State for AYUSH, Shri Shripad Yesso Naik launched the e-Aushadhi portal for online licensingof Ayurveda, Siddha, Unani and Homoeopathy drugs.Features of the e-Aushadhi PortalThe new e-portal, e-Aushadhi is an acronym for Ayurveda, Unani, Siddha and Homeopathy AutomatedDrug Help Initiative. The important features of the e-Aushadhi Portal are:

The portal is intended to bring increased transparency, improved information management facility,improved data usability and increased accountability in the licensing of Ayurveda, Siddha, Unaniand Homoeopathy drugs.The timelines will be fixed for processing of application through this portal with SMS and e-mailstatus updates at each step of the process.The portal will aid the licensing authority, manufactures and consumers, as it will provide real-timeinformation of the licensed manufactures and their products, cancelled and spurious drugs, contactdetails of the concerned authority for specific grievances.

The launch of the e-Aushadhi portal for online licensing is part of governments commitment towards e-governance, ease of doing business and Make in India.

Rajasthan Backward Classes Amendment Bill, 2019The Rajasthan Assembly has passed the Rajasthan Backward Classes Amendment Bill, 2019. The bill aimsto address the demands of the Gujjars who are protesting for reservations.Features of the BillThe features of the Rajasthan Backward Classes Amendment Bill, 2019 are:

It provides five per cent quota in jobs and educational institutions to Gurjars, Banjaras, GadiaLohars, Raikas and Gadaria communities.The communities, namely Gurjars, Banjaras, Gadia Lohars, Raikas and Gadaria are currentlyprovided one per cent reservation under More Backward Classes (MBC).The Bill seeks to grant them an additional four per cent quota.The statement on the objective and reasons for the bill said the five castes are the most backwardand required a separate reservation in government jobs and educational institutes.The bill increases the ceiling of creamy layer from Rs 2.5 lakh to Rs 8 lakh.

sri vishnu charan | [email protected] |

https://t.me/PDF4Examshttps://t.me/IAS201819 https://t.me/PDF4Exams

https://t.me/TheHindu_Zone_official

Page 84: Current Affairs -February 1-15, 2019 · 15-02-2019  · 10, theme: ‘Sadak Suraksha-Jeevan Raksha’ National Testing Agency (NTA) launches mobile app through which students can

Current Affairs [PDF] -February 1-15, 2019

© 2019 GKToday | All Rights Reserved | https://www.gktoday.in 84

With the 5 per cent quota for Gurjars, Banjaras, Gadia Lohars, Raikas and Gadaria communities thebackward classes’ reservation in the state has increased from present 21 per cent to 26 per cent.The reservations in Rajasthan now crosses the 50 per cent cap on reservations set by the SupremeCourt.To avoid the possible legal tussle the Rajasthan Assembly passed a resolution and urged the Centreto incorporate it in the 9th schedule to ensure the reservation for these communities.

The statement on the objective and reasons for the bill notes that Parliament has amended theConstitution to exceed the limit of 50 per cent laid down by the Indra Sawhney case having regard to thecompelling circumstances in which Economically Weaker Sections of the society are languishing.

World Radio Day 2019The United Nations Educational, Scientific and Cultural Organization (UNESCO) every year celebrateFebruary 13 as the World Radio Day. World Radio Day 2019 is celebrated to highlight the importance of theradio in its search for a more peaceful and tolerant world.The World Radio Day 2019 is celebrated with the theme Dialogue, Tolerance and Peace. The theme isselected to provide a platform for dialogue and democratic debate over issues like migration or violenceagainst women which can help to raise awareness among listeners and inspire understanding for newperspectives in paving the way for positive action.Genesis of the World Radio DaySpain’s Radio Academy in the year 2010 proposed the idea of World Radio Day. In the following year,UNESCO declared February 13 as the World Radio day and the first World Radio Day was celebrated in2012.February 13 was chosen because the United Nations Radio, the United Nation’s international broadcastingservice was established on February 13, 1946. The World Radio Day commemorates the anniversary of theestablishment of the United Nations Radio.

February 15, 2019Jallianwala Bagh National Memorial (Amendment) Bill 2018

The Jallianwala Bagh National Memorial (Amendment) Bill, 2018 was passed in Lok Sabha. The features ofthe bill are:

Jallianwala Bagh National Memorial Act, 1951 provided for the erection of a National Memorial inmemory of those killed or wounded on April 13, 1919, in Jallianwala Bagh, Amritsar.The 1951 Act also provided for a Trust to manage the National Memorial.The Trust as per the 1951 Act included the Prime Minister, as Chairperson, (ii) the President of theIndian National Congress, (iii) the Minister in-charge of Culture, (iv) the Leader of Opposition in LokSabha, (v) the Governor of Punjab, (vi) the Chief Minister of Punjab, and (vii) three eminent personsnominated by the central government.The 2018 amendment bill removes the President of the Indian National Congress as a Trustee.Further, the bill clarifies that when there is no Leader of Opposition in Lok Sabha, the leader of thesingle largest opposition party in the Lok Sabha will be the Trustee.The 1951 act provided that the three eminent persons nominated by the central government willhave a term of five years and will be eligible for re-nomination.The 2018 bill added a clause to allow the central government to terminate the term of a nominatedtrustee before the expiry of his term without assigning any reason.

The government claimed that the bill was brought in to address various deficiencies in the management ofthe National Memorial and to ensure that the Trust is an apolitical entity.

Measures to Protect and Safeguard Land Rights of Scheduled TribesThe protect and safeguarding the land rights and other rights of Scheduled Tribes who are the mostmarginalized, isolated and deprived population, following constitutional and legislative measure havebeen put in place:

The Scheduled Tribes (STs) and Other Traditional Forest Dwellers (OTFDs) (Recognition of ForestRights) Act, 2006 is being implemented to recognize and vest the forest rights and occupation inforest land to forest dwelling Scheduled Tribes.

sri vishnu charan | [email protected] |

https://t.me/PDF4Examshttps://t.me/IAS201819 https://t.me/PDF4Exams

https://t.me/TheHindu_Zone_official

Page 85: Current Affairs -February 1-15, 2019 · 15-02-2019  · 10, theme: ‘Sadak Suraksha-Jeevan Raksha’ National Testing Agency (NTA) launches mobile app through which students can

Current Affairs [PDF] -February 1-15, 2019

© 2019 GKToday | All Rights Reserved | https://www.gktoday.in 85

Right to Fair Compensation and Transparency in Land Acquisition, Rehabilitation and ResettlementAct, 2013 (RFCTLARR Act, 2013 in short) safeguards against displacement of Scheduled Tribes.Special provisions have been made for under Sections 41 and 42 of the RFCTLARR Act, 2013 toprotect the interests of Scheduled Castes and Scheduled Tribes.The Panchayats (Extension to Scheduled Areas) Act, 1996 mandates that the Gram Sabha or thePanchayats at the appropriate level shall be consulted before making the acquisition of land in theScheduled Areas or development projects and before resettling or rehabilitating persons affected bysuch projects in the Scheduled Areas.Constitutional provisions under Schedule – V also provide for safeguards against displacement ofthe tribal population because of land acquisitions etc. The Governor of the State, having scheduledAreas, is empowered to prohibit or restrict the transfer of land from tribals and regulate theallotment of land to members of the Scheduled Tribes in such casesThe Legal Services Authorities Act, 1987 provides for legal services to members of Scheduled Tribes.Wrongfully dispossessing members of Scheduled Castes or Scheduled Tribes from their land orpremises or interfering with the enjoyment of their rights, including forest rights, over any land orpremises or water or irrigation facilities or destroying the crops or taking away the producetherefrom amount to atrocities and are subject to punishment under the Scheduled Castes and theScheduled Tribes (Prevention of Atrocities) Act, 1989.

Further, the Ministry of Tribal Affairs is implementing several schemes and programmes for upliftmentand development of Scheduled Tribes in the country.

India Withdraws MFN Status of PakistanOne day after the Pulwama terror attack on 14th February, 2019, India has taken a stern step ofwithdrawing the Most Favoured Nation or MFN Status of Pakistan. This move would enable India toincrease customs duty on goods coming from Pakistan. The decision was taken in the meeting of theCabinet Committee on Security (CCS) that took place on 15th February 2019.What is the MFN Status?

The MFN status is given under WTO’s General Agreement on Tariffs and Trade (GATT). It is given toan international trade partner to ensure non-discriminatory trade amongst all the members ofWTO.As per the first clause in the General Agreement on Tariffs and Trade (GATT), a country providingMFN status to another country has to provide concessions, privileges, and immunity in tradeagreements.WTO states that if a special status is granted to one trade partner, the country is required to extendit to all members of the WTO without discrimination or any special treatment.An MFN status helps reduce trade barriers and results in a reduction in tariffs especially in customsduty. This in turn strengthens trade-ties between the two countries.As India and Pakistan both are members of the WTO, both are required to grant MFN status to eachother and other partner countries.

What is the State of MFN Status Between India and Pakistan?India had granted MFN status to Pakistan in 1996, a year after the formation of WTO. But Pakistan hasn’taccorded MFN status to India till now. The reason for this is decades of conflict, mistrust and war. Nowthe total India-Pakistan trade has increased marginally to $2.41 billion in 2017-18 as compared to $ 2.27billion in 2016-17. India had imported goods worth $488.5 million in 2017-18 and exported goods worth $1.92 billion in that fiscal. India’s exports mostly include cotton, dyes, chemicals, vegetables and iron andsteel; while its imports include fruits, cement, leather, chemicals and spices. Given that Indo-Pakistantrade-ties is not very strong, this step of revoking the MFN status is only symbolic.What is the Cabinet Committee on Security?There are 6 Cabinet Committees in India i.e. Appointments Committee of the Cabinet, Cabinet Committeeon Accommodation, Cabinet Committee on Economic Affairs, Cabinet Committee on ParliamentaryAffairs, Cabinet Committee on Political Affairs and Cabinet Committee on Security. The Cabinet

sri vishnu charan | [email protected] |

https://t.me/PDF4Examshttps://t.me/IAS201819 https://t.me/PDF4Exams

https://t.me/TheHindu_Zone_official

Page 86: Current Affairs -February 1-15, 2019 · 15-02-2019  · 10, theme: ‘Sadak Suraksha-Jeevan Raksha’ National Testing Agency (NTA) launches mobile app through which students can

Current Affairs [PDF] -February 1-15, 2019

© 2019 GKToday | All Rights Reserved | https://www.gktoday.in 86

Committee on Security (CCS) of the Central Government of India looks into the matters of defenceexpenditures and National Security. It consists of the Prime Minister, Minister of Home Affairs, Ministerof External Affairs and, Minister of Finance and Minister of Defence. It is chaired by the Prime Minister.About the Pulwama Terror Attack37 CRPF personnel were martyred in Jammu and Kashmir’s Pulwama district on 14th February 2019 whena terrorist attacked with an explosive-laden vehicle into one of the buses of the convoy of the securityforces. Jaish-e-Mohammed has taken responsibility for the attack. There were more than 78 vehicles in theconvoy in which 2500 personnels were present. This is the worst terror attack since the Uri attack in 2016.

First District Cooling System of India to come up in AmaravatiIndia’s First District cooling system will come up in Amravati, capital of the Andhra Pradesh. Thegovernment of Andhra Pradesh has entered into a 30-year concession with UAE-based internationalcooling provider, The agreement between the National Central Cooling Company (Tabreed) for theDistrict Cooling System at Amaravati.District cooling systemDistrict cooling systems produce chilled water, steam or hot water at a central plant and then pipe thatenergy out (either underground or over rooftops) to buildings for air conditioning, space heating andwater heating. As a result, these buildings don’t require their own chillers, air conditioners, boilers orfurnaces.They are considered to be highly efficient to address each of the challenges like high Capital andoperating costs, reliability, flexibility and environmental sustainability while meeting their comfort andprocess cooling and heating needs.District cooling uses only 50 per cent of primary energy consumption for cooling urban building ncompared to other cooling systems. This also reduces carbon emissions.District cooling system at AmaravatiDistrict cooling system at Amaravati is part of Andhra Pradesh government’s vision to create jobs andhomes along with a world-class infrastructure at Amaravati.The District cooling system will meet the cooling requirements for the State’s Assembly, High Court,Secretariat and other government buildings that are currently under construction, for which coolingservices will commence from early 2021.

Minimum Selling Price of Sugar Increased by Rs 2/kgThe Central government has increased the minimum selling price (MSP) of sugar by Rs. 2 per kg to Rs. 31.

Minimum Selling PriceMinimum Selling Price is the rate below which the mills cannot sell sugar in the open market to wholesalers and bulkconsumers like beverage and biscuit makers.

Rationale behind the IncreaseThe decision to increase the minimum selling price has been taken keeping in mind the debt burden of theindustry. As per the data provided by the Indian Sugar Mills Association, arrears of sugarcane farmersstood at around Rs. 20,000 crore as of January-end.The increase in the Minimum Selling Price is expected to help millers to make the payment to sugarcanefarmers. The government is also putting in place a mechanism to ensure the benefit due to the increase inthe minimum selling price is passed on to the sugarcane farmers.Other Steps taken to aid the ailing Sugar MillsThe government has taken several other steps like the increase in import duty on sugar to 100 per cent,scrapping of export duty, the creation of buffer stock, and subsidy for mandatory export of 5 milliontonnes in the 2018-19 marketing year to help cash-starved mills clear cane dues.

WPI Inflation Hits 10-Month LowThe Union Ministry of Commerce and Industries has released the data on WPI Inflation. The data makesthe following observations:

The WPI Inflation fell to a 10-month low of 2.76 per cent in January owing to softening prices of fueland some food items.The WPI inflation has stood at 3.8 per cent in December 2018, and 3.02 per cent in January 2018.

sri vishnu charan | [email protected] |

https://t.me/PDF4Examshttps://t.me/IAS201819 https://t.me/PDF4Exams

https://t.me/TheHindu_Zone_official

Page 87: Current Affairs -February 1-15, 2019 · 15-02-2019  · 10, theme: ‘Sadak Suraksha-Jeevan Raksha’ National Testing Agency (NTA) launches mobile app through which students can

Current Affairs [PDF] -February 1-15, 2019

© 2019 GKToday | All Rights Reserved | https://www.gktoday.in 87

Manufactured products inflation which has a weightage of 64.23 per cent in WPI declined to 2.61 percent in January, from a level of 2.96 per cent in January 2018.The wholesale based price inflation for ‘fuel and power’ segment fell sharply to 1.85 per cent asagainst 8.38 per cent in December 2018, due to easing in prices of motor fuel and LPG.The government also revised the November WPI downwards to 4.47% from 4.64% earlier.The primary articles inflation with the weightage of 22.62 per cent increased to 3.54 per cent inJanuary from 2.53 per cent in same month last year.

In its last monetary policy review, RBI had decreased the lending rate by 0.25 per cent. It is expected thatthe decrease in inflation may provide further head-room to the RBI to cut interest rate (repo) in thecoming months.

Government Approves Continuation of CLCS-TUS Scheme for MSMEsThe Cabinet Committee on Economic Affairs (CCEA) has approved a three-year extension of the CreditLinked Capital Subsidy and Technology Up-gradation Scheme (CLCS-TUS) for Micro, Small and MediumEnterprises (MSME) with a total outlay of 2,900 crores on 13th February 2019. This scheme will continuebeyond the 12th Five Year Plan for three years from 2017-18 to 2019-20.What is the CLCS-TUS Scheme?

Under this scheme, an upfront capital subsidy of 15 per cent will be provided to the MSMEs oninstitutional finance of up to Rs 1 crore availed by them. This will be done to help the MSMEs inemploying well-established and improved technology in the specified 51 sub-sectors/productsapproved.The scheme aims to improve the competitiveness of MSMEs by integrating various current schemesaimed at up-grading technology through Credit Linked Capital Subsidy, hand holding for zero defectzero effect manufacturing, increasing productivity through waste reduction, design intervention,cloud computing, facilitation of intellectual property and nurturing new ideas.It will also focus on improving the quality of products and enhancement in productivity. This willpromote a culture of continuous development.The scheme also incorporates special provisions to increase entrepreneurship for SC/STs, women,hill states, island territories and the aspirational districts/ LWE (Left-wing extremism) districts.

CommentThe MSME sector has emerged as a highly vibrant and dynamic sector of the Indian economy. They notonly provide employment in the rural areas but are also a prelude to industrialization. MSMEs arecomplementary to large industries as ancillary units and this sector contributes enormously to the socio-economic development of the country.

NASA’s Mars Rover Opportunity Concludes a 15-Year MissionThe National Aeronautics and Space Administration (NASA) announced that its Mars Mission,Opportunity has come to an end. This marked the conclusion of the 15-year saga. The decision to end themission was made after all the efforts to restore contact with the Opportunity Mars rover didn’t yielddesired results.Why the NASA lost the contact with Opportunity?A historic global dust storm reached the location of the Opportunity rover on Mars. The storm darkenedthe skies and cut off of the rover’s solar power. All the efforts of NASA to restore the rover did not yieldpositive results.Things worsened with the onset of the winter at the location of the Opportunity rover. The reducedsunlight and colder temperatures during winter made it unlikely for the recovery of the Opportunityrover.About the MissionOpportunity was the second of the twin Mars Exploration Rovers to land on Mars in January 2004.Opportunity landed 90 days after its twin rover Spirit landed. Spirit landed at Gusev Crater andOpportunity landed on the opposite side of Mars at Meridiani Planum.NASA expected 90-day lifetimes for the rovers. Both Opportunity and Spirit far exceeded their expectedlifetime. Spirit’s mission ended in May 2011 after travelling eight kilometres and Opportunity had logged

sri vishnu charan | [email protected] |

https://t.me/PDF4Examshttps://t.me/IAS201819 https://t.me/PDF4Exams

https://t.me/TheHindu_Zone_official

Page 88: Current Affairs -February 1-15, 2019 · 15-02-2019  · 10, theme: ‘Sadak Suraksha-Jeevan Raksha’ National Testing Agency (NTA) launches mobile app through which students can

Current Affairs [PDF] -February 1-15, 2019

© 2019 GKToday | All Rights Reserved | https://www.gktoday.in 88

45 kilometres before losing contact in June 2018.Multiple Choice Questions

1. Which international organization has officially launched the International Year of the PeriodTable of Chemical Elements (IYPT-2019)?[A] UNESCO[B] UNIDO[C] FAO[D] ITU

Correct Answer: A [UNESCO ]Notes:On January 29, UNESCO official launched the International Year of the Period Table of ChemicalElements (IYPT-2019) at its headquarters in Paris to raise awareness of chemistry and its applicationsfor sustainable development. The year 2019 also marks the 150th anniversary of its creation by Russianscientist Dmitry Ivanovich Mendeleev. The aim of the year-long celebration is to recognize theimportance of the periodic table as one of the most important and influential achievements in modernscience, reflecting the essence not only of chemistry, but also of physics, biology and other basic sciencesdisciplines. The Periodic Table of Chemical Elements is a unique tool that enables scientist to predict theappearance and properties of matter on the Earth and in the rest of the Universe.

2. Who has been elected as the new chairman of Broadcast Audience Research Council of India(BARC)?[A] Nakul Chopra[B] Partho Dasgupta[C] Punit Goenka[D] Milan Sharma

Correct Answer: C [Punit Goenka]Notes:Punit Goenka, MD and CEO of Zee Entertainment Enterprises Limited (ZEEL), has been elected as thenew chairman of Broadcast Audience Research Council of India (BARC). He succeeded Nakul Chopra. Itmay be recalled that Goenka was the founder chairman of BARC India and played an instrumental role insetting up of the TV Viewership measurement company. It was under his leadership that BARC Indiaestablished its measurement system. BARC India is a joint industry company founded by stakeholderbodies that represent Broadcasters, Advertisers and Advertising & Media Agencies. Its headquarters islocated in Mumbai.

3. Which of the following countries have launched a common digital currency “Aber”?[A] Germany and France[B] Japan and South Korea[C] China and North Korea[D] UAE and Saudi Arabia

Correct Answer: D [UAE and Saudi Arabia]Notes:The central banks of the UAE and Saudi Arabia have launched a common digital currency called “Aber”,which will be used in financial settlements between the two countries through Blockchains andDistributed Ledgers technologies. It will establish an additional means for the central financial transfersystems of the two countries and enable banks to directly deal with each other in conducting financialremittances. Through this digital currency, both the United Arab Emirates Central Bank (UAECB) andthe Saudi Arabian Monetary Authority (SAMA) are studying the impact on the improvement andreduction of remittance costs and the assessments of risks.

4. The Africa Centre for Climate and Sustainable Development has recently launched in which ofthe following countries?[A] South Africa[B] Italy

sri vishnu charan | [email protected] |

https://t.me/PDF4Examshttps://t.me/IAS201819 https://t.me/PDF4Exams

https://t.me/TheHindu_Zone_official

Page 89: Current Affairs -February 1-15, 2019 · 15-02-2019  · 10, theme: ‘Sadak Suraksha-Jeevan Raksha’ National Testing Agency (NTA) launches mobile app through which students can

Current Affairs [PDF] -February 1-15, 2019

© 2019 GKToday | All Rights Reserved | https://www.gktoday.in 89

[C] Netherlands[D] Brazil

Correct Answer: B [Italy]Notes:In Rome, Italian Prime Minister Giuseppe Conte has recently launched the new ‘Africa Centre forClimate and Sustainable Development’ in partnership with the United Nations Development Programme(UNDP) and FAO to fight climate change and boost sustainable development in Africa. The centre willprovide a fast-track, demand-driven mechanism for African countries to access grant resources thatsupport policies, initiatives, and best practices on climate change, food security, access to water, cleanenergy, and accelerating progress on the Sustainable Development Goals (SDGs) in Africa. The AfricaCentre has its roots in a declaration endorsed by the G7 meeting of the Environment Ministers in 2017.The new centre is established to facilitate coordination among the G7 and African countries on commoninitiatives in Africa to develop and scale-up innovative solutions for sustainable development, and toachieve the goals set by the 2015 Paris Agreement and the UN’s 2030 Agenda.

5. What is the India’s rank at the 2018 Corruption Perception Index (CPI) of TransparencyInternational (TI)?[A] 78th[B] 81st[C] 62nd[D] 54th

Correct Answer: A [78th]Notes:Global watchdog Transparency International (TI) has recently released 2018 Corruption PerceptionsIndex (CPI), which ranks 180 countries and territories by their perceived levels of public sectorcorruption according to experts and businesspeople. The index uses a scale of 0 to 100, where 0 is highlycorrupt and 100 is very clean. Here, India has improved its ranking and jumped three positions to 78thwith a CPI score of 41 at the 2018 index. Denmark has topped the index with 88 points, followed by NewZealand (with 87 points) and Finland with 85. Sudan was the worst with a score of 10 points, whileSomalia and Syria followed it up with 13 points each. Overall, more than two-thirds of countries scorebelow 50 in the 2018 CPI, with an average score of only 43. The 2018 index revealed that the continuedfailure of most countries to significantly control corruption is contributing to a crisis in democracyaround the world. While there are exceptions, the data shows that despite some progress, mostcountries are failing to make serious inroads against corruption.

6. Who has become the fastest Indian bowler to reach 100 ODI wickets?[A] Yuzvendra Chahal[B] Kuldeep Yadav[C] Kedar Jadhav[D] Mohammed Shami

Correct Answer: D [Mohammed Shami]Notes:Mohammed Shami has become the fastest Indian bowler to claim 100 ODI wickets, during the first ODIagainst New Zealand at McLean Park in Napier on 23rd January 2019. Shami reached the milestone inhis 56th match, with the wicket of opener Martin Guptill. Before Shami, Irfan Pathan reached the 100-wicket mark in 59 ODIs, while Zaheer Khan (65 matches), Ajit Agarkar (67) and Javagal Srinath (68) arethe other Indian bowlers to complete a century of wickets in the quickest time. The world record iscurrently held by Afghanistan leg-spinner Rashid Khan, who took 44 matches to reach the mark andbroke the record of 52 matches, set by Mitchell Starc of Australia.

7. India’s second Tulip Garden will come up in which of the following states?[A] Nagaland[B] Jharkhand[C] Uttarakhand[D] Sikkim

sri vishnu charan | [email protected] |

https://t.me/PDF4Examshttps://t.me/IAS201819 https://t.me/PDF4Exams

https://t.me/TheHindu_Zone_official

Page 90: Current Affairs -February 1-15, 2019 · 15-02-2019  · 10, theme: ‘Sadak Suraksha-Jeevan Raksha’ National Testing Agency (NTA) launches mobile app through which students can

Current Affairs [PDF] -February 1-15, 2019

© 2019 GKToday | All Rights Reserved | https://www.gktoday.in 90

Correct Answer: C [Uttarakhand ]Notes:After Srinagar (J&K), India’s second Tulip Garden will come up at Pithoragarh district of Uttarakhandover 50 hectares of forest land near Chandak hill top at a cost of Rs 50 crore. The proposed tulip gardenwill be developed by the ONGC under its Corporate Social Responsibility (CSR) programme. Tulip is theflower from Holland that blooms from mid March to May. The site in Mad village near Chandak hilltopfor the garden has been selected under the state Government’s ’13 districts, 13 new destinations’ schemeto attract more tourists to the district. According to the project officer of the scheme the land availablefor the project in Chandak hill top is ideal for the growth of the flower as it is situated at a height of 1,950metres from the sea surface and remains sunny and cold throughout the year. Tulip is the flower fromHolland that blooms from mid March to May.

8. Dara Dotiwalla, who passed away recently, was associated with which sports?[A] Cricket[B] Badminton[C] Tennis[D] Football

Correct Answer: A [Cricket]Notes:Former Indian umpire Dara Dotiwalla (85), who officiated in the famous tied Test match in Chennaibetween India and Australia in 1986, has passed away in Mumbai on January 30, 2019. Born on 30thOctober 1933, Dotiwalla had officiated in six Test matches from 1982 to 1987, including the tied Testbetween India and Australia at the M A Chidambaram Stadium in Chennai. He had also officiated ineight One Day Internationals (ODIs) from 1982 to 1988. He is best remembered for officiating theChennai Tied Test of 1986 along with V Vikramraju. The match is the only the second tied Test in crickethistory, and ended with Vikramraju controversially adjudging India’s No. 11 Maninder Singh lbw to GregMatthews. Dotiwalla’s first ODI as an umpire was the fixture between India and Sri Lanka in Bengaluruin 1982, while his last one was India versus New Zealand in 1988 in Cuttack. The Delhi Test betweenIndia and West Indies in 1987 was his last assignment as a Test umpire.

9. Which Indian personality has received Carnot Prize for Power Reforms, Rural Electrification?[A] Mukesh Ambani[B] Piyush Goyal[C] Birender Singh Chaudhary[D] Ratan Tata

Correct Answer: B [Piyush Goyal]Notes:Union Finance Minister Piyush Goyal has been awarded the Carnot prize for his contribution towardssustainable energy solutions by the team of University of Pennsylvania on January 30. The awardrecognizes Goyal’s tenure as minister of power in which he is credited to have reformed the electricitysector and extended access to electricity to 18,000 villages, as documented by the International EnergyAgency (IEA). He was to be awarded the prize by the Kleinman Center for Energy Policy at the UniversityOf Pennsylvania School Of Design in October 2018. But, he had to skip the event because of the Amritsartragedy in which over 60 people died after being mowed down by a train.

10. India’s flagship hydrocarbon international conference ‘PETROTECH-2019’ is scheduled to beheld in which of the following cities?[A] Pune[B] Kolkata[C] Greater Noida[D] Hyderabad

Correct Answer: C [Greater Noida]Notes:The 13th International Oil & Gas Conference & Exhibition ‘PETROTECH-2019’ is being organised underthe aegis of the Ministry of Petroleum & Natural Gas in Greater Noida from 10th to 12th February, 2019.

sri vishnu charan | [email protected] |

https://t.me/PDF4Examshttps://t.me/IAS201819 https://t.me/PDF4Exams

https://t.me/TheHindu_Zone_official

Page 91: Current Affairs -February 1-15, 2019 · 15-02-2019  · 10, theme: ‘Sadak Suraksha-Jeevan Raksha’ National Testing Agency (NTA) launches mobile app through which students can

Current Affairs [PDF] -February 1-15, 2019

© 2019 GKToday | All Rights Reserved | https://www.gktoday.in 91

India’s flagship hydrocarbon international conference is expected to be inaugurated by Prime Ministerof India. Over 95 Energy Ministers from partner countries have been invited by the Minister forPetroleum and Natural Gas to participate in the Conference. The 3-day mega event will showcase therecent market and investor friendly developments that have taken place in the India’s oil and gas sector.It is expected to see participation of over 86 eminent speakers and 7000 delegates from around 70countries, including technologists, scientists, planners, policy-makers, management experts,entrepreneurs, service-providers and vendors.

11. Which country’s Aerospace Industries has signed $93 million Naval MRSAM deal with IndianNavy?[A] South Korea[B] United States[C] Israel[D] Germany

Correct Answer: C [Israel ]Notes:Israel Aerospace Industries (IAI) has recently signed agreements worth $93 million for providing NavalMRSAM (Medium Range Surface-to-Air Missile) systems to India. The contracts for the air defencesystem were entered with the Indian Navy and the Cochin Shipyard. Under the contracts, IAI willprovide complementary systems for the air defense system (ADS). The MRSAM family is an operationalair-defense system used by Israel’s navy as well as by India’s naval, air and ground forces. It providesbroad as well as topical defense against a range of assault air, marine and ground threats. MRSAMcomprises several key state-of-the-art systems, including a digital radar, command and control,launchers, and interceptors with advanced homing seekers.

12. What is the theme of the 2019 World Leprosy Day (WLD)?[A] To live is to help to live[B] Leprosy defeats, Live transform[C] Ending discrimination, stigma and prejudice[D] Zero Disabilities in girls and boys

Correct Answer: C [Ending discrimination, stigma and prejudice]Notes:The World Leprosy Day (WLD) is observed every year on the last Sunday of January to spread awarenessabout the disease, the impact of discrimination and social stigma attached to it and how these negativeeffects hamper the efforts to stop the spread of the disease. This year, in 2019, it falls on 27 January andits theme is ‘Ending discrimination, stigma and prejudice’. Leprosy (or Hansen disease) is one of theoldest recorded diseases in the world. The disease is caused by the bacteria called Mycobacterium lepraeand is contagious. It is an infectious chronic disease that targets the nervous system, especially thenerves in the cooler parts of the body – the hands, feet, and face. This chronic disease can be passed onfrom one person to another or even by breathing airborne droplets from the affected individuals. Evencough and sneezes, or coming into contact with the affected individual nasal fluids could lead to leprosy.The infection can be contracted at any age.

13. Who is the newly appointed DG of National Cadet Corps (NCC)?[A] Rajeev Chopra[B] Neeraj Sharma[C] P S Bhanubhat[D] S K Kamath

Correct Answer: A [Rajeev Chopra]Notes:Lt Gen Rajeev Chopra has taken charge as new Director General of National Cadet Corps (NCC) in NewDelhi. Commissioned in the Madras Regiment in December 1980, Lt Gen Chopra is an alumnus of theNational Defence Academy, Khadakwasla and Indian Military Academy, Dehradun. The NCC is an Indianmilitary cadet corps with its headquarters at New Delhi, It is the world’s largest uniformed youthorganization. Its motto is ‘Unity and Discipline’.

sri vishnu charan | [email protected] |

https://t.me/PDF4Examshttps://t.me/IAS201819 https://t.me/PDF4Exams

https://t.me/TheHindu_Zone_official

Page 92: Current Affairs -February 1-15, 2019 · 15-02-2019  · 10, theme: ‘Sadak Suraksha-Jeevan Raksha’ National Testing Agency (NTA) launches mobile app through which students can

Current Affairs [PDF] -February 1-15, 2019

© 2019 GKToday | All Rights Reserved | https://www.gktoday.in 92

14. The National Salt Satyagraha Memorial (NSSM) is located in which of the following districts ofGujarat?[A] Mehsana[B] Kheda[C] Bharuch[D] Navsari

Correct Answer: D [Navsari ]Notes:On 30th January2019, Prime Minister Narendra Modi dedicated the National Salt Satyagraha Memorialat Dandi in Navsari district, Gujarat to the nation on Mahatma Gandhi’s death anniversary. At thememorial site, there are statues of Mahatma Gandhi and 80 Satyagrahis who had marched with himduring the historic Dandi Salt March in 1930 to make salt from sea water against the British law. Thememorial also has 24-narrative murals depicting various events and stories from the historic 1930 SaltMarch. Solar trees are installed to meet the energy requirements of the memorial complex. Thememorial project of the Union Ministry of Culture is advised by a High Level Dandi MemorialCommittee (HLDMC), and is designed, coordinated and implemented by IIT Bombay in association withan international design team.

15. The 2019 International conference on Guru Padmasambhava was held in which of the followingcities?[A] Gorakhpur[B] New Delhi[C] Dharamshala[D] Sanchi

Correct Answer: B [New Delhi]Notes:On January 29, the 2-day International conference on 8th century Himalayan sage GuruPadmasambhava was held in New Delhi that saw the participation of distinguished scholars from bothcountries. The conference titled ‘Life and Legacy of Guru Padmasambhava’, was organized by the Centrefor Escalation of Peace (CEP). Scholars from both countries discussed the Guru who was born in Indiaand moved towards Bhutan in the 8th century to spread Buddhism and Buddhist teachings all across theHimalayan region. The conference was dedicated to the 50 years of diplomatic relations between Indiaand Bhutan. In Tibetan Buddhism, Guru Padmasambhava is generally referred to as Guru Rinpoche,which means “precious master.” Guru Rinpoche is a totally enlightened being, a fully awakened one, abuddha.

16. ISRO has launched Human Space Flight Centre (HSFC) in which of the following cities?[A] Kochi[B] Chennai[C] Bengaluru[D] Hyderabad

Correct Answer: C [Bengaluru ]Notes:On January 30, ISRO launched the Human Space Flight Centre (HSFC) at its headquarters in Bengalurufor the human spaceflight programme. The centre is the hub of ISRO’s future manned missions. HSFCshall be responsible for the implementation of Gaganyaan project — which involves mission planning,development of engineering systems for crew survival in space, crew selection and training and alsopursue activities for sustained human space flight missions. HSFC will take the support of ISRO centresto implement the first developmental [crewed] flight. Under the Gaganyaan project, a 3-member crewwill be sent to space for at least 7 days by 2022, the 75th year of Independence. Dr. S Unnikrishnan Nairis the founder director of HSFC, while R Hutton is the project director of Gaganyaan.

17. Which of the following countries have officially set up a payment channel with Iran calledINSTEX?[A] Italy, Russia and the Netherlands

sri vishnu charan | [email protected] |

https://t.me/PDF4Examshttps://t.me/IAS201819 https://t.me/PDF4Exams

https://t.me/TheHindu_Zone_official

Page 93: Current Affairs -February 1-15, 2019 · 15-02-2019  · 10, theme: ‘Sadak Suraksha-Jeevan Raksha’ National Testing Agency (NTA) launches mobile app through which students can

Current Affairs [PDF] -February 1-15, 2019

© 2019 GKToday | All Rights Reserved | https://www.gktoday.in 93

[B] Brazil, South Africa and Egypt[C] Saudi Arabia, Denmark and Japan[D] Germany, France and the UK

Correct Answer: D [Germany, France and the UK]Notes:Germany, France and the UK have officially set up a payment channel with Iran called INSTEX, to helpcontinue trade and circumvent US sanctions. The long-awaited special payment system will help savethe Iran nuclear deal by allowing Tehran to keep trading with EU companies despite Washington re-imposing sanctions after US President Donald Trump abruptly quit the accord in 2018. The threecountries have launched the device at a meeting of EU foreign ministers in Bucharest, Romania. The newinstitution ‘INSTEX’ is a project of the three governments and will receive the formal endorsement of all28 EU members. The Instrument In Support Of Trade Exchanges (INSTEX) is registered in France andwill be headed by German banker Per Fischer, a former Commerzbank director.

18. Who has become the first female cricketer to play 200 ODIs?[A] Jemimah Rodrigues[B] Mithali Raj[C] Jhulan Goswami[D] Smriti Mandhana

Correct Answer: B [Mithali Raj]Notes:Indian captain Mithali Raj has scripted history by becoming the first female cricketer to play 200 ODIswhen she played 3rd and final ODI against New Zealand on February 1. Though, she could not make a bigimpact in her 200th game, scoring nine off 28 balls as India were bowled out for 149 in the third and finalODI against New Zealand at Hamilton. Mithali is also the highest run-getter in ODIs with 6622 runs atan average of 51.33, including seven hundreds. She also holds the record for having captained the IndiaWomen team 123 times with England’s Charlotte Edwards again second on this list having captained theThree Lions for 117 matches. Mithali made her ODI debut way back in 1999 and has featured in 200games out of the 263 ODIs India have played. She has also played 10 Tests and 85 T20.

19. The newly-named Department for Promotion of Industry and Internal Trade will work underwhich union ministry?[A] Ministry of Labour and Employment[B] Ministry of Commerce and Industry[C] Ministry of Finance[D] Ministry of Consumer Affairs

Correct Answer: B [Ministry of Commerce and Industry]Notes:The Department of Industrial Policy and Promotion (DIPP) has been renamed as the Department forPromotion of Industry and Internal Trade with a mandate to deal with matters related to start-ups,facilitating ease of doing business among others. The move comes amidst demand by traders’ body CAIT(Confederation of All India Traders) for forming a separate ministry for internal trade. The newly-nameddepartment under the Ministry of Commerce and Industry will look into matters related to promotion ofinternal trade, including retail trade, welfare of traders and their employees, facilitating ease of doingbusiness and start-ups. Earlier, the matters related to internal trade were under the domain of theMinistry of Consumer Affairs.

20. A music and dance festival ‘Sopan 2019’ has started in which of the following cities?[A] New Delhi[B] Bhopal[C] Jaipur[D] Kanpur

Correct Answer: A [New Delhi]Notes:

sri vishnu charan | [email protected] |

https://t.me/PDF4Examshttps://t.me/IAS201819 https://t.me/PDF4Exams

https://t.me/TheHindu_Zone_official

Page 94: Current Affairs -February 1-15, 2019 · 15-02-2019  · 10, theme: ‘Sadak Suraksha-Jeevan Raksha’ National Testing Agency (NTA) launches mobile app through which students can

Current Affairs [PDF] -February 1-15, 2019

© 2019 GKToday | All Rights Reserved | https://www.gktoday.in 94

On 1st February, a music and dance festival ‘Sopan 2019’ has started at Central Park in New Delhi toencourage young and upcoming artistes to take up traditional art forms. The fest is organised by SahityaKala Parishad and the Delhi government in association with New Delhi Municipal Council (NDMC). It isan attempts to take forward the traditional Indian art forms. This 6-day long festival is an opportunityfor upcoming talents from vocals and instruments to dance forms to set an audience base forthemselves. Young artistes such as Odissi dancers Madhur Gupta and Premananda Sahoo, vocalistShivam Bhardwaj, percussionists Aamir Hussan and Amaan Jamil, and Kathak dancers Mrinalini amongothers will perform during the musical extravaganza.

21. The Kala Ghoda Arts Festival (KGAF-2019) has started in which of the following cities?[A] Mumbai[B] New Delhi[C] Hyderabad[D] Jaipur

Correct Answer: A [Mumbai]Notes:On 2nd February, the Kala Ghoda Arts Festival (KGAF-2019) has started in Mumbai to commemorate 20years of its inception. The fest is the India’s largest multi-cultural street festival, which will celebratetwo glorious decades of art and culture through cinema, theatre, dance, literature and sculpture throughvarious spots on Mumbai’s map. The festival will also host exhibition at the historic Jehangir Art Galleryto celebrate the 150th birth anniversary of Mahatma Gandhi and some rich history of Mumbai. Some ofthe festival highlights include screening of Khalil Mohammad’s documentary on Iranian cafes,performances by Pt. Hariprasad Chaurasia, Shaan, Prakriti and Sukriti Kakar, panel discussion on 26/11attacks and its survivors, a session on working of drones, heritage walks, dances and shopping atRampart Row. The festival will end on 10th February.

22. Who has become the first Indian golfer to grab Australian LPGA card?[A] Diksha Dagar[B] Astha Madan[C] Vani Kapoor[D] Ridhima Dilawari

Correct Answer: C [Vani Kapoor]Notes:Vani Kapoor has become the first Indian golfer to earn a card for the Australian Ladies PGA Tour (LPGA)after coming through its first-ever Qualifying Tournament at the Ballarat Golf Club. Vani shot rounds of71, 78 and 69 to finish at two-over 218 and was tied-12th with three others. Vani has also qualified for VicOpen, co-sanctioned by the Australian LPGA and the US LPGA.

23. Which country has won the 2019 Asian Cup Football tournament?[A] Japan[B] Qatar[C] Iran[D] Singapore

Correct Answer: B [Qatar ]Notes:The Qatar national football team has won their first major football title after defeating four-timechampion Japan 3-1 in the 2019 AFC Asian Cup final on 1st of February in United Arab Emirates (UAE). Itwas an improbable run to glory for the Gulf nation as it prepares to host the 2022 World Cup. In it,Almoez Ali has broken the record for the most goals ever scored by a single player at an Asian Cuptournament (9) and won most valuable player award of 2019 tournament. Earlier, the record was held byAli Daei who scored eight for Iran in the 1996 edition. Almoez Ali is a Qatari professional footballer ofSudanese descent who plays for Qatar and Al-Duhail in the Qatar Stars League as a forward.

24. Which of the following teams has won the Kochi Area Pulling Regatta 2019 ?[A] INS Vikrant[B] INS Dronacharya

sri vishnu charan | [email protected] |

https://t.me/PDF4Examshttps://t.me/IAS201819 https://t.me/PDF4Exams

https://t.me/TheHindu_Zone_official

Page 95: Current Affairs -February 1-15, 2019 · 15-02-2019  · 10, theme: ‘Sadak Suraksha-Jeevan Raksha’ National Testing Agency (NTA) launches mobile app through which students can

Current Affairs [PDF] -February 1-15, 2019

© 2019 GKToday | All Rights Reserved | https://www.gktoday.in 95

[C] INS Betwa[D] INS Vikramaditya

Correct Answer: B [INS Dronacharya]Notes:A team from INS Dronacharya has won the Regetta trophy (also called The Cock) in the Kochi AreaPulling Regatta 2019 of the Southern Naval Command (SNC) in the Ernakulam Channel on January 31.The Boat Pulling regatta is one of the most prestigious and traditional naval sporting activitiesconducted in the Navy. The boats used in the regatta are 27-feet-long ‘whalers’ — a type of seaboat —each having a crew of five ‘pullers’ (rowers) and one coxswain (helm). The boats are named as ‘whalers’ asthey were used in ancient times for hunting whales at sea. Races during the regatta were held in fourdifferent categories – – junior sailors, senior sailors, officers and best whaler. The boats had to cover adistance of 1.6 km in the Ernakulam Channel, starting from the Venduruthy-Vikrant Bridge andfinishing at the North Jetty of the navy. The regatta is considered a signature event of the Command,which requires the highest standards of physical and mental fitness and sustained hard work.

25. Porcine Reproductive and Respiratory Syndrome (PRRS), which is in news recently, is relatedto which of the following animals?[A] Cow[B] Camel[C] Pig[D] Goat

Correct Answer: C [Pig]Notes:The Mizoram government has banned import of pigs and piglets from other countries in view of possibleoutbreak of the Porcine Reproductive and Respiratory Syndrome (PRRS) that has cost the lives ofthousands of swines in the state. The orders were issued as the dreaded PRRS outbreak always occurredfrom March to July, during a dry spell after pre-monsoon downpours. According to the records of theVeterinary department, the PRRS has cost the lives of over 10,000 swines in Mizoram since 2013 and it isbelieved that it happened due to import of pigs and piglets from Myanmar where the disease was knownto be prevalent. The PRRS is also known as blue-ear pig disease, which affectes domestic pigs. Thesymptoms include reproductive failure, pneumonia and increased susceptibility to secondary bacterialinfection.

26. What is the theme of the 2019 World Wetlands Day (WWD)?[A] Forests and Wetlands[B] Wetlands and Climate Change[C] Wetlands for our Future[D] Wetlands for a Sustainable Urban Future

Correct Answer: B [Wetlands and Climate Change]Notes:The World Wetland Day (WWD) is observed every year on 2nd of to raise awareness about the value ofwetlands for humanity and the planet. The day marks adoption of the Convention on Wetlands (alsocalled as Ramsar Convention) on 2nd February 1971, in the Iranian city of Ramsar on shores of theCaspian Sea. The 2019 theme “Wetlands and Climate Change” highlights the importance of healthy andintact wetlands to one of the most pressing challenges of climate change.

27. The first-ever global report “Future of the Rail” has prepared by which internationalorganisation?[A] IEA[B] WHO[C] IMF[D] AIIB

Correct Answer: A [IEA]Notes:

sri vishnu charan | [email protected] |

https://t.me/PDF4Examshttps://t.me/IAS201819 https://t.me/PDF4Exams

https://t.me/TheHindu_Zone_official

Page 96: Current Affairs -February 1-15, 2019 · 15-02-2019  · 10, theme: ‘Sadak Suraksha-Jeevan Raksha’ National Testing Agency (NTA) launches mobile app through which students can

Current Affairs [PDF] -February 1-15, 2019

© 2019 GKToday | All Rights Reserved | https://www.gktoday.in 96

The International Energy Agency (IEA) has prepared the report titled ‘The Future of Rail’ in associationwith the International Union of Railways (UIC). This first-ever global report has a specific focus on India,where rail passenger traffic increased by nearly 200% since 2000. The report stated that Indian railwayswill account for nearly 40% of total global rail activity by 2050. The investments in Indian urban railinfrastructure are estimated to reach nearly $190bn by 2050. As a result, India can save up to $64bn onfuel expenses by that time. The IAEA report analyses the current and future importance of rail aroundthe world through the perspective of its energy and environmental implications. As per the report, therail sector carries 8% of the world’s passengers and 7% of global freight transport but it utilises only 2%of total transport energy demand. The regions with the highest share of electric train activity areEurope, Japan and Russia, while North and South America still rely heavily on diesel. Most conventionalrail networks are located in North America, Europe, China, Russia, India, and Japan.

28. What is the theme of the 12th edition of Aero India 2019?[A] The Runway to a Billion Opportunities[B] The Runway to a Million Opportunities[C] The Runway to a Trillion Opportunities[D] The Runway to Unlimited Opportunities

Correct Answer: A [The Runway to a Billion Opportunities]Notes:The 12th edition of the India’s premier air show – Aero India 2019 will be held at Air Force Station atYelahanka in Bengaluru from February 20 to 24. The Ministry of Defence has lined up programmes forStart-Ups Day, Women’s Day, Daily Students’ Day, Drone Olympics and photography contests assideshows to make it an interactive affair. More than 350 exhibitors will be accommodated in fivehangers at the Air Force Station. This year, the aero show is themed ‘The Runway to a BillionOpportunities’, to communicate India’s value proposition in addressing the immense opportunities inthe aerospace and aviation sectors in the country today. The logo for Aero India 2019 is inspired by theTejas Light Combat Aircraft (LCA).

29. Which Indian state has topped in budgetary practices, as per latest survey by TransparencyInternational (TI)?[A] Andhra Pradesh[B] Odisha[C] Assam[D] Punjab

Correct Answer: C [Assam]Notes:As per latest survey conducted by Transparency International (TI), Assam has occupied the top slot inthe ranking of best practices followed by states in Budget formulation and is followed by AndhraPradesh and Odisha. The states which figured lower in the ranking were Meghalaya, Manipur andPunjab. The survey was carried out on the basis of four parameters – public disclosure, budgetaryprocess, post budget fiscal management and efforts to make budget more transparent and citizenfriendly. Transparency International has stated in its survey report that Assam is the only state out ofthe 29 states and two union territories which has published a Citizens Budget in the public domain. Italso stated that the Assam Government is the only government that has conducted budget awarenesscampaigns across 17 districts.

30. India’s unemployment rate hit a 4 decade high to what percent in 2017-18, as per PLFS surveyby National Sample Survey Office (NSSO)?[A] 5.2%[B] 6.1%[C] 4.3%[D] 7.3%

Correct Answer: B [6.1%]Notes:As per the National Sample Survey Office’s (NSSO’s) periodic labor force survey (PLFS), India’sunemployment rate hit a 45 year high of 6.1% in 2017-18. The latest data on unemployment in India is the

sri vishnu charan | [email protected] |

https://t.me/PDF4Examshttps://t.me/IAS201819 https://t.me/PDF4Exams

https://t.me/TheHindu_Zone_official

Page 97: Current Affairs -February 1-15, 2019 · 15-02-2019  · 10, theme: ‘Sadak Suraksha-Jeevan Raksha’ National Testing Agency (NTA) launches mobile app through which students can

Current Affairs [PDF] -February 1-15, 2019

© 2019 GKToday | All Rights Reserved | https://www.gktoday.in 97

first comprehensive survey on employment conducted by a government agency after Prime MinisterNarendra Modi announced demonetisation move in November 2016. The report, which is yet to come inpublic domain, stated that this level of unemployment was last seen in 1972-73. According to the survey,the rate of unemployment among rural males in the age group of 15-29 years was at 17.4%, more thanthree times the 2011-12 rate of 5%. A similar trend was seen in females in the same age group, where therate jumped from 4.8% in 2011-12 to 13.6% in 2017-18. The rate of joblessness for urban youths was awhopping 18.7% for males and 27.2% for females.

31. Rishi Kumar Shukla, who has been appointed the new Director of the Central Bureau ofInvestigation (CBI), is from which state?[A] Rajasthan[B] Madhya Pradesh[C] Uttar Pradesh[D] Jharkhand

Correct Answer: B [Madhya Pradesh]Notes:Rishi Kumar Shukla, a 1983-batch IPS officer of Madhya Pradesh Cadre, has been appointed the newDirector of the Central Bureau of Investigation (CBI). He will serve in the post for a period of two years.The High-Level Committee chaired by Prime Minister approved the appointment of Mr Shukla to thepost. He succeeded M. Nageshwara Rao, who was working as Interim Chief of the Agency. The post ofCBI Chief has been lying vacant since 10th of January 2019 after the exit of Alok Kumar Verma followingaccusations of corruption against him by former CBI Special Director Rakesh Asthana who is alsoallegedly accused of corruption charges. Prior to being appointed as the director of the CBI, Mr. Shuklahas served as Madhya Pradesh DGP in 2016 and was presently the chairman of the Madhya PradeshPolice Housing Corporation (PHC) in Bhopal.

32. Who has been appointed as the new Nepalese ambassador to India?[A] Nilambar Acharya[B] Deep Kumar Upadhyay[C] Arjun Karki[D] Shankar Prasad Sharma

Correct Answer: A [Nilambar Acharya]Notes:Nilambar Acharya, Nepal’s former law minister, has been appointed as the country’s ambassador toIndia. The position of the Nepalese ambassador to India was vacant for more than a year after DeepKumar Upadhyay resigned to join politics in October 2017. President Bidya Devi Bhandari has recentlyadministered oath of office to Acharya at Rastrapati Bhawan at Sheetal Niwas. Acharya is the firstNepalese ambassador to take oath of office and secrecy from the president under a new provision.Earlier, ambassadors were administered oath of office by Supreme Court chief justice.

33. Who is the recipient of K.Veeramani Award for Social Justice 2018?[A] P S Krishnan[B] Nitish Kumar[C] S T Moorthy[D] Veera Munusam

Correct Answer: A [P S Krishnan]Notes:P.S. Krishnan, former Secretary to Government of India (GoI), has been conferred prestigious K.Veeramani Award for Social Justice 2018 for his works in the field of social justice for SCs, STs, BCs forthe last more than six-and-half decades. The Periyar International Inc., a Chicago-based voluntaryorganisation, gives an annual award for outstanding contributions to Social Justice at a function in NewDelhi. The award carries a plaque and a cash prize of Rs 1 lakh.

34. Behrouz Boochani, who won the Australia’s richest literary prize, is from which country?[A] Palestine

sri vishnu charan | [email protected] |

https://t.me/PDF4Examshttps://t.me/IAS201819 https://t.me/PDF4Exams

https://t.me/TheHindu_Zone_official

Page 98: Current Affairs -February 1-15, 2019 · 15-02-2019  · 10, theme: ‘Sadak Suraksha-Jeevan Raksha’ National Testing Agency (NTA) launches mobile app through which students can

Current Affairs [PDF] -February 1-15, 2019

© 2019 GKToday | All Rights Reserved | https://www.gktoday.in 98

[B] Saudi Arabia[C] Iran[D] Oman

Correct Answer: C [Iran]Notes:The Kurdish Iranian journalist, filmmaker and detained refugee, Behrouz Boochani has won theVictorian Prize for Literature for a book he reportedly wrote using the online messaging serviceWhatsApp. He was awarded the Aus$100,000 (US$72,600) prize for his book “No Friend But theMountains: Writing from Manus Prison”. He will receive an additional Aus$25,000 after it also won thenon-fiction category. The award was accepted by the book’s translator Omid Tofighian, who workedwith Boochani over five years to bring the stories to life. It is the most valuable literary prize of Australia.Boochani is an Iranian asylum-seeker, who detained in Papua New Guinea (PNG)’s Manus Island since2013 under Australian asylum laws when he was found attempting to enter mainland Australia withouta valid visa. He is among 600 refugees who remain in camps on the island despite Australia havingclosed its “regional processing center” there in 2017. Under Australia’s hardline immigration policy,asylum seekers who try to reach the country by boat to Manus Island or Nauru in the Pacific forprocessing, with those found to be refugees barred from resettling in Australia. The harsh policy ismeant to deter people embarking on treacherous sea journeys, but the United Nations and other rightsgroups have criticised the camps’ conditions and long detention periods.

35. Who has become the first Asian woman cricketer to play hundred T20Is?[A] Sana Mir[B] Bismah Maroo[C] Harmanpreet Kaur[D] Daendra Dottin

Correct Answer: A [Sana Mir]Notes:Sana Mir, the former Pakistan captain, has become the first Asian woman cricketer to play hundred T20Internationals (T20Is). She achieved the feat when she took the field against West Indies during the thirdT20I in Karachi on February 3, 2019. Overall, Mir is the only sixth woman from around the world to play100 T20Is. Daendra Dottin of West Indies tops the list with a total of 109 T20Is to her name. In the Asianlist, Sana is followed by fellow Pakistan cricketer Bismah Maroo (94 T20Is) and India’s HarmanpreetKaur on third position (93 T20Is).

36. Which of the following states has recently declared Indus river dolphin as the state’s aquaticanimal?[A] Haryana[B] Himachal Pradesh[C] Punjab[D] Jammu & Kashmir

Correct Answer: C [Punjab ]Notes:Punjab has recently declared the endangered Indus river dolphin as the state’s aquatic animal. Thedecision was recently taken at a meeting of the state board for wildlife chaired by chief minister CaptainAmarinder Singh. Indus Dolphin is an endangered freshwater aquatic mammal found only in Indian andPakistan in the Beas river. These are brown/grey in colour, but blind and live for at least for 30 years. TheChief Minister also gave approval to declare Kanjli Wetland and holy Kali Bein River as a wildlifeconservation reserve to mark the historic 550th Parkash Purb of Sri Guru Nanak Dev Ji. The sacred KaliBein River was associated with the life of Sri Guru Nanak Dev Ji, who got enlightenment while taking adip in the holy river.

37. Which state government has launched the 2nd phase of Pasupu-Kumkuma scheme?[A] West Bengal[B] Telangana[C] Odisha

sri vishnu charan | [email protected] |

https://t.me/PDF4Examshttps://t.me/IAS201819 https://t.me/PDF4Exams

https://t.me/TheHindu_Zone_official

Page 99: Current Affairs -February 1-15, 2019 · 15-02-2019  · 10, theme: ‘Sadak Suraksha-Jeevan Raksha’ National Testing Agency (NTA) launches mobile app through which students can

Current Affairs [PDF] -February 1-15, 2019

© 2019 GKToday | All Rights Reserved | https://www.gktoday.in 99

[D] Andhra Pradesh

Correct Answer: D [Andhra Pradesh]Notes:On February 2, Andhra Pradesh Chief Minister Nara Chandrababu Naidu launched the 2nd phase ofPasupu-Kumkuma (Turmeric-Kumkum) scheme and welfare pensions. Under the scheme, women whoare a part of Development of Women and Children in Rural Areas (DWCRA) will be given Rs 10,000 inthree instalments. Approx Rs 9400 crore have been granted for the scheme. The welfare pensions forsenior citizens, widows, handloom workers, toddy tappers, fishermen, single women, dappu (one kind ofdrum) artists, cobblers and AIDS patients have been increased from Rs 1000 to Rs 2000, while pensionfor divyangs and eunuchs have been increased from Rs 1500 to Rs 3000. Furthermore, kidney diseasevictims will be given a pension of Rs 3500.

38. What is the theme of the 2019 World Cancer Day (WCD)?[A] Not Beyond Us[B] Debunk the Myths[C] I Am and I Will[D] Cancer Myths – Get the Facts

Correct Answer: C [I Am and I Will]Notes:The World Cancer Day (WCD) is observed every year on 4th of February to spread awareness aboutcancer, its treatment and to encourage methods of its prevention. The aim of the day is to save millionsof preventable deaths each year through education, raising awareness and by pressing governments andindividuals across the world to take action against the deadly disease. This year, in 2019, Union forInternational Cancer Control, which organizes WCD, has launched a new 3-year campaign with thetheme: “I Am and I Will”. The campaign is an empowering call-to-action urging for personalcommitment and represents the power of individual action taken now to impact the future. Over 9.6million People die each year from cancer which is more than HIV/AIDS, malaria and tuberculosiscombined. India has around 2.25 million cases with over 1 lakh new cases being registered every year. In2018, the disease led to nearly 7 lakh deaths. The Indian Council of Medical Research (ICMR) estimatesthat the country is likely to register over 17 lakh new cases and report over 8 lakh deaths by 2020.

39. Alaa Mashzoub, who was recently shot dead, was the renowned novelist of which country?[A] Iraq[B] Afghanistan[C] Pakistan[D] Bangladesh

Correct Answer: A [Iraq]Notes:In Iraq, a renowned novelist & critic Alaa Mashzoub (51) was shot dead by unknown assailants on 2nd ofFebruary in the city of Karbala. He was known as a bold writer who was not afraid to broach political andreligious taboos in his writings. His research into the history of his hometown was published in twoparts as ‘A Summary of Karbala Cultural History’. Mashzoub spoke out against sectarianism and militias.He was known as an outspoken writer who published in a variety of newspapers and wrote severalnovels, including The Chaos of the Nation (2014) Crime on Facebook (2015), and The Jewish Baths (2017).Mashzoub has 20 published books and has written extensively about Karbala’s history and culture. Hehas also written about Iraq’s once-thriving Jewish minority. He had been recognized by a number offilmmaking and writing prizes, including a Katara Prize for Arabic Fiction and a prize for hisdocumentary film ‘Doors and Windows’. In recent times, Iraq has seen a number of politicalassassinations, particularly of outspoken activists. Karbala is about 100 kilometers southwest ofBaghdad.

40. The World Heritage Centre (WHC) would be set up at which of the following states?[A] Chennai[B] Dehradun[C] New Delhi[D] Kolkata

sri vishnu charan | [email protected] |

https://t.me/PDF4Examshttps://t.me/IAS201819 https://t.me/PDF4Exams

https://t.me/TheHindu_Zone_official

Page 100: Current Affairs -February 1-15, 2019 · 15-02-2019  · 10, theme: ‘Sadak Suraksha-Jeevan Raksha’ National Testing Agency (NTA) launches mobile app through which students can

Current Affairs [PDF] -February 1-15, 2019

© 2019 GKToday | All Rights Reserved | https://www.gktoday.in 100

Correct Answer: D [Kolkata]Notes:The World Heritage Centre (WHC) housing ‘spiritual camps’ of 45 countries would be set up at Mayapurin Nadia district, Kolkata. The camp of each country would reflect its culture, dress, lifestyle, food andambiance. The process for setting up the centre got momentum after West Bengal Chief MinisterMamata Banerjee inquired about the project at a recent administrative meeting in Nadia district. Theproposed project would also showcase traditional Bengali handloom and craft as suggested by the chiefminister. The estimated project cost of the proposed centre has been fixed at Rs. 3,000 crore. Mayapur isthe global headquarters of the International Society for Krishna Consciousness (ISKCON).

41. Who is the recipient of the 2019 Kalamaharshi Baburao Painter Award at the 2019 KolhapurInternational Film Festival (KIFF)?[A] Gajendra Ahire[B] Abhijit Deshpande[C] Govind Nihalani[D] Rohini Hattangadi

Correct Answer: C [Govind Nihalani]Notes:An acclaimed cinematographer and screenwriter and filmmaker, Govind Nihalani will be honoured withthe Kalamaharshi Baburao Painter Award for his contribution to the film industry at the 2019 KolhapurInternational Film Festival (KIFF). The 7th edition of the film festival is scheduled to be held fromFebruary 7 to 14. The award is named after Painter, a native of Kolhapur who was a painter, sculptor anda filmmaker of the silent film era. More than 50 feature and short films, including those from Japan,South Korea, France and Iran, among others, will be screened at a well-known multiplex here as part ofthe 2019 KIFF. The country focus will be on Hungarian cinema.

42. Ramesh Bhatkar, who passed away recently, was the renowned actor of which regionalcinema?[A] Marathi[B] Tamil[C] Telugu[D] Odia

Correct Answer: A [Marathi]Notes:Ramesh Bhatkar (70), the veteran Marathi actor, has passed away in Mumbai, Maharashtra on February4, 2019. He was known for his roles in the TV series “Commander” and “Hello Inspector” and worked formore than 30 years as an actor in mainstream Hindi and Marathi films such as “Aai Pahije”, “Kucch ToHai” and “Bhavesh Joshi Superhero”. For the last 30 years, Ramesh has played lead roles in such plays asOoghadale Swargache Daar (1982), Denaryaache Haath Hazaar (1980), Shadyantra (1991), Kevha TariPahate, Akher Tu Yeshilach, Rahu Ketu, Mukta, The Boss- Sutradhar, Kinara.

43. Which city is the venue of the ICT Academy Bridge 2019 conference?[A] Jaipur[B] New Delhi[C] Pune[D] Chennai

Correct Answer: D [Chennai ]Notes:In Tamil Nadu, the 37th edition of the ICT Academy Bridge 2019 conference was held in Chennai withtheme “Fostering India for Industry 4.0″. The event brought together industry experts, academicleaders and government policy-makers to address the challenges faced by India Inc. in the era of FourthIndustrial Revolution & therefore creating a correlated education ecosystem and formulating a strategicregulatory framework. It is the largest Industry-Institute Interaction Event of Asia and is organized byICT Academy. The city witnessed more than 1,000 participants from government, industry andacademia.

sri vishnu charan | [email protected] |

https://t.me/PDF4Examshttps://t.me/IAS201819 https://t.me/PDF4Exams

https://t.me/TheHindu_Zone_official

Page 101: Current Affairs -February 1-15, 2019 · 15-02-2019  · 10, theme: ‘Sadak Suraksha-Jeevan Raksha’ National Testing Agency (NTA) launches mobile app through which students can

Current Affairs [PDF] -February 1-15, 2019

© 2019 GKToday | All Rights Reserved | https://www.gktoday.in 101

44. Which Indian squash player has won the 2019 Seattle Open title?[A] Saurav Ghosal[B] Ramit Tandon[C] Harinder Pal Sandhu[D] Cyrus Poncha

Correct Answer: B [Ramit Tandon]Notes:Indian squash player Ramit Tandon has won the 2019 Seattle Open title by defeating Mohamed ElSherbini of Egypt in the final in USA. It is his fourth title in his career. The Seattle Open is a ProfessionalSquash Association (PSA) challenger squash Tour event.

45. Which state government has launched Kalia Students’ Scholarship Scheme to boost educationfor farmers children?[A] Chhattisgarh[B] Tamil Nadu[C] Assam[D] Odisha

Correct Answer: D [Odisha ]Notes:In Keonjhar district, Odisha Chief Minister Naveen Patnaik has launched the new ‘KALIA ChhatravrittiYojana’ to facilitate the children of the farmers, who are the beneficiaries of the Krushak Assistance forLivelihood and Income Augmentation (KALIA) scheme. Under this scholarship scheme, the beneficiarieswill be provided scholarships for pursuing higher education in professional courses free of cost. Childrenof farmers who have taken admission at Government Professional colleges on merit basis will be eligiblefor the scholarship scheme. The state government has also launched drinking water projects worth Rs1,050 crore to supply safe drinking water in the district. Patnaik also launched Mission Shakti Conclavededicated the development projects worth Rs 45 crore.

46. Which of the following states has become the first Indian state to set up a Price Monitoringand Research Unit (PMRU)?[A] Punjab[B] Kerala[C] Karnataka[D] Telangana

Correct Answer: B [Kerala ]Notes:Kerala has become the first State to set up a Price Monitoring and Research Unit (PMRU) to trackviolation of prices of essential drugs and medical devices under the Drugs Price Control Order (DPCO).The move comes more than 5 years after the National Pharmaceutical Pricing Authority (NPPA)proposed such a system for the States and the Union Territories. The new watchdog will offer technicalhelp to the State Drug Controllers and the NPPA to monitor notified prices of medicines, detect violationof the provisions of the DPCO, look at price compliance, collect test samples of medicines, and collect andcompile market-based data of scheduled as well as non-scheduled formulations.

47. Who won the 2019 El Salvador presidential election?[A] Nayib Bukele[B] Josue Alvarado[C] Hugo Martínez[D] Carlos Calleja

Correct Answer: A [Nayib Bukele]Notes:The media-savvy former mayor of San Salvador, Nayib Bukele of the Grand Alliance for National Unity(GANA) has won a landslide victory in 2019 El Salvador presidential election by winning over 53% of the

sri vishnu charan | [email protected] |

https://t.me/PDF4Examshttps://t.me/IAS201819 https://t.me/PDF4Exams

https://t.me/TheHindu_Zone_official

Page 102: Current Affairs -February 1-15, 2019 · 15-02-2019  · 10, theme: ‘Sadak Suraksha-Jeevan Raksha’ National Testing Agency (NTA) launches mobile app through which students can

Current Affairs [PDF] -February 1-15, 2019

© 2019 GKToday | All Rights Reserved | https://www.gktoday.in 102

total vote. With this, he brought an end to a two-party system that has held sway over the violence-plagued Central American country for three decades. Bukele will succeed Salvador Sánchez Cerén , afterwhom the leadership chair was vacant for a period of time. He will assume office on 1 June 2019.

48. Which space telescope has discovered a new dwarf galaxy ‘Bedin 1’ in the cosmicneighbourhood?[A] Astro 2[B] Astrosat[C] Hisaki[D] Hubble

Correct Answer: D [Hubble ]Notes:The Hubble Space Telescope (HST) has made an unexpected discovery of a new dwarf galaxy in ourcosmic neighborhood. Named Bedin 1, the galaxy is approximately 30 million light-years away and 2million light-years from the nearest plausible large galaxy host ‘NGC 6744’. The object is classified as adwarf spheroidal galaxy because it measures only around 3,000 light-years at its greatest extent, and itis roughly a thousand times dimmer than our own Milky Way Galaxy. The galaxy is named afterdiscovery team leader Dr. L. R. Bedin of the INAF-Osservatorio Astronomico di Padova in Italy. Thedwarf spheroidal galaxies are not uncommon, Bedin 1 has some notable features. Not only is it one of justa few dwarf spheroidals that have a well established distance but it is also extremely isolated. Thismakes it possibly the most isolated small dwarf galaxy discovered to date. The Dwarf spheroidal galaxiesare defined by their small size, low-luminosity, lack of dust and old stellar populations. 36 galaxies of thistype are already known to exist in the Local Group, 22 of which are satellite galaxies of the Milky Way.The researchers used the NASA/ESA Hubble Space Telescope to study white dwarf stars within theglobular cluster NGC 6752. The aim of their observations was to use these stars to measure the age ofthe globular cluster, but in the process they made an unexpected discovery of Bedin 1.

49. INF Treaty, which is in news recently, is in between which of the following countries?[A] Taiwan & China[B] France & Germany[C] United States & Russia[D] Israel & Palestine

Correct Answer: C [United States & Russia]Notes:The Intermediate-Range Nuclear Forces (INF) Treaty is a Cold War-era missile treaty that was a signedin 1987 between the United States and the Soviet Union. Signed by Ronald Reagan and MikhailGorbachev, the treaty barred both countries from deploying land-launched cruise missiles in the 500- to5,500-km range. It was a key to ending the arms race between the (then) two super powers and helpedprotect the U.S.’s NATO allies in Europe from Soviet missile attacks. The deal addressed Soviet nuclear-tipped ballistic missiles targeting Western capitals, but put no restrictions on other major militaryactors such as China. Recently, it is in news because the United States has suspended its obligationsunder the INF Treaty with Russia effective on 2nd February 2019 and will withdraw from the treaty insix months. Article XV of the treaty mandates a six-month notice period before withdrawal. The reasonsfor withdrawal are as follows: the Trump and Obama administrations have repeatedly alleged thatRussia was violating the treaty by fielding a ground-based cruise missile, the Novator 9M729 (“SSC-8” inNATO terminology) that could strike Europe at a short notice, an allegation that Russia has repeatedlydenied. The Russians have raised counter-allegations against the U.S., with regard to launchers forantiballistic missile systems in Europe.

50. Aribam Syam Sharma, who has decided to return his Padma Shri award in protest against theCitizenship (Amendment) Bill of 2016, is from which north-eastern state?[A] Manipur[B] Mizoram[C] Nagaland[D] Tipura

Correct Answer: A [Manipur]

sri vishnu charan | [email protected] |

https://t.me/PDF4Examshttps://t.me/IAS201819 https://t.me/PDF4Exams

https://t.me/TheHindu_Zone_official

Page 103: Current Affairs -February 1-15, 2019 · 15-02-2019  · 10, theme: ‘Sadak Suraksha-Jeevan Raksha’ National Testing Agency (NTA) launches mobile app through which students can

Current Affairs [PDF] -February 1-15, 2019

© 2019 GKToday | All Rights Reserved | https://www.gktoday.in 103

Notes:Manipur-based filmmaker and music composer Aribam Syam Sharma has decided to return his 2006Padma Shri award in protest against the Citizenship (Amendment) Bill, 2016 that the Centre wants topush through. The 83-year-old filmmaker made the decision after families of around 855 martyrs of theanti-foreigners Assam Agitation of 1979-85 returned the citation that the Sarbananda Sonowalgovernment had given them in 2016. The government had also honored these families with Rs 5 lakheach for the sacrifice of the martyrs. The protests have rocked the entire northeast against the proposalto grant citizenship to non-Muslims from neighbouring Muslim-majority countries. The film fraternityof the region backed the decision of Manipur filmmaker Aribam Syam Sharma to return his Padma Shriin protest against the bill. The Bill seeks to fast-track the process of granting citizenship to non-Muslimsfrom Afghanistan, Bangladesh and Pakistan who came to India till December 31, 2014, allegedly becauseof religious persecution.

51. GSAT-31, which is in news recently, is launched from which of the following places?[A] French Guiana[B] Sriharikota[C] Florida[D] Moscow

Correct Answer: A [French Guiana]Notes:India’s latest communication satellite GSAT-31 was successfully launched by European launch servicesprovider- Arianespace’s rocket from Kourou Launch Base in French Guiana on February 6, 2019. TheAriane-5 vehicle (Flight VA247) also carried Saudi Geostationary Satellite 1/Hellas Sat 4 along withGSAT-31. After a 42-min flight, GSAT-31 separated from the Ariane-5 in an elliptical GeosynchronousTransfer Orbit (GTO) with a perigee (nearest point to Earth) of 250 km and an apogee (farthest point toEarth) of 35,850 km, inclined at an angle of 3.0 degree to the equator. After separation from Ariane-5, thetwo solar arrays of GSAT-31 were automatically deployed in quick succession and ISRO’s Master ControlFacility at Hassan in Karnataka took over the command and control of GSAT-31 and found its healthparameters normal. The GSAT-31 is a high power communication satellite with Ku-band and will providecommunication services to Indian mainland and islands. It has an operational life of over 15 years & willoffer seamless services by replacing the ageing INSAT-4CR satellite. GSAT-31 is India’s 40thcommunication satellite.

52. Ladu Kishore Swain, who passed away recently, was the current MP from which Lok Sabhaconstituency of Odisha?[A] Aska[B] Keonjhar[C] Jagatsinghpur[D] Sundargarh

Correct Answer: A [Aska ]Notes:Ladu Kishore Swain (71), the senior Biju Janata Dal (BJD) leader, has passed away in Bhubaneswar on 5February 2019. He was elected to the 16th Lok Sabha in May 2014 from Aska constituency of Odisha.

53. Who is the head of the committee which is constituted by the Maharashtra to review welfareschemes for tribals?[A] Vivek Pandit[B] Vasantrao More[C] A T Nana Patil[D] Sudhakar Wagh

Correct Answer: A [Vivek Pandit ]Notes:The Maharashtra government has set up a 17-member committee to review various schemes beingimplemented for the welfare of tribals in the state. The committee will be headed by former MLA andShramjeevi Sanghatna president Vivek Pandit. The panel will undertake the study of various works donefor providing employment opportunities, minimum wages and proper livelihood to tribals. As per a

sri vishnu charan | [email protected] |

https://t.me/PDF4Examshttps://t.me/IAS201819 https://t.me/PDF4Exams

https://t.me/TheHindu_Zone_official

Page 104: Current Affairs -February 1-15, 2019 · 15-02-2019  · 10, theme: ‘Sadak Suraksha-Jeevan Raksha’ National Testing Agency (NTA) launches mobile app through which students can

Current Affairs [PDF] -February 1-15, 2019

© 2019 GKToday | All Rights Reserved | https://www.gktoday.in 104

recent government order, the committee will suggest measures to ensure that children in tribal regionswere not deprived of education. The committee will hold its meetings every three months and submit itsrecommendations to the government.

54. What is the theme of the 30th National Road Safety Week (NRSW-2019)?[A] Your Security, Safeguards your Family- Be warned on the Roads[B] Road Security a Goal, Not an Intermission[C] Sadak Suraksha-Jeevan Raksha[D] Walk for Road Security

Correct Answer: C [Sadak Suraksha-Jeevan Raksha]Notes:On 4th February, the Ministry of Road Transport and Highways launched the 30th National Road SafetyWeek (NRSW-2019) in New Delhi to raise public awareness about traffic rules and ultimately to reducecasualties due to road accidents. The 2019 theme is “Sadak Suraksha-Jeevan Raksha”. In an event atGandhi Smriti and Darshan Samiti in Rajghat, Union Home Minister Rajnath Singh, External AffairsMinister Sushma Swaraj and Minister for Road Transport and Highways Nitin Gadkari also flagged off amotor car rally to commemorate the 150th Birth Anniversary of Mahatma Gandhi. The motor rally willcover around 7250 km and will travel through those places in India, Bangladesh and Myanmar which arehistorically associated with Gandhiji. The conclusion of the rally will take place at Yangon in Myanmaron the 24th February 2019.

55. Which union ministry has launched Shehri Samridhi Utsav?[A] Ministry of Health and Family Welfare[B] Ministry of Home Affairs[C] Ministry of Urban Development[D] Ministry of Housing & Urban Affairs

Correct Answer: D [Ministry of Housing & Urban Affairs]Notes:On 5th February, the Ministry of Housing & Urban Affairs (MoHUA) has launched Shehri SamridhiUtsav to extend the outreach of DAY-NULM (Deendayal Antyodaya Mission – National UrbanLivelihoods Mission) to the most vulnerable. The event will also showcase its initiatives and facilitateaccess of Self-Help Group (SHG) members to the other government schemes. Through Shehri SamridhiUtsav, SHG members across cities are being linked to national government schemes viz. SwachchhBharat Mission (U), Pradhan Mantri Awas Yojana (U), Pradhan Mantri Ujjwala Yojana (PMUY), PradhanMantri Jan Dhan Yojana (PMJDY), Pradhan Mantri Suraksha Bima Yojana (PMSBY), Pradhan MantriJeevan Jyoti Yojana (PMJJY) and National Nutrition Mission (NNM). The major highlight of Utsav istheNational Exhibition cum sale of SHG products and National Street Food Festival being organized inNew Delhi.

56. Which of the following PSEs has been declared winner under Miniratna Category by Ministryof MSME for exemplary work amongst CPSEs for promotion of SC/ST entrepreneurs?[A] SAIL[B] AAI[C] NFDC[D] IRCTC

Correct Answer: C [NFDC]Notes:The National Film Development Corporation of India (NFDC) has been selected as winner under theMiniratna Category by Ministry of Micro, Small and Medium Enterprises (MSME). This has been done aspart of endeavor by Ministry of MSME to felicitate select Central Public Sector Enterprises (CPSEs) torecognize their exemplary work towards promotion of SC/ST entrepreneurs. As per data uploaded onSambandh Portal , the performance parameters for the felicitation were procurement from SC/STentrepreneurs, number of vendor development programmes for SC/ST and number of SC/STentrepreneurs benefitting. The NFDC, incorporated in the year 1975, was formed by the Government ofIndia (GoI) with the primary objective of planning and promoting an organised, efficient and integrateddevelopment of the Indian film industry.

sri vishnu charan | [email protected] |

https://t.me/PDF4Examshttps://t.me/IAS201819 https://t.me/PDF4Exams

https://t.me/TheHindu_Zone_official

Page 105: Current Affairs -February 1-15, 2019 · 15-02-2019  · 10, theme: ‘Sadak Suraksha-Jeevan Raksha’ National Testing Agency (NTA) launches mobile app through which students can

Current Affairs [PDF] -February 1-15, 2019

© 2019 GKToday | All Rights Reserved | https://www.gktoday.in 105

57. Sentinelese Tribe, which is in news recently, is located in which of the following islands?[A] South Sentinel Island[B] Kondul Island[C] Bompuka[D] North Sentinel Island

Correct Answer: D [North Sentinel Island]Notes:The North Sentinel Island (NSI) is home to the Sentinelese tribe, which have consistently refused anyinteraction with the outside world. They are hostile to outsiders and have killed people who approachedor landed on the island. Sentinelese people are believed to be the only pre-Neolithic people in the world.This means they have a culture that existed in the middle period of Stone Age. Their use of metals is notspecifically known with many believe that they don’t use metals for purposes other than in weapons. Thetribe is in news recently because the Minister of State for Home Affairs Hansraj Gangaram Ahir gaveinformation about the tribe in a written reply to question in the Lok Sabha. The tribe is on the verge ofextinction as per 2011 census; there are a number of laws/regulations to protect them. These are beingstrictly implemented and enforced by the UT Administration. These Laws/Regulation include 1) A &NIslands (PAT) Regulation 1956, 2) Scheduled Castes and the Scheduled Tribes (Prevention of Atrocities)Act, 1989, 3) Restrictions under Foreigner (Restricted Area) Orders, 1963, 4) Visa ManualConditions/Passport Act 1920, and 5) Indian Forest Act, 1927 and Wildlife (Protection) Act, 1972. The NSIis one of the Andaman Islands, an archipelago in the Bay of Bengal which also includes South SentinelIsland.

58. The joint HADR Exercise ‘RAHAT-2019’ will be conducted in which of the following states?[A] Madhya Pradesh[B] Jharkhand[C] Rajasthan[D] Uttar Pradesh

Correct Answer: C [Rajasthan]Notes:In Rajasthan, the joint Humanitarian Assistance and Disaster Relief (HADR) Exercise ‘RAHAT-2019’ willbe conducted by Jaipur based Sapta Shakti Command on behalf of Indian Army from 11th February. The2-day exercise will be conducted simultaneously in three places viz. Jaipur in the form of a table topexercise and at Kota and Alwar on-ground capability and coordination. The Joint exercise incoordination with NDMA is being conducted to synergise efforts for humanitarian assistance anddisaster relief operations. Representatives from Armed Forces, National Disaster Management ResponseMechanism (NDMRM), SDMA Rajasthan and DLMAs will be participating in the exercise.

59. Which high court has ruled against negative marking in competitive examinations?[A] Delhi High Court[B] Calcutta High Court[C] Madras High Court[D] Bombay High Court

Correct Answer: C [Madras High Court]Notes:In a huge relief to a number of students who fail to qualify to enter into prestigious institutions like IITsdue to negative marking, the Madras High Court ruled against negative marking in competitiveexaminations saying they are bad in law. The Court stated that the practice of awarding negative markswill eradicate intelligent guessing in competitive exams. The judgment was passed while disposing of awrit petition filed in 2013 by an IIT JEE aspirant S. Nelson Prabhakar (a Scheduled Caste student), whofailed to clear the Mains due to negative marking.

60. Dakshayani, which is in news recently, is a/an _____?[A] Space Center[B] Novel[C] Animal

sri vishnu charan | [email protected] |

https://t.me/PDF4Examshttps://t.me/IAS201819 https://t.me/PDF4Exams

https://t.me/TheHindu_Zone_official

Page 106: Current Affairs -February 1-15, 2019 · 15-02-2019  · 10, theme: ‘Sadak Suraksha-Jeevan Raksha’ National Testing Agency (NTA) launches mobile app through which students can

Current Affairs [PDF] -February 1-15, 2019

© 2019 GKToday | All Rights Reserved | https://www.gktoday.in 106

[D] Religious tradition

Correct Answer: C [Animal]Notes:Dakshayani, an 88-year-old captive she-elephant in Kerala, has died at a care centre at nearbyPappanamcode in Thiruvananthapuram, Kerala on 5th February 2019. She was the oldest captiveelephant in Asia and Guinness record holder. She was also called the ‘Gaja Muthassi’ (grandmother ofelephants). In 2016, she was conferred the “Gaja Muthassi” (elephant granny) title and entered theGuinness book of records. The Postal department had also brought out a postal cover on the elephant.Dakshayini had been a resident at the Chengallur Mahadeva temple in Thiruvananthapuram for the last70 years and was owned by Travancore Devaswom Board (TDB).

61. Which city is the venue of the 2nd National Conference of Micro Missions of National PoliceMission (NPM)?[A] Pune[B] Hyderabad[C] Bangaluru[D] New Delhi

Correct Answer: D [New Delhi]Notes:The 2nd National Conference of Micro Missions of National Police Mission (NPM) has started in NewDelhi on 7th of February 2019, which is organized by Bureau of Police Research & Development (BPR&D).The two day Conference will discuss the issues of Skills and Competence at the grass-root level,Attitudinal Changes in Police, Gender Sensitization, harnessing technology and Community Policing. Amega-d version of NPM Compendium on the projects developed by the Mission will also be released. TheNPM awards will be given to 09 officers for their contribution in Micro Missions and successfulpreparation of the implementable projects. NPM Division of BPRD strives to transform the Police Forcesof the country into an effective instrument for the maintenance of Internal Security. To achieve themandate of NPM, 8 micro missions are working under BPRD. BPRD organizes Conference on NationalPolice Mission annually.

62. The Rashtriya Kamdhenu Aayog (RKA) would be set up for ___?[A] Conservation of Cows[B] Protection of Cows[C] Progeny of Cows[D] All of the above

Correct Answer: D [All of the above]Notes:The Union Cabinet has approved the proposal for the establishment of Rashtriya Kamdhenu Aayog(RKA) for conservation, protection and development of cows and their progeny. It will lead to thedevelopment of the cattle population in the country including the development and conservation ofindigenous breeds. It will result in the growth of the livestock sector which is more inclusive, benefitwomen, and small and marginal farmers. The Aayog will work in collaboration with Veterinary, AnimalSciences or Agriculture University or departments or organizations of the Central or State Governmentengaged in the task of research in the field of breeding and rearing of the cow, organic manure, biogas,etc. Rashtriya Kamdhenu Aayog was announced in the Union Budget 2019-20 on the 1st of this month.

63. Which project has been launched by Union Government to study Apparel Consumption inIndia?[A] India Apparel[B] India Size[C] India Attire[D] India Outfit

Correct Answer: B [India Size]Notes:In Mumbai, the India Size project has recently launched by the Union Minister of Textiles Smriti Irani.

sri vishnu charan | [email protected] |

https://t.me/PDF4Examshttps://t.me/IAS201819 https://t.me/PDF4Exams

https://t.me/TheHindu_Zone_official

Page 107: Current Affairs -February 1-15, 2019 · 15-02-2019  · 10, theme: ‘Sadak Suraksha-Jeevan Raksha’ National Testing Agency (NTA) launches mobile app through which students can

Current Affairs [PDF] -February 1-15, 2019

© 2019 GKToday | All Rights Reserved | https://www.gktoday.in 107

It’s a first-of-its-kind project in the history of the country, which aims to arrive at a standard Indian Sizefor the ready-to-wear clothing industry, on the lines of the standardized sizes available in countries suchas the USA, the UK and Mexican. A size chart that is specific to Indian consumers’ measurements will bedeveloped. The project is being undertaken by the Ministry of Textiles, in collaboration with TheClothing Manufacturers Association of India (CMAI). The Minister also launched a Study of ApparelConsumption in India. The report of this study would be ready for release by July 2019. Under India Size,an all-India anthropometric study of measurements of the average Indian apparel consumer will beundertaken. The study will include an analysis of close to 25,000 consumers spread over various agegroups, and covering the cities of Mumbai, Delhi, Chennai, Hyderabad, Kolkata, and Shillong. This willenable the study to cover all important regions of the country.

64. Which country has recently signed NATO accession agreement?[A] Romania[B] Croatia[C] Macedonia[D] Kosovo

Correct Answer: C [Macedonia ]Notes:On 6th February, Macedonia signed an accord to join NATO to become the 30th member of the US-ledalliance. The admittance of a new member highlighted that all European nations who meet the entrancecriteria could join the regional alliance. However, Moscow has continually rejected NATO’s position,saying that by taking in Balkan members, the alliance is undermining security in the region.TheMacedonia-NATO accord follows a deal with Greece ending a 27-year-old dispute over Macedonia’sname. The accord must now be ratified by allied governments. The move followed a deal betweenMacedonia and NATO-member Greece under which Macedonia agreed to change its name to theRepublic of North Macedonia. North Macedonia will formally join the alliance in 2020.

65. The 462nd Kandoori festival 2019 has celebrated in which of the following states?[A] Tamil Nadu[B] Andhra Pradesh[C] Kerala[D] Karnataka

Correct Answer: A [Tamil Nadu]Notes:In Tamil Nadu, the 462nd annual Kandoori festival 2019 has started at nearby Nagore Dargah, a famousMuslim pilgrim centre in Nagore. The 14-day long festival began on 6th February after the holy flag wasbrought in a procession and hoisted. The Kandoori festival is held to commemorate the deathanniversary of 16th century saint Hazrath Syed Shahul Hameed Quadir Wali, who migrated from hisnative Manickapur near Allahabad in Uttar Pradesh and lived in Nagore till his death. As per tradition,one of the five flags was brought from Singapore. The main event of the festival-‘Sandanakoodu'(sandalpot) procession- will be held on February 15. The Dargah of Saint Hazrath Syed Shahul Hameed inNagore is more than 500 years old. Saint Hazrath Syed Shahul Hameed came to Thanjavur on apilgrimage during the 16th century AD when he cured the illness of the then King of ThanjavurAchuthappa Naicken. The King gifted the saint lands in Nagore where the latter lived for the rest of hislife serving the people, irrespective of religion, caste or creed.

66. What is the current repo rate, as per RBI’s 6th bi-monthly monetary policy statement for2018-19?[A] 6.0%[B] 6.75%[C] 6.50%[D] 6.25%

Correct Answer: D [6.25%]Notes:The six-member Monetary Policy Committee (MPC), headed by RBI Governor Shaktikanta Das,announced the sixth bi-monthly monetary policy statement for 2018-19. In it, the RBI has cut the key

sri vishnu charan | [email protected] |

https://t.me/PDF4Examshttps://t.me/IAS201819 https://t.me/PDF4Exams

https://t.me/TheHindu_Zone_official

Page 108: Current Affairs -February 1-15, 2019 · 15-02-2019  · 10, theme: ‘Sadak Suraksha-Jeevan Raksha’ National Testing Agency (NTA) launches mobile app through which students can

Current Affairs [PDF] -February 1-15, 2019

© 2019 GKToday | All Rights Reserved | https://www.gktoday.in 108

policy rate under the liquidity adjustment facility (LAF) by 25 basis points to 6.25% from 6.5% withimmediate effect. Consequently, the reverse repo rate stands adjusted to 6.0%, and the MarginalStanding Facility (MSF) rate and the Bank Rate to 6.5%. The MPC also decided to change the monetarypolicy stance from calibrated tightening to neutral. These decisions are in consonance with the objectiveof achieving the medium-term target for consumer price index (CPI) inflation of 4 per cent within a bandof +/- 2 per cent, while supporting growth. The bank also projected an economic growth rate of 7.4% forFY20, up from 7.2% estimated for the current fiscal by Central Statistics Office (CSO). This is the firstpolicy review, post the appointment of RBI Governor Shaktikanta Das and also the first post the interimbudget. The next meeting of the MPC is scheduled from April 2 to 4, 2019.

67. Which city was the venue of the ‘Parmanu Tech 2019’?[A] Kanpur[B] New Delhi[C] Udaipur[D] Dehradun

Correct Answer: B [New Delhi]Notes:The “Parmanu Tech 2019” Conference was organised by the Ministry of External Affairs (MEA) andDepartment of Atomic Energy (DAE) at New Delhi on February 6. The conference discussed issuesrelated to Nuclear Energy and Radiation Technologies. Dr Jitendra Singh, the Union Minister of State forAtomic Energy and Space, delivered the keynote address at the conference. During the conference sectorspecific sessions were held with Panel discussions on the following topics: 1) Health care: NuclearMedicine and Radiation therapy – Care to cure, 2) Food Preservation, Agricultural & IndustrialApplications: From farms to factories – Serving the National Cause and 3) Showcasing India’scapabilities in Nuclear Energy: Energy Security with environmental responsibility – the Onward March.

68. Which state government has launched the Zero Fatality Corridor (ZFC)?[A] Punjab[B] Assam[C] Delhi[D] Odisha

Correct Answer: C [Delhi ]Notes:The Arvind Kejriwal-led Delhi government has recently launched the ‘Zero Fatality Corridor’ (ZFC) toexamine the cause for accidents and crashes on the stretch between Burari Chowk and Bhalswa chowk,located on the outer ring road. The aim of the pilot project is to reduce deaths to near zero levels on thisparticular stretch. The project will entail a scientific study to reduce the conflict involving pedestriansand cyclists on one hand and the cars and heavy vehicles on the other. The initiative is to be run inpartnership with SaveLIFE Foundation, a leading road safety non-profit organization.

69. Which country has official declared Siamese fighting fish as its national aquatic animal?[A] Thailand[B] Indonesia[C] Vietnam[D] Malaysia

Correct Answer: A [Thailand ]Notes:Thailand has official declared the Siamese fighting fish as its national aquatic animal to boostconservation efforts and commercial breeding in the country. Known in the West as a betta fish, theunderwater brawler is popular in home aquariums for its iridescent body and many-hued tail. It bringsup feelings of nostalgia for older generations in Thailand – which changed its name from Siam in 1939.They remember a time when the fish were traded and sparred in childhood games. The Siamese fightingfish is listed as Vulnerable by the IUCN. Thailand’s official national animal is the elephant.

70. The 52nd Boori Boot Yollo festival 2019 is celebrated by which tribe of Arunachal Pradesh?[A] Singhpo

sri vishnu charan | [email protected] |

https://t.me/PDF4Examshttps://t.me/IAS201819 https://t.me/PDF4Exams

https://t.me/TheHindu_Zone_official

Page 109: Current Affairs -February 1-15, 2019 · 15-02-2019  · 10, theme: ‘Sadak Suraksha-Jeevan Raksha’ National Testing Agency (NTA) launches mobile app through which students can

Current Affairs [PDF] -February 1-15, 2019

© 2019 GKToday | All Rights Reserved | https://www.gktoday.in 109

[B] Mishmi[C] Nyishi[D] Apatani

Correct Answer: C [Nyishi ]Notes:The 52nd Boori Boot Yollo festival 2019 has recently celebrated by the Nyishi tribe of Arunachal Pradesh.The fest is observed in the month of February every year to welcome spring and a successful harvest.During this festival, the consecrated soul is summoned to sanctify the people. Apart from it, it is anoccasion of mass prayer for prosperity eliminating the possibility of any fatal ailments. The strikingfeature here is the giant assemblage of people cutting across the barriers of cast, creed, age and sex. Theyoung ones take the lead to be guided by the elders where Nibu (Priest) would perform the rituals. Thefestival has an anthem which speaks of the sun rising, planets, mighty rivers, flora and fauna. A portionof the lyric deals with the earth which keeps on rotating around the sun along with other heavenlybodies.

71. Which of the following cricket teams has clinched the 2019 Ranji Trophy?[A] Vidharbha[B] Saurashtra[C] Bengal[D] Railways

Correct Answer: A [Vidharbha ]Notes:On 7th February, Vidharbha lifted the Ranji Trophy for the second time in a row, after defeatingSaurashtra by 78 runs in the final match of the 2018–19 season held at Vidarbha Cricket AssociationStadium, Nagpur. With this, Vidarbha has become only the 6th team in the tournament’s long history tohave defended the title.

72. Which city is the venue of 9th Senior Women’s Hockey National Championship 2019?[A] Balaghat[B] Hisar[C] Jaipur[D] New Delhi

Correct Answer: B [Hisar]Notes:The 9th Senior Women’s Hockey National Championship 2019 has started in Hisar, Haryana fromFebruary 8. Top India stars will turn up for their respective state units and employers for thetournament. RSPB will begin their campaign against Hockey Rajasthan in the Pool A opening match.Twenty teams will participate in the 11-day domestic extravaganza. Teams that take the top two spots inPools A, B, C and D in the round-robin stage will qualify for the quarterfinals to be played on February 15.The semifinal and final will be played on February 17 and February 18, respectively.

73. Who of the following Indian female cricketers has scored fastest T20I fifty?[A] Harmanpreet Kaur[B] Smriti Mandhana[C] Ekta Bisht[D] Nagarajan Niranjana

Correct Answer: B [Smriti Mandhana]Notes:On 6th February 2019, Opener Smriti Mandhana hammered the fastest fifty for India in the first T20Iagainst New Zealand in Wellington in just 24 deliveries. She broke her own record of fastest T20I fiftyfor India in women’s cricket, which she had achieved against England in Mumbai in 2018. Though, SmritiMandhana’s 58 and Jemimah Rodrigues’ 39 went in vain as the Indian eves were bundled out for 136 in19.1 overs. New Zealand Women defeated India by 23 runs in the first T20I. The overall record for thefastest fifty in women’s T20Is belongs to Sophie Devine of New Zealand who hit a half-century in 18 balls

sri vishnu charan | [email protected] |

https://t.me/PDF4Examshttps://t.me/IAS201819 https://t.me/PDF4Exams

https://t.me/TheHindu_Zone_official

Page 110: Current Affairs -February 1-15, 2019 · 15-02-2019  · 10, theme: ‘Sadak Suraksha-Jeevan Raksha’ National Testing Agency (NTA) launches mobile app through which students can

Current Affairs [PDF] -February 1-15, 2019

© 2019 GKToday | All Rights Reserved | https://www.gktoday.in 110

against India during a T20 International back in 2015.

74. The Robert Bosch Center for Data Science and Artificial Intelligence (RBC-DSAI) hasestablished at which IIT?[A] IIT Madras[B] IIT Kanpur[C] IIT Kharagpur[D] IIT Bombay

Correct Answer: A [IIT Madras]Notes:The Robert Bosch Center for Data Science and Artificial Intelligence (RBC-DSAI) has recentlyinaugurated at the IIT Madras with an objective of advance scientific innovation for societal benefit. TheRBC-DSAI will undertake foundational research in many areas of AI and Data Science viz. deep learning,reinforcement learning, network analytics, interpretable machine learning, and domain aware AI. Thecentre is opened by Germany-based Bosch in collaboration with IIT-Madras. Bosch will invest around Rs20 crore (2.5 million euros) over five years in the centre.

75. What is the India’s rank at the 2019 US Chambers International IP Index?[A] 52nd[B] 14th[C] 36th[D] 43rd

Correct Answer: C [36th]Notes:The Global Innovation Policy Center (GIPC) of the US Chambers of Commerce has recently released itsInternational IP Index for Year 2019. In it, India has moved up 8 places to rank 36th among 50 economiesin the 2019 Index.The improvement reflects important reforms implemented by Indian policymakerstowards building and sustaining an innovation ecosystem for domestic entrepreneurs and foreigninvestors alike. The report also focuses on thorny issues. According to the report, the key problems inIndia include barriers to licensing and technology transfer, strict registration requirements, limitedframework for the protection of bio-pharmaceutical IP rights, patentability requirements outsideinternational standards, lengthy pre-grant opposition proceedings and previously used compulsorylicensing for commercial and non-emergency situations. The index ranks countries based on 45indicators covering patent, trademark, copyright and trade secrets protection.

76. The India Heritage Walk Festival (IHWF-2019) has organized by Sahapedia in partnership withwhich international organization?[A] ILO[B] UNESCO[C] UNICEF[D] FAO

Correct Answer: B [UNESCO]Notes:The 2nd edition of India Heritage Walk Festival (IHWF-2019) is a one-of-its-kind month-long festivalcovering 37 cities has started from February 2. It is organized by Sahapedia in partnership withUNESCO. The fest is also supported by the National Mineral Development Corporation (NMDC). Thisfestival offers the opportunity to discover the hidden gems linked to history. The primary objective ofthe festival is to widen access to various aspects of India’s tangible and intangible heritage. Through 87heritage walks and over 100 outreach events, the event will explore the cultural fabric of India, focusingon museums, historically significant monuments and markets, natural landscapes, areas known for richcuisine, and locations that are rooted in women-oriented narratives. For this edition, Sahapedia hascollaborated with about 40 local-level partners to conduct its pan-India heritage walk festival. Theyinclude the Kerala History Museum, The Kochi Heritage Project, INTACH Srinagar Chapter, Art DecoMumbai, Maharana of Mewar Charitable Foundation and Oxford Bookstore.

77. What is the theme of the Asia LPG 2019 Summit?

sri vishnu charan | [email protected] |

https://t.me/PDF4Examshttps://t.me/IAS201819 https://t.me/PDF4Exams

https://t.me/TheHindu_Zone_official

Page 111: Current Affairs -February 1-15, 2019 · 15-02-2019  · 10, theme: ‘Sadak Suraksha-Jeevan Raksha’ National Testing Agency (NTA) launches mobile app through which students can

Current Affairs [PDF] -February 1-15, 2019

© 2019 GKToday | All Rights Reserved | https://www.gktoday.in 111

[A] LPG – Safe for Life[B] LPG – Energy for Life[C] LPG – Spirit for Life[D] LPG – Reliable for Life

Correct Answer: B [LPG – Energy for Life]Notes:The 2nd edition of the Asia LPG Summit 2019 was held in New Delhi with theme ‘LPG – Energy for Life’.The summit was organized by the World LPG Association (WLPGA) in association with major Indianpublic-sector oil companies (OMCs) Indian Oil, Hindustan Petroleum, and Bharat Petroleum. In it, UnionPetroleum Minister Dharmendra Pradhan stated that India has become the world’s second largestimporter of Liquefied Petroleum Gas (LPG) as its government pushes cleaner alternatives to traditionalcooking fuels such as firewood and cow dung. The imports of LPG grew 12.5% over the past five years to12 million metric tons (13 million tons) in 2018-19. Hence, in terms of LPG import, India has surpassedJapan to acquire 2nd spot. China is at first spot. India imports LPG mainly from Saudi Arabia, Qatar, theUnited Arab Emirates (UAE), Kuwait and Iran. The summit offered a unique opportunity to the globalLPG industry to interact with development agencies, NGOs and non-profit organizations who havefacilitated last-mile access to LPG for the beneficiaries.

78. Dard Aryan Tribe, which is in news recently, is from which state/UT?[A] Arunachal Pradesh[B] J&K[C] Jharkhand[D] Himachal Pradesh

Correct Answer: B [J&K]Notes:The Dard Aryan Tribe of Jammu and Kashmir’s Ladakh region known for its liberal customs and ornatecostumes is struggling to keep its vibrant cultural legacy alive. Recently, at the 2019 Dard Aryan Festival(Aryan Utsav) in New Delhi, the delegation of the Dard Aryans has requested the Central government tomake serious interventions. The rich heritage of the Dard Aryans — believed to be the originaldescendants of ‘pure Aryan race’ — is under threat due to rapid modernisation, migration and religiousconversion. The Dard Aryans inhabit Dha, Hanu, Beema, Darchik and Garkone villages in Leh and Kargildistricts. The villages are together called the Aryan valley. The people of this region have unique physicalfeatures, social life, ethnic culture and language. Many researchers believe that the ‘Aryans of Ladakh’ orthe ‘Brokpas’ were a part of Alexander’s army and had come to the region over 2,000 years ago. Theword ‘Dard’ is derived from a Sanskrit word, ‘Daradas’, which refers to people who live on hillsides.

79. The circuit bench of the Calcutta High Court will be set up in which of the following districtsof West Bengal?[A] Malda[B] Hooghly[C] Darjeeling[D] Jalpaiguri

Correct Answer: D [Jalpaiguri]Notes:The Union Cabinet has recently approved the establishment of Circuit Bench of Calcutta High Court atJalpaiguri. Te proposed bench will have jurisdiction over four districts namely Darjeeling, Kalimpong,Jalpaiguri and Cooch Behar. The decision comes in the backdrop of the decision of the Calcutta HighCourt Full Court Meeting in 1988. The Union cabinet had also approved the move in June 2006. A circuitbench is for territories which are far flung, but do not have too many matters to justify a full-fledgedpermanent bench. The Calcutta High Court is the oldest High Court in India. It has jurisdiction over thestate of West Bengal and the Union Territory of the Andaman and Nicobar Islands. The High Courtbuilding’s design is based on the Cloth Hall, Ypres, in Belgium.

80. Which warship of the Indian Navy participated in the multinational maritime exercise ‘CutlassExpress 2019’?

sri vishnu charan | [email protected] |

https://t.me/PDF4Examshttps://t.me/IAS201819 https://t.me/PDF4Exams

https://t.me/TheHindu_Zone_official

Page 112: Current Affairs -February 1-15, 2019 · 15-02-2019  · 10, theme: ‘Sadak Suraksha-Jeevan Raksha’ National Testing Agency (NTA) launches mobile app through which students can

Current Affairs [PDF] -February 1-15, 2019

© 2019 GKToday | All Rights Reserved | https://www.gktoday.in 112

[A] INS Talwar[B] INS Trishul[C] INS Trikand[D] INS Tarkash

Correct Answer: C [INS Trikand]Notes:INS Trikand (F51), a Talwar-class frigate of the Indian Navy, participated in the 8th edition ofmultinational maritime exercise ‘Cutlass Express 2019’. INS Trikand is commanded by Captain SrinivasMaddula and is equipped with a versatile range of sensors and weapons enabling her to address threatsin all the three dimensions – air, surface and sub-surface. The exercise was held near Djibouti,Mozambique and the Seychelles from 27 Jan to 06 Feb 19. The aim of the exercise was to improve lawenforcement capacity, promote regional security and progress inter-operability between the armedforces of the participating nations for the purpose of interdicting illegal maritime activity in theWestern Indian Ocean. The 15 participating nations in this iteration of Cutlass Express are Canada,Comoros, Djibouti, France, India, Kenya, Madagascar, Mauritius, Mozambique, Portugal, Seychelles,Somalia, Tanzania, The Netherlands and the United States.

81. Which international organisation has recently called for a total ban on nuclear weapons?[A] Red Cross[B] UNESCO[C] UNSC[D] Interpol

Correct Answer: A [Red Cross]Notes:On 8th of February 2019, the Red Cross called for a total ban on nuclear weapons & also gave warning ofthe growing risk that such arms could again be used with devastating effect. For this burning issue, theInternational Committee of the Red Cross (ICRC) and the International Federation of Red Cross and RedCrescent Societies (IFRC) launched a global campaign to raise awareness about the rising nuclear threatfacing the world. The notonukes.org campaign comes after the United States and Russia announced toabandon the 1987 Intermediate-Range Nuclear Forces (INF)Treaty. The campaign aims to encouragepeople to lobby their governments to sign and ratify the UN Treaty on the Prohibition of NuclearWeapons, which has so far been signed by 70 nations and ratified by 21.

82. Albert Finney, who passed away recently, was the Oscar-nominated actor of which country?[A] France[B] England[C] Germany[D] Italy

Correct Answer: B [England]Notes:Oscar-nominated British actor Albert Finney (82) has passed away in London on 7th of February 2019.He was a five-time Oscar nominee who began his career at the Royal Shakespeare Company beforemaking his mark in film. He is known for his roles in Saturday Night and Sunday Morning (also 1960),Tom Jones (1963), Two for the Road (1967), Scrooge (1970), Annie (1982), The Dresser (1983), Miller’sCrossing (1990), etc.

83. Jeremy Lalrinnunga is related to which sports?[A] Boxing[B] Judo[C] Weightlifting[D] Wrestling

Correct Answer: C [Weightlifting]Notes:Weightlifter Jeremy Lalrinnunga from Mizoram has recently clinched silver in men’s 67kg category &

sri vishnu charan | [email protected] |

https://t.me/PDF4Examshttps://t.me/IAS201819 https://t.me/PDF4Exams

https://t.me/TheHindu_Zone_official

Page 113: Current Affairs -February 1-15, 2019 · 15-02-2019  · 10, theme: ‘Sadak Suraksha-Jeevan Raksha’ National Testing Agency (NTA) launches mobile app through which students can

Current Affairs [PDF] -February 1-15, 2019

© 2019 GKToday | All Rights Reserved | https://www.gktoday.in 113

fetched India’s second medal in the EGAT’s Cup International Weightlifting Championships in ChiangMai, Thailand. Lalrinnunga lifted 131kg in the snatch and 157kg in clean and jerk for a total of 288 kg.Earlier, in the on-going event, world champion Saikhom Mirabai Chanu had won a gold in the women’s48-kilogram category with a total lift of 192 kilograms.

84. Who is the author of the book “Let’s Talk On Air: Conversations with Radio Presenters”?[A] Hrishikay Kannan[B] Ameen Sayani[C] Anuraag Pandey[D] Rakesh Anand Bakshi

Correct Answer: D [Rakesh Anand Bakshi]Notes:The book titled “Let’s Talk On Air: Conversations with Radio Presenters” has been authored by RakeshAnand Bakshi, which serves as a comprehensive guide on radio-presenting in India. The new bookcelebrates the lives of 14 popular radio presenters in India – from the legendary Ameen Sayani topresent-day RJs who have been entertaining us via the airwaves. Besides Sayani, among those RJsfeatured in the book are Yunus Khan, Rohini, Mamta Singh, Sayema, Hrishikay Kannan, Anuraag‘picture’ Pandey and Anmol. This book can help shed light on intricacies of radio as a medium and theRJs behind the microphones, connect fans to their favourite RJs and, eventually attract more talent to bepart of the industry. The book, published by Penguin Random House, will be launched on World RadioDay (WRD-2019) on February 13 in Mumbai.

85. Who has been appointed as the new Union Sports Secretary?[A] Rahul Bhatnagar[B] Radheyshyam Julaniya[C] Vinod Sahay[D] Preetam Kumar

Correct Answer: B [Radheyshyam Julaniya]Notes:Radheyshyam Julaniya, a 1985 batch officer of Madhya Pradesh cadre, has been appointed the new UnionSports Secretary. He succeeded Rahul Bhatnagar, who is set to join the Ministry of Panchayati Raj.Earlier, Bhatnagar had also served as Director General of Sports Authority of India (SAI) and the NationalAnti-Doping Agency (NADA). During his tenure, IIM Rohtak has also started a MBA programme inSports Management with Ministry funding. Bhatnagar also outlined the achievements of the SportsMinistry during his tenure, which includes, opening the National Sports University (NSU), Khelo IndiaYouth Games (KIYG) besides hosting important international tournaments like the Hockey World Cup inBhubaneswar and World Women’s Boxing Championships in New Delhi.

86. Father Francois Laborde, who has been conferred the France’s highest civilian honour, is fromwhich state?[A] West Bengal[B] Assam[C] Goa[D] Maharashtra

Correct Answer: A [West Bengal]Notes:Father Francois Laborde, a 92-year-old Bengal priest, has been conferred the Légion d’Honneur (Legionof Honour), the highest civilian award of France, in recognition of his work for specially-abled children.Father Laborde is of French-origin but he has been given Indian citizenship. He is the third person fromthe state to have been awarded the Legion of Honor after film maestro Satyajit Ray and actor SoumitraChatterjee.

87. Which of the following will be the official mascot of 13th COP of the Convention on theConservation of Migratory Species of Wild Animals (COP CMS 13)?[A] Great hornbill

sri vishnu charan | [email protected] |

https://t.me/PDF4Examshttps://t.me/IAS201819 https://t.me/PDF4Exams

https://t.me/TheHindu_Zone_official

Page 114: Current Affairs -February 1-15, 2019 · 15-02-2019  · 10, theme: ‘Sadak Suraksha-Jeevan Raksha’ National Testing Agency (NTA) launches mobile app through which students can

Current Affairs [PDF] -February 1-15, 2019

© 2019 GKToday | All Rights Reserved | https://www.gktoday.in 114

[B] Yellow-footed green pigeon[C] Indian roller[D] Great Indian Bustard

Correct Answer: D [Great Indian Bustard]Notes:The 13th Conference of Parties (COP) of the Convention on the Conservation of Migratory Species ofWild Animals (CMS 13) will be held at 15th to 22nd February 2020 at Gandhinagar in Gujarat. Recently,the Government of India (GoI) has announced that the Great Indian Bustard (GIB) will be its mascot forCMS COP-13. It is an environmental treaty under the aegis of United Nations Environment Programme(UNEP). Representatives from 129 Parties and eminent conservationists and international NGOsworking in the field of wildlife conservation are expected to attend the COP. India has been a Party to theCMS since 1983. The Conference of Parties (COP) is the decision-making organ of this convention. CMS,also known as the Bonn Convention, recognizes that States must be the protectors of migratory speciesthat live within or pass through their national jurisdictions and aims to conserve terrestrial, marine, andavian migratory species throughout their ranges.

88. On which date, the National Deworming Day (NDD-2019) is observed in India?[A] February 10[B] February 11[C] February 09[D] February 08

Correct Answer: A [February 10]Notes:The National Deworming Day (NDD) is an initiative of Union Ministry of Health and Family Welfare(MoHFW) to make every child in the country worm free. The day is observed every year on 10th ofFebruary to tackle worm infections among pre-school (anganwadi) and school-age children across India.On 8th of February 2019, Union Health Ministry conducted its 8th round of NDD across the country toreduce the prevalence of parasitic intestinal worms among children. During the programme,Albendazole tablets were orally administered to pre-school and school-going children in the age group of1 to 19 years. As per World Health Organization (WHO) report, 64% of the Indian population less than 14years are at risk of Soil-Transmitted Helminths (STH) infections. During this round, over 24 CroreChildren and Adolescents will be covered.

89. India has signed pact with which neighboring country to train Civil Servants?[A] Nepal[B] Bangladesh[C] Bhutan[D] Myanmar

Correct Answer: B [Bangladesh ]Notes:India has signed a MoU with Bangladesh to train 1800 Bangladesh Civil Servants at the National Centrefor Good Governance (NCGG) over the next 6 years. They will be trained in e-governance and servicedelivery, public policy and implementation, information technology, decentralization, urbandevelopment and planning, ethics in Administration and challenges in implementation of SustainableDevelopment Goals (SDGs). This is the second time that NCGG signed an MOU for a training programfor Bangladesh Civil Servants. Under the first MOU signed 5 years ago, 1500 Bangladesh Civil Servantshave already been trained in NCGG. The NCGG is an autonomous institute under the aegis ofDepartment of Administrative Reforms and Public Grievances (DAR&PG), Government of India. Its headoffice is at New Delhi. It’s a think tank for governance & policy reforms.

90. The Union Cabinet has approved to set up an unified authority for regulating all financialservices in which organisation?[A] RBI[B] SEBI[C] IFSC

sri vishnu charan | [email protected] |

https://t.me/PDF4Examshttps://t.me/IAS201819 https://t.me/PDF4Exams

https://t.me/TheHindu_Zone_official

Page 115: Current Affairs -February 1-15, 2019 · 15-02-2019  · 10, theme: ‘Sadak Suraksha-Jeevan Raksha’ National Testing Agency (NTA) launches mobile app through which students can

Current Affairs [PDF] -February 1-15, 2019

© 2019 GKToday | All Rights Reserved | https://www.gktoday.in 115

[D] IRDAI

Correct Answer: C [IFSC]Notes:The Union Cabinet has recently approved establishment of a unified authority for regulating all financialservices in International Financial Services Centres (IFSCs) in India through IFSCs Authority Bill of 2019.The establishment of a unified financial regulator for IFSCs will result in providing world-classregulatory environment to market participants from an ease of doing business perspective. This willprovide a stimulus for further development of IFSCs in India. Currently, the banking, capital marketsand insurance sectors in IFSC are regulated by multiple regulators — the RBI, SEBI and IRDAI,respectively. An IFSC caters to customers outside the jurisdiction of the domestic economy. Suchcentres deal with flows of finance, financial products and services across borders. It provides businessand regulatory environment that is comparable to other leading international financial centres in theworld like London and Singapore. IFSC provides Indian companies easier access to global financialmarkets. The first IFSC in India was set up at GIFT City in Gandhinagar, Gujarat.

91. Which city is the venue of the 2nd ASEAN-India Youth Summit 2019?[A] Pune[B] Hyderabad[C] Guwahati[D] Jaipur

Correct Answer: C [Guwahati]Notes:The 2nd ASEAN-India Youth Summit 2019 was held in Guwahati, Assam on the theme “Connectivity isthe pathway to shared prosperity” from 03-06 February 2019. It is organized by India Foundation incollaboration with Union Ministry of External Affairs (MEA) and Government of Assam. Over 100 youthdelegates the ten ASEAN countries – Brunei, Cambodia, Indonesia, Lao PDR, Malaysia, Myanmar,Philippines, Singapore, Thailand, and Vietnam took part in the summit. The purpose of the summit wasto strengthen the mutual cooperation among the youth leaders from India and the ASEAN countries.Hence, the 2019 summit is a step to facilitate Socio-cultural exchanges between the countries to furtherstrengthen the values of tolerance, pluralism and diversity.

92. Under which section of the Banking Regulation Act of 1949, RBI has slapped Rs 1 crore penaltyon SBI for violating norms?[A] Section 35A[B] Section 47A[C] Section 44A[D] Section 49A

Correct Answer: B [Section 47A]Notes:The Reserve Bank of India has exercised its powers conferred under Section 47A of the BankingRegulation Act of 1949 & imposed a penalty of Rs 1 crore on the State Bank of India (SBI) for notmonitoring the end use of funds in respect of one of its borrowers. However, SBI did not share details ofthe borrower and the loan amount given to the borrower.

93. RBI has recently raised the limit for collateral-free agriculture loans from Rs 1 Lakh to ____?[A] Rs 1.5 lakh[B] Rs 1.6 lakh[C] Rs 1.7 lakh[D] Rs 1.4 lakh

Correct Answer: B [Rs 1.6 lakh]Notes:The Reserve Bank of India (RBI) has recently raised the limit for collateral-free agriculture loans to Rs 1.6lakh from Rs. 1 lakh to enhance coverage of small and marginal farmers in the formal credit system.Currently, the banks are mandated to extend collateral-free agricultural loans up to Rs. 1 lakh. This limitwas fixed in the year 2010. The RBI also proposed to set up an internal working group to review various

sri vishnu charan | [email protected] |

https://t.me/PDF4Examshttps://t.me/IAS201819 https://t.me/PDF4Exams

https://t.me/TheHindu_Zone_official

Page 116: Current Affairs -February 1-15, 2019 · 15-02-2019  · 10, theme: ‘Sadak Suraksha-Jeevan Raksha’ National Testing Agency (NTA) launches mobile app through which students can

Current Affairs [PDF] -February 1-15, 2019

© 2019 GKToday | All Rights Reserved | https://www.gktoday.in 116

issues plaguing farm credit, such as regional disparity, extent of coverage and limited capital formationfrom such credit. As per interim Budget of 2019, the Government of India (GoI) will also launch the‘Pradhan Mantri Kisan Samman Nidhi (PM-KISAN) programme to provide assured income support tosmall and marginal farmers. Under this, vulnerable landholding farmer-families, having cultivable landup to two hectares, will be provided direct income support at the rate of Rs. 6,000 per year. This incomesupport will be transferred directly into the bank accounts of beneficiary farmers in three equalinstalments of Rs. 2,000 each. The programme will be funded by the Government of India (GoI).

94. The first-ever World Pulses Day (WPD) is observed on which of the following dates?[A] February 11[B] February 09[C] February 08[D] February 10

Correct Answer: D [February 10]Notes:The first-ever World Pulses Day (WPD) is observed on 10th February to raise awareness about themultiple benefits of eating pulses. Pulses are a type of leguminous crop that are harvested solely for thedry seed. Dried beans, lentils and peas are the most commonly known and consumed types of pulses.Pulses do not include crops that are harvested green (e.g. green peas, green beans)—these are classifiedas vegetable crops. In 2018, the UN General Assembly (UNGA) designated 10 February as World PulsesDay (WPD) to keep alive the positive momentum surrounding these healthy, nutritious, protein-rich,nitrogen-fixing legumes after FAO’s successful 2016 International Year of Pulses (IYP) Campaign.

95. Who authored the book “Law, Justice and Judicial Power – Justice P N Bhagwati’s Approach”?[A] Mool Chand Sharma[B] Rupinder Singh Suri[C] Shashi Tharoor[D] Soli J Sorabjee

Correct Answer: A [Mool Chand Sharma]Notes:The book titled “Law, Justice and Judicial Power – Justice P N Bhagwati’s Approach” has been authoredby Professor (Dr.) Mool Chand Sharma. It was formally released by Chief Justice of India, Justice RanjanGogoi at a function held in New Delhi. The first edition of the book was received by President RamnathGovind. Justice Bhagwati has been called the father of Public Interest Litigation (PIL) in India. The bookis a collection of 24 essays written in honour of Justice Bhagwati, celebrating his judicial work andlandmark judgments as well as his work towards introducing PIL in India.

96. The Himalayan Cloud Observatory has been set up in which district of Uttarakhand?[A] Bageshwar[B] Pithoragarh[C] Tehri Garhwal[D] Rudraprayag

Correct Answer: C [Tehri Garhwal ]Notes:The Himalayan Cloud Observatory has recently been set up in the SRT campus Chamba of TehriGarhwal district in Uttarakhand to receive forecast and monitor cloud burst incidents in the Himalayanregion and help minimise the damage. This is the second observatory in the country to monitor cloudactivities by the Indian Science and Technology (IST) Department and Indian Institute of Technology(IIT) Kanpur that can function in high-altitudes. The cloudburst incidents are very frequent in the hillyareas of Uttarakhand and resulted in a huge loss of life. The Himalayan Cloud Observatory has recentlyprepared a six-month data while being in the testing period, so that after the completion of the testingperiod, cloud bursting incidents in future could be easily predicted and the severe loss of lives andproperty caused to the localities due to this could be stopped.

97. Which country has decided to include Hindi as the third official language in courts?[A] Iran

sri vishnu charan | [email protected] |

https://t.me/PDF4Examshttps://t.me/IAS201819 https://t.me/PDF4Exams

https://t.me/TheHindu_Zone_official

Page 117: Current Affairs -February 1-15, 2019 · 15-02-2019  · 10, theme: ‘Sadak Suraksha-Jeevan Raksha’ National Testing Agency (NTA) launches mobile app through which students can

Current Affairs [PDF] -February 1-15, 2019

© 2019 GKToday | All Rights Reserved | https://www.gktoday.in 117

[B] UAE[C] Saudi Arabia[D] Oman

Correct Answer: B [UAE]Notes:The Abu Dhabi Judicial Department (ADJD) has decided to include Hindi as the third official languageused in its courts, alongside Arabic and English, as part of a move designed to improve access to justice.The move will help the Hindi speakers to learn about litigation procedures, their rights and dutieswithout a language barrier. It is an estimated that 3.3 million Indian expats live in United Arab Emirates(UAE), and a large number of them are blue collared workers. Hence, this is an excellent step taken byAbu Dhabi that will certainly improve access to justice for India’s blue collar workers. It also fitsperfectly into the wider agenda of UAE’s Year of Tolerance”.

98. Which city is the venue of the 13th edition of International Oil and Gas Conference‘PETROTECH-2019’?[A] Greater Noida[B] Dehradun[C] Shimla[D] Bhopal

Correct Answer: A [Greater Noida]Notes:On 11th of February, the 13th edition of International Oil and Gas Conference ‘PETROTECH-2019’ hasinaugurated by Prime Minister Narendra Modi at India Expo Mart in Greater Noida, Uttar Pradesh. Thethree-day biennial event is organised under the aegis of the Union Ministry of Petroleum & Natural Gas.The conference will showcase the recent market and investor friendly developments in India’s oil andgas sector. Over 95 Energy Ministers from partner countries and 7000 delegates from around 70countries are expected to be part of PETROTECH -2019.

99. Which country’s President has been elected as the chairman of the African Union (AU)?[A] Zambia[B] Egypt[C] South Africa[D] Ethiopia

Correct Answer: B [Egypt]Notes:Egyptian President, Abdel-Fattah el-Sissi has been elected as the chairman of the African Union (AU) atthe continental body’s summit in Ethiopia. El-Sisi’s election brought to an end the one-yearchairmanship of Rwanda’s President Paul Kagame. The Egyptian leader is expected to focus on the fightagainst armed groups on the continent and rebuilding efforts of countries recovering from conflict. It isthe first time since the AU was founded in 2002 that Egypt assumes the top seat of the body. The AfricanUnion (AU) is a continental union consisting of 55 countries of the continent of Africa. The purpose ofAU is to achieve greater unity and solidarity between the African countries and Africans. The primarylanguages spoken include Arabic, English, French and Portuguese and the languages of Africa. Withinthe African Union, there are official bodies such as the Peace and Security Council and the Pan-AfricanParliament.

100. Dikkibandi stadium, which is in news recently, is located in which of the following states?[A] Nagaland[B] Tripura[C] Meghalaya[D] Arunachal Pradesh

Correct Answer: C [Meghalaya ]Notes:The Meghalaya Cabinet has recently decided to rename the Dikkibandi stadium in Tura town after

sri vishnu charan | [email protected] |

https://t.me/PDF4Examshttps://t.me/IAS201819 https://t.me/PDF4Exams

https://t.me/TheHindu_Zone_official

Page 118: Current Affairs -February 1-15, 2019 · 15-02-2019  · 10, theme: ‘Sadak Suraksha-Jeevan Raksha’ National Testing Agency (NTA) launches mobile app through which students can

Current Affairs [PDF] -February 1-15, 2019

© 2019 GKToday | All Rights Reserved | https://www.gktoday.in 118

former Lok Sabha Speaker Purno A Sangma. The state Cabinet meeting presided over by Chief MinisterConrad K Sangma approved the recommendations received from the state naming authority tasked withthe job to name roads, and institutions after personalities. PA Sangma is also a former chief ministerofMeghalaya. He was elected from the Tura Lok Sabha seat a number of times. Conrad K Sangma andJames K Sangma are sons of Purno A Sangma. Formerly, Dikkibandi stadium was known as the MPstadium. The state Cabinet has also decided to name a road leading from Mawlai-Mawroh to Madan inhonour of Saint Teressa.

101. Jamie Chadwick, who became the first female driver to win MRF Challenge title, is fromwhich country?[A] United States[B] England[C] France[D] Belgium

Correct Answer: B [England ]Notes:In February 2019, Jamie Chadwick from England has scripted history by becoming the first femalemotor racing driver to clinch the MRF Challenge Championship title after scoring a triple win in thefinal round of the season in Chennai, Tamil Nadu. She is best known for becoming the first ever femaleand youngest winner of the British GT Championship in 2015.

102. Which Indian filmmaker is the recipient of the Lifetime Achievement Award (LTA) by theAcademy of Canadian Cinema and Television?[A] Mira Nair[B] Shyam Benegal[C] Deepa Mehta[D] Mani Ratnam

Correct Answer: C [Deepa Mehta]Notes:Acclaimed Indo-Canadian filmmaker Deepa Mehta will be given the Lifetime Achievement Award by theAcademy of Canadian Cinema and Television (ACCT). She is best known for her Elements Trilogy — Fire(1996), 1947 Earth (1999) and Water (2005). Mehta’s “Earth” was submitted by India as its official entryfor the Academy Award for Best Foreign Language Film, and “Water” was Canada’s official entry forAcademy Award for Best Foreign Language Film. “Earth” was the first to receive an Oscar nomination.The Canadian screen awards are presented annually by the ACCT to recognise excellence in Canadianfilm, English-language television, and digital media productions. In May 2012, Mehta received thegovernor general’s Performing Arts Award for Lifetime Artistic Achievement — Canada’s highesthonour in the performing arts.

103. Which city is the venue of the World Government Summit (WGS-2019)?[A] New Delhi[B] Dubai[C] Paris[D] London

Correct Answer: B [Dubai]Notes:On 10th of February, the 7th edition of World Government Summit (WGS-2019) was held at MadinatJumeirah in Dubai, UAE to deliver a road map towards a more prosperous future for all nations. The 3-day event was held under the patronage of Sheikh Mohammed bin Rashid, Vice President and Ruler ofDubai. The focus of the summit was on 7 key topics: the future of technology, the future of health andquality of life, the environment and climate change, education and the labour market, trade andinternational co-operation, societies and politics and information and communication betweengovernments and society. More than 4,000 decision-makers from 140 countries – including heads ofstate, ministers and business leaders – are participating the event. The summit is a knowledge exchangeplatform at the intersection of government, futurism, technology, and innovation. It functions as athought leadership platform and networking hub for policymakers, experts and pioneers in human

sri vishnu charan | [email protected] |

https://t.me/PDF4Examshttps://t.me/IAS201819 https://t.me/PDF4Exams

https://t.me/TheHindu_Zone_official

Page 119: Current Affairs -February 1-15, 2019 · 15-02-2019  · 10, theme: ‘Sadak Suraksha-Jeevan Raksha’ National Testing Agency (NTA) launches mobile app through which students can

Current Affairs [PDF] -February 1-15, 2019

© 2019 GKToday | All Rights Reserved | https://www.gktoday.in 119

development. It is also showcases innovations, best practices, and smart solutions to inspire creativity totackle these future challenges.

104. What is the theme of the 2019 International Day of Women and Girls in Science?[A] Transforming the World: Parity for All[B] Investment in Women and Girls in Science for Inclusive Green Growth[C] Gender Equality and Parity in Science for Peace and Development[D] Women’s participation in policy-making processes

Correct Answer: B [Investment in Women and Girls in Science for Inclusive Green Growth]Notes:The International Day of Women and Girls in Science is celebrated every year on 11th of February topromote full and equal access to and participation in science for women and girls. The day is a reminderthat women and girls play a critical role in science and technology communities and that theirparticipation should be strengthened. The 2019 theme is “Investment in Women and Girls in Science forInclusive Green Growth”.

105. The multinational maritime exercise ‘AMAN 19’ was conducted in which of the followingcountries?[A] Pakistan[B] India[C] Sri Lanka[D] Myanmar

Correct Answer: A [Pakistan ]Notes:On February 8, the 5-day multinational maritime exercise ‘AMAN 19’ has commenced at Pakistan NavyDockyard near the coastal city of Karachi with the slogan “Together for Peace”. The flags of 46 countriesincluding Pakistan were hoisted and the contingents of Pakistan Navy presented a march-past. Theprincipal purpose of the exercise is to provide a forum for understanding of each other’s maritimeconcepts and operational cultures and come up with ways and means to combat common threats at sea.The exercise is aimed at fostering maritime cooperation, promoting a safe and secure maritimeenvironment for regional and global stability and for preserving oceans which is the common heritage ofmankind.

106. Which union ministry is organizing the first-ever Annual Conference of Media Units?[A] Ministry of Information & Broadcasting[B] Ministry of Human Resource Development[C] Ministry of Science and Technology[D] Ministry of Electronics and Information Technology

Correct Answer: A [Ministry of Information & Broadcasting]Notes:The first-ever Annual Conference of Media Units will be organized by Union Ministry of Information &Broadcasting at Vigyan Bhawan in New Delhi on 13th of February 2019. The purpose of the Conference isto give a unique platform to the officers of the Indian Information Service (IIS) working under differentMedia Units to converge at an All-India level where they can discuss and deliberate upon the emergingareas of the evolving communication paradigm. Through the Conference, the Ministry aims tostrengthen synergy between the Media Units in order to ensure last mile delivery, while simultaneouslybrainstorm the use of new technology in communication. The participants will also discuss the ways ofusing Social Media to reach target audience groups and push the boundaries of communication outreachin terms of geographical and cultural barriers.

107. The Hollong Modular Gas Processing Plant (HMGPP), which is in news recently, is located inwhich state?[A] Odisha[B] Tripura[C] Nagaland

sri vishnu charan | [email protected] |

https://t.me/PDF4Examshttps://t.me/IAS201819 https://t.me/PDF4Exams

https://t.me/TheHindu_Zone_official

Page 120: Current Affairs -February 1-15, 2019 · 15-02-2019  · 10, theme: ‘Sadak Suraksha-Jeevan Raksha’ National Testing Agency (NTA) launches mobile app through which students can

Current Affairs [PDF] -February 1-15, 2019

© 2019 GKToday | All Rights Reserved | https://www.gktoday.in 120

[D] Assam

Correct Answer: D [Assam]Notes:On 9th of February, Prime Minister Narendra Modi formally inaugurated and dedicated to the Nationthe Hollong Modular Gas Processing Plant (HMGPP) of Hindustan Oil Exploration Co. Ltd. (HOEC) atDirok Field in Assam. Initially, the HMGPP will have a capacity to process 35 million standard cubic feetper day of gas equivalent to 15% of Assam gas production and 800 barrels of condensate. The plant islocated in Margherita, Tinsukia district of Assam. The gas produced from this field will be supplied toconsumers in the North-East region, including Brahmaputra Cracker and Polymer Ltd. Dirok Field isbeing developed in phases by HOEC along with its JV partners – Oil India Ltd. and Indian OilCorporation.

108. The 3rd edition of Swachh Shakti-2019 was held at which of the following cities?[A] Dehradun[B] Lucknow[C] Gandhinagar[D] Kurukshetra

Correct Answer: D [Kurukshetra]Notes:The third edition of Swachh Shakti-2019 was held at Kurukshetra, Haryana on February 12, which wasattended by woman sarpanches and women associated with the ‘Swachhta’ campaign from across thecountry. It is a national event to recognize the leadership role played by rural women in Swachh BharatMission towards achieving a Clean and Open Defecation Free (ODF) country by 2nd October 2019.Around 15,000 women participated in this year’s Swachh Shakti event aimed at empowering the women.Prime Minister Narendra Modi distributed the ‘Swachh Shakti 2019’ awards too. The Ministry ofDrinking Water and Sanitation in association with the Govt. of Haryana organized the SwachhShakti-2019. Best practices from grass root level in the rural areas for Swachh Bharat will be shared bythem. The event will showcase the achievements of Swachh Bharat and the recently conducted SwachhSunder Shauchalay, (neat and clean toilet) – a unique and first of its kind in the world campaign.

109. On which date, the National Unani Day (NUD-2019) was celebrated in India?[A] February 10[B] February 11[C] February 12[D] February 13

Correct Answer: B [February 11]Notes:The National Unani Day (NUD) is celebrated every year on 11th February to mark the birth anniversaryof great Unani researcher Haqim Ajmal Khan. He was an eminent Indian Unani physician who wasversatile genius, freedom fighter, educationist and founder of scientific research in Unani Medicine. Aspart of 3rd Unani Day 2019 celebration, the two-day National Conference on Unani Medicine was held atNew Delhi with theme ‘Unani Medicine for Public Health’. It was organized by the Central Council forResearch in Unani Medicine (CCRUM). The conference highlighted the important role played by UnaniMedicine in public health especially in combating NCDs, lifestyle disorders and various chronic diseasesby providing cost effective treatment and quality products for patient healthcare. About 1300 delegates,resource persons, academicians, researchers and industry representatives participated in theconference.

110. Which Indian Historian has won the 2019 Dan David Prize?[A] Sanjay Subrahmanyam[B] Irfan Habib[C] M. G. S. Narayanan[D] Sumit Sarkar

Correct Answer: A [Sanjay Subrahmanyam]Notes:

sri vishnu charan | [email protected] |

https://t.me/PDF4Examshttps://t.me/IAS201819 https://t.me/PDF4Exams

https://t.me/TheHindu_Zone_official

Page 121: Current Affairs -February 1-15, 2019 · 15-02-2019  · 10, theme: ‘Sadak Suraksha-Jeevan Raksha’ National Testing Agency (NTA) launches mobile app through which students can

Current Affairs [PDF] -February 1-15, 2019

© 2019 GKToday | All Rights Reserved | https://www.gktoday.in 121

Renowned Indian historian Sanjay Subrahmanyam has been awarded the Israel’s prestigious Dan DavidPrize 2019 for his work on inter-cultural encounters between Asians, Europeans and people of North andSouth America during the early modern era. He won the award in the category of ”past time dimension”for his work in macro history. He will share the $1-million prize money with historian Prof. KennethPomeranz of University of Chicago. The prize is named after philanthropist Dan David and administeredby Israel’s Tel Aviv University. It is awarded annually to those who have made outstanding scientific,technological and humanistic accomplishments in fields representing the past, present and future ofhuman achievement. The award ceremony will take place in Tel Aviv in May 2019.

111. Which city is the venue of the CREDAI YouthCon -19?[A] Ahmedabad[B] Varanasi[C] Kolkata[D] New Delhi

Correct Answer: D [New Delhi]Notes:On 13th February 2019, Prime Minister Narendra Modi addressed over 3000 young Real Estatedevelopers at the CREDAI YouthCon -19 at the Talkatora Stadium in New Delhi. The 2-day conferencewas organized by Confederation of Real Estate Developers’ Association of India (CREDAI). The annualconference will bring together the next generation of real estate builders and developers to enlightentheir young minds and share their thinking and approach. Established in 1999, CREDAI is the apex bodyrepresenting real-estate developers from over 200 cities in the country.

112. What is the theme of the 2019 World Radio Day (WRD)?[A] Radio and Sports[B] Radio is You[C] Dialogue, Tolerance and Peace Broadcasts[D] Securing freedom of expression and journalists’ safety

Correct Answer: C [Dialogue, Tolerance and Peace Broadcasts]Notes:The World Radio Day (WRD) is observed every year on the 13th of February to celebrate radio as a forumfor information and entertainment, as a bridge of communication for remote communities and its role inempowering people. It is also a day to remember the unique power of radio which touches lives andbrings people together from every corner of the globe. The day was proclaimed on 3rd November 2011 byUNESCO’s 36th General Conference. The 2019 theme “Dialogue, Tolerance and Peace Broadcasts”provides a platform for dialogue and democratic debate over issues, such as migration or violenceagainst women, can help to raise awareness among listeners and inspire understanding for newperspectives in paving the way for positive action.

113. Vijaya Bapineedu, who passed away recently, was the prominent film director-producer ofwhich regional cinema?[A] Malayalam[B] Odia[C] Tamil[D] Telugu

Correct Answer: D [Telugu ]Notes:Vijaya Bapineedu (83), the prominent Telugu film director-producer, has passed away in Hyderabad,Telangana on February 12, 2019. Hails from West Godavari district in Andhra Pradesh, he was known forTelugu films like Gang Leader, Khiladi No. 786 and Magadheerudu. He is also credited for Chiranjeevi’ssuccessful career in films today.

114. Which state government has recently banned the Popular Front of India (PFI)?[A] Jharkhand[B] Tamil Nadu

sri vishnu charan | [email protected] |

https://t.me/PDF4Examshttps://t.me/IAS201819 https://t.me/PDF4Exams

https://t.me/TheHindu_Zone_official

Page 122: Current Affairs -February 1-15, 2019 · 15-02-2019  · 10, theme: ‘Sadak Suraksha-Jeevan Raksha’ National Testing Agency (NTA) launches mobile app through which students can

Current Affairs [PDF] -February 1-15, 2019

© 2019 GKToday | All Rights Reserved | https://www.gktoday.in 122

[C] J & K[D] Arunachal Pradesh

Correct Answer: A [Jharkhand ]Notes:The Jharkhand government has recently banned the Popular Front of India (PFI) in the state to curb itsanti-national activities and for keeping links with terror outfits like ISIS. According to the governmentnotification, the PFI is banned with immediate effect under the Criminal Law (Amendment) Act, 1908.The decision to ban PFI has been taken after a review that its activities have increased and is dangerousto the state and the nation and it has the power to disrupt peace, communal harmony and secularstructure. As per notification, the PFI has been involved in spreading social division, anti-India and pro-Pakistan slogans, and links with ISIS and JMB. The PFI is an extremist and militant Islamicfundamentalist organisation in India, formed as a successor to National Development Front (NDF) in2006.

115. Which Indian armed force has conducted ‘Exercise Topchi 2019’?[A] Indian Air Force[B] Indian Army[C] Indian Navy[D] Indian Coast Guard

Correct Answer: B [Indian Army]Notes:Indian Army has recently demonstrated its artillery firepower by using ultra light Howitzers andindigenous Swathi weapon-locating radar at the annual ‘Exercise Topchi 2019’ at the vast firing rangesat Deolali Camp near Nasik, Maharashtra. The aviation and surveillance capabilities were also showcasedin the Exercise. In addition to the gun fire, the display of rockets, missiles, surveillance and targetacquisition radars, remotely piloted aircraft and hi-tech equipments was awe-inspiring.

116. Which city was the venue of the first-ever LAWASIA Human Rights Conference 2019?[A] New Delhi[B] Ranchi[C] Jaipur[D] Pune

Correct Answer: A [New Delhi]Notes:On 9th February, the first-ever LAWASIA Human Rights Conference 2019 was held in New Delhi withtheme “State Power, Business and Human Rights: Contemporary Challenges”. The 2-day conference washosted by LAWASIA, in association with the Bar Association of India (BAI). The conference focussed ontopics ranging from “Gender, Sexuality and Human Rights” to “Climate Change, Water Conflicts andHuman Rights”. It provided a unique opportunity for lawyers and associated professional members toexchange insights and expertise on topics of significance. LAWASIA is a regional association of lawyers,judges, jurists and legal organisations, which advocates for the interests and concerns of the Asia Pacificlegal profession.

117. India and which country has recently launched initiative to combat Marine Pollution?[A] Iceland[B] Norway[C] France[D] Japan

Correct Answer: B [Norway ]Notes:The Union Ministry of Environment & Forests (MoEF) has recently signed a letter of intent with theNorwegian government to launch the India-Norway Marine Pollution Initiative. In partnership, both thecountries will share experiences and competence, and collaborate on efforts to develop clean and healthyoceans, sustainable use of ocean resources and growth in the blue economy. The first Joint initiative ofboth the governments will combat marine pollution, which is one of the fastest growing environmental

sri vishnu charan | [email protected] |

https://t.me/PDF4Examshttps://t.me/IAS201819 https://t.me/PDF4Exams

https://t.me/TheHindu_Zone_official

Page 123: Current Affairs -February 1-15, 2019 · 15-02-2019  · 10, theme: ‘Sadak Suraksha-Jeevan Raksha’ National Testing Agency (NTA) launches mobile app through which students can

Current Affairs [PDF] -February 1-15, 2019

© 2019 GKToday | All Rights Reserved | https://www.gktoday.in 123

concerns. The initiative will seek to support local governments in implementing sustainable wastemanagement practices, develop systems for collecting and analyzing information about sources andscope of marine pollution and improve private sector investment. The Indian and Norwegiangovernments will also work towards beach clean-up efforts, awareness raising campaigns and pilotproject using plastic waste as fuel substitution for coal in cement production and developingframeworks for deposit schemes.

118. What is the theme of the 2019 National Productivity Week (NPW)?[A] Circular Economy for Productivity & Sustainability[B] Industry 4.0 Leapfrog Opportunity for India[C] From waste to profits-through reduce, recycle and reuse[D] Ease of Doing Business for higher productivity

Correct Answer: A [Circular Economy for Productivity & Sustainability]Notes:The National Productivity Council (NPC) has started the celebration of its 61st Foundation Day on 12thFebruary with the theme “Circular Economy for Productivity & Sustainability”. NPC observedfoundation day as National Productivity Day (NPD) and the National Productivity Week (NPW) fromFebruary 12-18, 2019. The 2019 theme represents a unique opportunity for circular business model forMake, Use & Return. It presents an opportunity for long term economic prospects and regeneration ofmaterials. The circular economy follows the principle of preservation and enhancement of naturalcapital by controlling finite stocks and balancing renewable resource flows. It has the potential toincrease productivity and create jobs, whilst reducing carbon emissions and preserving valuable rawmaterials.

119. Which state government has recently launched online clearance mechanism forentrepreneurs?[A] Kerala[B] Assam[C] Uttar Pradesh[D] Rajasthan

Correct Answer: A [Kerala ]Notes:The K-SWIFT (Kerala Single-window Interface for Fast, Transparent Clearances) has been launched bychief minister Pinarayi Vijayan at ASCEND Kerala 2019. The aim of the online clearance mechanism is tomake the state a top investment destination and empower entrepreneurs to launch their enterprises in aspeedy and hassle-free manner. K-SWIFT has been developed to simplify and expedite issuance ofclearances from departments/agencies for launching enterprises in the state. Here, the proposals ofentrepreneurs will get clearance within 30 days of application.

120. Who has been named EY Entrepreneur of the Year 2018?[A] Satyanarayana Chava[B] Siddhartha Lal[C] Karan Bhagat[D] Kishore Biyani

Correct Answer: B [Siddhartha Lal]Notes:Siddhartha Lal, MD & CEO of Eicher Motors Ltd, was named the EY Entrepreneur of the Year 2018 at the20th edition of the EOY Awards in India. He is recognised for engineering the operational and financialturnaround of the company and reviving the Royal Enfield brand. Lal will now represent India at the EYWorld Entrepreneur of the Year Award (WEOY) in Monte Carlo from 6 – 8 June 2019. The EntrepreneurOf The Year awards aims to honour entrepreneurs who with their creative ideas and exceptionalenterprises have emerged as “Builders of a better India. In addition, Chairman of Wipro group AzimPremji was honoured with the lifetime achievement award for leading Wipro’s transformation from avegetable oil business into a multi-business technology organization & for putting the Indianinformation technology industry on the global map.

sri vishnu charan | [email protected] |

https://t.me/PDF4Examshttps://t.me/IAS201819 https://t.me/PDF4Exams

https://t.me/TheHindu_Zone_official

Page 124: Current Affairs -February 1-15, 2019 · 15-02-2019  · 10, theme: ‘Sadak Suraksha-Jeevan Raksha’ National Testing Agency (NTA) launches mobile app through which students can

Current Affairs [PDF] -February 1-15, 2019

© 2019 GKToday | All Rights Reserved | https://www.gktoday.in 124

121. Which of the following instruments has been declared heritage musical instrument of Goa?[A] Kanjira[B] Idakka[C] Ghumot[D] Chenda

Correct Answer: C [Ghumot]Notes:Ghumot, Goa’s indigenous traditional percussion instrument made from an earthen vessel, will benotified as a heritage musical instrument of Goa. The instrument was banned due to the use of the skinof the endangered monitor lizard for the drum membrane. In recent years, ghumot makers have startedusing goat skin instead. The ban is applicable to the use of any animal listed in the Schedule I of theWildlife (Protection) Act of 1972, and that the goat is not one of them. The Ghumot is part of religiousand folk music of both Hindus and Christians in Goa. It is an essential instrument for the Ganeshfestival, during arti and in Suvari Vadan orchestra that performs at Goan temples during spring.Ghumot is part of the Mando, a musical form of the Goan Catholics that combines elements of bothIndian and Western music. It is also part of the Zagor folk dance and Dulpod.

122. Which political leader has authored the book ‘Undaunted: Saving the Idea of India’?[A] Shashi Tharoor[B] P Chidambaram[C] Arun Jaitley[D] Sushma Swaraj

Correct Answer: B [P Chidambaram]Notes:The book titled ‘Undaunted: Saving the Idea of India’ has been authored by former finance minister PChidambaram, which was recently released by former vice president Hamid Ansari. The book claimedthat fear rules the country today and there is a danger that the Constitution will be replaced by adocument inspired by Hindutva. According to the author, an economy derailed can be put back on therails and a society divided can be healed and united but one thing cannot be fixed if it is broken, and thatis the Constitution and the constitutional values embodied in that document. At present, every value ofthe Constitution is under attack — freedom, equality, liberalism, secularism, privacy, scientific temper,etc.

123. Which of the following will be the official mascot for 39th National Games 2022?[A] Sumatran serow[B] Gayal[C] Clouded Leopard[D] Ox

Correct Answer: C [Clouded Leopard]Notes:Meghalaya’s state animal – Clouded Leopard will be the official mascot for the 39th National Games2022, which is coinciding with the 50th year of the creation of the state. This decision was taken at thefirst National Games 2022 executive committee meeting chaired by Meghalaya Sports and Youth AffairsMinister . Meghalaya will be the third northeastern state after Manipur and Assam hosted the NationalGames in 1999 and 2007, respectively. It had co-hosted the 2016 South Asian Games with itsneighbouring state Assam. An estimated 14,450 athletes and officials are expected to visit Meghalayaduring the 2022 National Games. Clouded leopard (Neofelis nebulosa) is one of the wild cats that livethroughout the forests of Garo, Khasi and Jaintia Hills. Since 2008, it is listed as Vulnerable on the IUCNRed List.

124. The 3rd Indo-German Environment Forum was held in which of the following cities?[A] New Delhi[B] Pune[C] Hyderabad[D] Panaji

sri vishnu charan | [email protected] |

https://t.me/PDF4Examshttps://t.me/IAS201819 https://t.me/PDF4Exams

https://t.me/TheHindu_Zone_official

Page 125: Current Affairs -February 1-15, 2019 · 15-02-2019  · 10, theme: ‘Sadak Suraksha-Jeevan Raksha’ National Testing Agency (NTA) launches mobile app through which students can

Current Affairs [PDF] -February 1-15, 2019

© 2019 GKToday | All Rights Reserved | https://www.gktoday.in 125

Correct Answer: A [New Delhi]Notes:On 13th Feb, the 3rd Indo-German Environment Forum was held in New Delhi with the theme “CleanerAir, Greener Economy”. The one-day event focuses on challenges, solutions and necessary frameworkconditions of air pollution control, waste management and circular economy as well as implementationof NDCs and SDGs based on Paris Agreement and Agenda 2030 of UN respectively. Around 250representatives of ministries, business and science as well as non-governmental organizationsparticipated in the forum, which is organized by the two environment ministries in cooperation with theAsia-Pacific Committee of German Business and the Federation of Indian Chamber of Commerce andIndustry (FICCI). The Forum provided a platform for the bilateral exchange of high-level policy-makersand other key players on international environmental and climate policy and cooperation between thetwo countries.

125. Which state assembly has passed bill to end the minimum education criterion for panchayatand civic poll candidates?[A] Chhattisgarh[B] Rajasthan[C] Madhya Pradesh[D] Karnataka

Correct Answer: B [Rajasthan ]Notes:The Rajasthan Assembly has recently passed two Bills which seek to end the minimum educationcriterion for panchayat and civic poll candidates. The House passed by voice vote the RajasthanPanchayati Raj (Amendment) Bill, 2019 and the Rajasthan Municipality (Amendment) Bill, 2019. Theprevious Vasundhara Raje-led government had introduced education criterion in 2015 which required acandidate to pass Class X for contesting zila parishad, panchayat samiti and municipal elections.Defending the bill, Panchayati Raj Minister Sachin Pilot stated that the previous Act was against thebasics of the Constitution. Society cannot be divided on the basis of education. He further stated thateducation criterion should first be introduced for contesting Assembly and parliamentary elections.

126. The 5th International Dam Safety Conference – 2019 was the initiative of which internationalorganisation?[A] IMF[B] World Bank[C] AIIB[D] ADB

Correct Answer: B [World Bank]Notes:The 5th International Dam Safety Conference – 2019 was held in Bhubaneswar, Odisha during 13 and 14February 2019. It is the joint initiative of the Government of India, Government of Odisha and the WorldBank (WB) under aegis of the ongoing World Bank assisted Dam Rehabilitation and Improvement Project(DRIP) as a part of institutional strengthening. Dam Safety Conferences are being organized as anannual event in different DRIP States in collaboration with the Implementing Agencies and leadingacademic institutes to provide a common platform for all stakeholders including non-DRIP States.Presently, DRIP covers rehabilitation of 198 large dam projects located in seven States namelyJharkhand, Karnataka, Kerala, Madhya Pradesh, Odisha, Tamil Nadu, and Uttarakhand. Globally, Indiaranks third after China and the USA in terms of the number of large dams with 5264 large dams inoperation and 437 large dams under construction. The total storage capacity of the impounded water bythese dams is about 283 billion cubic meters (BCM).

127. Which state government has announced a universal old age pension scheme – MukhyamantriVridhajan Pension Yojna (MVPY)?[A] Himachal Pradesh[B] Odisha[C] Jharkhand

sri vishnu charan | [email protected] |

https://t.me/PDF4Examshttps://t.me/IAS201819 https://t.me/PDF4Exams

https://t.me/TheHindu_Zone_official

Page 126: Current Affairs -February 1-15, 2019 · 15-02-2019  · 10, theme: ‘Sadak Suraksha-Jeevan Raksha’ National Testing Agency (NTA) launches mobile app through which students can

Current Affairs [PDF] -February 1-15, 2019

© 2019 GKToday | All Rights Reserved | https://www.gktoday.in 126

[D] Bihar

Correct Answer: D [Bihar ]Notes:On 14th of February, the Bihar government announced a universal old age pension scheme –Mukhyamantri Vridhajan Pension Yojna (MVPY) for all people above 60 years of age. The scheme willcome into force from the 1st of April 2019. Except those senior citizens who have retired fromgovernment service, all will be eligible for Rs. 400 monthly pension irrespective of caste, religion orcommunity. Currently old age pension scheme is applicable to only the BPL category. The Chief Ministeralso announced a pension of Rs. 6,000 for journalists above 60 years of age. Those who have been inregular service in the field of media and not getting any other pension will be eligible for Bihar PatrakarSamman Yojana (BPSY). This scheme will also be implemented from the 1st of April 2019.

128. The banned poem ‘Khooni Vaisakhi’ on Jallianwala massacre will now be published in English.It was authored by whom?[A] Balwant Gargi[B] Waris Shah[C] Nanak Singh[D] Mohan Singh

Correct Answer: C [Nanak Singh]Notes:The poem ‘Khooni Vaisakhi’ was written by acclaimed Punjabi writer Nanak Singh on the JallianwalaBagh massacre, which was banned by the British after its publication in 1920. The poem was a scathingcritique of the British Raj. Singh was present at Jallianwala Bagh on April 13, 1919. He was 22 years old atthe time. As the British troops opened fire on the unarmed gathering protesting against the Rowlatt Act,killing hundreds, Singh fainted and his unconscious body was piled up among the corpses. After goingthrough the traumatic experience, he proceeded to write “Khooni Vaisakhi”, a long poem that narratesthe political events in the run-up to the massacre and its immediate aftermath. Its manuscript wassubsequently lost. After long years, the poem has been rediscovered and now translated into English bythe author’s grandson and diplomat, Navdeep Suri. It will be published by HarperCollins India nextmonth to mark the centenary of the Jallianwala Bagh massacre. Nanak Singh (1897-1971) is widelyregarded as the father of the Punjabi novel. With little formal education beyond the fourth grade, hewrote an astounding 59 books and received the Sahitya Akademi Award in 1962.

129. The book ‘Sudhir Sukta’ was written by which acclaimed author, who passed away recently?[A] Vishnu Wagh[B] Mangesh Padgaonkar[C] Arun Kolatkar[D] Namdeo Dhasal

Correct Answer: A [Vishnu Wagh]Notes:Vishnu Wagh (53), the veteran writer and former deputy speaker of Goa Legislative Assembly, haspassed away in Panaji on February 14, 2019. Wagh was a member of the Goa Legislative Assembly fromSt Andre constituency between 2012 and 2017. He was the chairman of the state-run Goa Kala Academy.He has written over 20 plays in Marathi, three sangeet nataks, 18 Konkani plays and 16 one-act plays. Hehas directed over 50 plays in Konkani and Marathi. Some of his critically-acclaimed plays include”TukaAbhang Abhang”, “Suvari”, “Teen Poishancho Tiatro”, “Dharmashree”, and”Pedru Poddlo Baient”. He hasalso written six volumes of poetry. His book of Konkani poems titled ‘Sudhir Sukta’ has been embroiledin controversy since 2017 for its offensive language and content, and for portraying women in a badlight. The book questions Brahmins and has apparently stirred the volatile caste divide between Goa’sdominant Gaud Saraswat Brahmins and the majority Bahujan Goans.

130. Who has been conferred the Ashoka Chakra for year 2019?[A] Dinesh Raghu Raman[B] Nazir Ahmad Wani[C] Takht Singh[D] Bachittar Singh

sri vishnu charan | [email protected] |

https://t.me/PDF4Examshttps://t.me/IAS201819 https://t.me/PDF4Exams

https://t.me/TheHindu_Zone_official

Page 127: Current Affairs -February 1-15, 2019 · 15-02-2019  · 10, theme: ‘Sadak Suraksha-Jeevan Raksha’ National Testing Agency (NTA) launches mobile app through which students can

Current Affairs [PDF] -February 1-15, 2019

© 2019 GKToday | All Rights Reserved | https://www.gktoday.in 127

Correct Answer: B [Nazir Ahmad Wani]Notes:Lance Naik Nazir Ahmad Wani has been posthumously awarded ‘Ashok Chakra’, India’s highest peacetime gallantry award for his role in a counter-insurgency operation in Kashmir in 2018. The award waspresented by President Ram Nath Kovind to his wife Mrs. Mahajabeen at the 2019 Republic Day parade.Wani is the first Kashmiri to be conferred the Ashoka Chakra. He was also awarded Sena Medal forgallantry twice in 2007 and 2018 for his acts of valour. Wani was once a terrorist but had later turned inand joined the Army’s 162 Infantry Battalion (Territorial Army) in Jammu and Kashmir Light Infantry in2004. A resident of Cheki Ashmuji of Kulgam district in J&K, he lost his life in a counter-terror operation‘Operation Batagund’ against six terrorists in Hirapur village near Batgund in Shopian on 25thNovember 2018. Ashoka Chakra is the highest peacetime military decoration for valour, courageousaction or self-sacrifice away from the battlefield.

131. Who is the newly appointed Election Commissioner (EC) of India?[A] Shelesh Bhardwaj[B] Sushil Chandra[C] Ashutosh Saxena[D] Alok Chaudhary

Correct Answer: B [Sushil Chandra]Notes:Sushil Chandra, an IRS-IT officer of the 1980 batch, has assumed charge as the new ElectionCommissioner (EC) of India on 15th of February 2019. Prior to joining ECI, he was the Chairman of theCentral Board of Direct Taxes (CBDT).With his appointment, the Commission now has Sunil Arora as theChief Election Commissioner (CEC) and Ashok Lavasa and Mr Chandra as the two commissioners. TheCommission is expected to announce the date and the schedule for the Lok Sabha polls by the end of thismonth.

132. Who is the recipient of the Tagore Award for Cultural Harmony for year 2016?[A] Ram Sutar Vanji[B] Chhayanaut[C] Rajkumar Singhajit Singh[D] Milind Kumar

Correct Answer: A [Ram Sutar Vanji]Notes:The President of India, Ram Nath Kovind will present the Tagore Award for Cultural Harmony toRajkumar Singhajit Singh; Chhayanaut (a cultural organization of Bangladesh); and Ram Sutar Vanji forthe years 2014, 2015 & 2016 respectively on 18th of February 2019 at Pravasi Bhartiya Kendra, New Delhi.Tagore Award for Cultural Harmony was instituted by the Government of India from 2012 recognizingthe contributions made by Gurudev Rabindranath Tagore to humanity at large with his works and ideas,as part of the Commemoration of his 150thBirth Anniversary in 2012, for promoting values of CulturalHarmony. It is awarded annually and carries an amount of Rs. One Crore (convertible to foreigncurrency), a citation in a Scroll, a Plaque as well as an exquisite traditional handicraft ( or handloom)item.

133. India has recently signed Legal Agreements with which international organization for Shimlawater supply?[A] ADB[B] IMF[C] AIIB[D] World Bank

Correct Answer: D [World Bank]Notes:On 15th February, the Government of India (GoI), Government of Himachal Pradesh (GoHP) and theWorld Bank signed a $40 Million Loan Agreement to help bring clean and reliable drinking water to thecitizens of the Greater Shimla area, who have been facing severe water shortages and water-borne

sri vishnu charan | [email protected] |

https://t.me/PDF4Examshttps://t.me/IAS201819 https://t.me/PDF4Exams

https://t.me/TheHindu_Zone_official

Page 128: Current Affairs -February 1-15, 2019 · 15-02-2019  · 10, theme: ‘Sadak Suraksha-Jeevan Raksha’ National Testing Agency (NTA) launches mobile app through which students can

Current Affairs [PDF] -February 1-15, 2019

© 2019 GKToday | All Rights Reserved | https://www.gktoday.in 128

epidemics over the last few years. The Shimla Water Supply and Sewerage Service Delivery ReformProgrammatic Development Policy Loan 1 is expected to improve water supply and sanitation (WSS)services in and around the iconic hill city of Shimla. The $40 Million Loan from the International Bankfor Reconstruction and Development (IBRD), has a 4-year grace period and a maturity of 15.5 years.

134. What is the theme of the World Sustainable Development Summit (WSDS-2019)?[A] Attaining the 2030 Agenda: Delivering on Our Promise[B] Attaining the 2050 Agenda: Delivering on Our Promise[C] Attaining the 2040 Agenda: Delivering on Our Promise[D] Attaining the 2020 Agenda: Delivering on Our Promise

Correct Answer: A [Attaining the 2030 Agenda: Delivering on Our Promise]Notes:The World Sustainable Development Summit (WSDS) is The Energy and Resources Institute’s (TERI)annual flagship event. The 2019 edition of WSDS was held in New Delhi with theme ‘Attaining the 2030Agenda: Delivering on Our Promise’ on 11th-13th February. It was inaugurated by the Vice President ofIndia, M. Venkaiah Naidu. The WSDS brings together Nobel laureates, political leaders, decision-makersfrom bilateral and multilateral institutions, business leaders, high-level functionaries from thediplomatic corps, scientists and researchers, media personnel, and members of civil society; on acommon platform to deliberate on issues related to sustainable development.

135. Which global leader has won the Sustainable Development Leadership Award for year 2019 atWSDS?[A] Pawan Kumar Chamling[B] Anand Mahindra[C] Frank Bainimarama[D] James Alix Michel

Correct Answer: C [Frank Bainimarama]Notes:Frank Bainimarama, Prime Minister of Fiji, has been conferred with ‘Sustainable DevelopmentLeadership Award 2019’ at the World Sustainable Development Summit (WSDS-2019) in New Delhi. Hewon the award for his outstanding contributions towards sustainable development in Fiji. The summitreiterated India’s vision for a greener tomorrow and to focus on energy transitions, clean oceans,mobilising development finance, environmental sustainability and sustainable mobility. Over 2000delegates and speakers from over 40 countries participated in the summit. The summit was organisedby The Energy and Resources Institute (TERI). At the event, the Union Ministry of Environment, Forestsand Climate Change (MoEFCC) has collaborated with TERI to set up a Resource Efficiency Cell to helpframe blueprint for national integrated resource efficiency policy. The cell will enable inter-ministerialand inter-departmental synergy in promoting material resource efficiency in India while helping framethe blueprint of the integrated resource efficiency policy for India.

136. The e-AUSHADHI portal has been launched for online licensing of which of the followingdrugs?[A] Ayurveda[B] Unani[C] Siddha[D] All of the above

Correct Answer: D [All of the above]Notes:On 13th Feb, Minister of State (IC) for AYUSH, Shripad Yesso Naik launched the e-AUSHADHI portal foronline licensing of Ayurveda, Siddha, Unani and Homoeopathy drugs and related matters at New Delhi.The portal is aimed at increasing transparency, and improving information management, data usabilityand accountability. The acronym for new e-portal is ‘Ayurveda, Unani, Siddha and HomeopathyAutomated Drug Help Initiative’. The timelines would be fixed for processing of applications through theportal with text message and e-mail status updates at each step of the process. This portal will not onlyaid the licensing authority , manufactures and consumers, as it will provide real time information of thelicensed manufactures and their products, cancelled and spurious drugs, contact details of the

sri vishnu charan | [email protected] |

https://t.me/PDF4Examshttps://t.me/IAS201819 https://t.me/PDF4Exams

https://t.me/TheHindu_Zone_official

Page 129: Current Affairs -February 1-15, 2019 · 15-02-2019  · 10, theme: ‘Sadak Suraksha-Jeevan Raksha’ National Testing Agency (NTA) launches mobile app through which students can

Current Affairs [PDF] -February 1-15, 2019

© 2019 GKToday | All Rights Reserved | https://www.gktoday.in 129

concerned authority for specific grievances.

137. Who has been appointed as the chairman of Central Board of Direct Taxes (CBDT)?[A] Pramod Chandra Mody[B] Poonam Kishore Saxena[C] Kaiser Chand Bohra[D] Pritam Jain

Correct Answer: A [Pramod Chandra Mody]Notes:Pramod Chandra Mody, a 1982 batch Indian Revenue Service (Income Tax cadre), has been appointed asthe new chairman of Central Board of Direct Taxes (CBDT), the top policy making body of income taxdepartment,. He succeeded Sushil Chandra, who was named as an new Election Commissioner (EC) ofIndia. The CBDT Chairperson is ex-officio special secretary and is under the direct charge of the revenuesecretary of India. He is placed above the officers of the rank of Lieutenant-General, Vice-Admiral or AirMarshal, CBI Director and Deputy Comptroller and Auditor General (CAG) in the Order of Precedence.

138. Which movie has won the best film award at the 72nd Bafta Film Awards 2019?[A] The Favourite[B] A Star Is Born[C] Roma[D] Bohemian Rhapsody

Correct Answer: C [Roma]Notes:On 10th of February, the 72nd British Academy Film Awards 2019 were held at the Royal Albert Hall inLondon. Mexican filmmaker Alfonso Cuaron’s semi-autobiographical black-and-white drama “Roma”claimed the top prize of best film, alongside director and cinematography honours. “Roma” also won theBAFTA for film not in the English language. “The Favourite” dominated the ceremony with seven wins,including for leading actress (Olivia Colman), supporting actress (Rachel Weisz) and outstanding Britishfilm. First-time nominee Rami Malek won the best actor award for his portrayal of Queen frontmanFreddie Mercury in biopic “Bohemian Rhapsody”. The highest honour ‘the BAFTA fellowship’ was givento film editor Thelma Schoonmaker, a three-time Oscar winner. The special BAFTA award foroutstanding British contribution to cinema was presented to producers Elizabeth Karlsen and StephenWoolley of Number 9 Films.

139. What is the route of the “Vande Bharat Express”?[A] New Delhi to Varanasi[B] Allahabad to Hooghly[C] Shimla to Kolkata[D] Mumbai to Chennai

Correct Answer: A [New Delhi to Varanasi]Notes:India’s first engine-less Semi-High Speed Train (SHST), “Vande Bharat Express” has been flagged off byPrime Minister Narendra Modi on New Delhi-Kanpur-Allahabad-Varanasi route train from the NewDelhi Railway Station on 15th February 2019. Vande Bharat Express can run up to a maximum speed of160 kmph and has travel classes like Shatabdi Train but with better facilities. It aims to provide a totallynew travel experience to passengers. The Train will cover the distance between New Delhi and Varanasiin 8 hours and will run on all days except Mondays and Thursdays. All coaches are equipped withautomatic doors, GPS based audio-visual passenger information system, on-board hotspot Wi-Fi forentertainment purposes, and very comfortable seating. All toilets are bio-vacuum type. The lighting isdual mode, viz. diffused for general illumination and personal for every seat. Every coach has a pantrywith facility to serve hot meals, hot and cold beverages. The insulation is meant to keep heat and noise tovery low levels for additional passenger comfort. The train has 16 air-conditioned coaches of which 2 areexecutive class coaches. The total seating capacity is 1,128 passengers.

140. India has recently signed a security protection agreement with which Nordic country?[A] Denmark

sri vishnu charan | [email protected] |

https://t.me/PDF4Examshttps://t.me/IAS201819 https://t.me/PDF4Exams

https://t.me/TheHindu_Zone_official

Page 130: Current Affairs -February 1-15, 2019 · 15-02-2019  · 10, theme: ‘Sadak Suraksha-Jeevan Raksha’ National Testing Agency (NTA) launches mobile app through which students can

Current Affairs [PDF] -February 1-15, 2019

© 2019 GKToday | All Rights Reserved | https://www.gktoday.in 130

[B] Norway[C] Sweden[D] Iceland

Correct Answer: C [Sweden]Notes:India has recently signed a security protection agreement with Sweden that will enable both countriesto share classified information with each other. The agreement was signed on during Defence MinisterNirmala Sitharaman’s visit to the Nordic country. Sweden has had similar agreements with over 30countries around the world, as well as with the EU and NATO. The Nordic countries are generallyconsidered to refer to Denmark, Finland, Iceland, Norway and Sweden, including their associatedterritories (Greenland, the Faroe Islands and the Åland Islands).

sri vishnu charan | [email protected] |

https://t.me/PDF4Examshttps://t.me/IAS201819 https://t.me/PDF4Exams

https://t.me/TheHindu_Zone_official